Sie sind auf Seite 1von 300

ll

mmill

AfTRONOMY

ANALYTIC GEOMETRY
FOR

COLLEGES, UNIVERSITIES, AND


TECHNICAL SCHOOLS.

BY
E.

VV.

NICHOLS,

PROFESSOR OF MATHEMATICS IN THE VIRGINIA MILITARY


INSTITUTE.

LEACH, SHEWELL, & SANBORN,


BOSTON. NEW YORK. CHICAGO.

COPYRIGHT, iwi,

BY LEACH, SIIKWELL, & SANBOKN.

C.

J.

PETERS & SON,

TYPOGRAPHERS AND ELECTROTYPERS.


PRESS OF BERWICK

&

SMITH.

Nsf
PEEFACE.
THIS text-book

is

designed

and Technical Schools.


to prepare a

make

it

work

for

The aim

for beginners,

Colleges,

of

and

the

Universities,

author has been

at the

same time to

for the requirements

of
comprehensive
For the methods of develop
ment of the various principles he has drawn
largely upon his
sufficiently

the usual undergraduate course.

experience in the class-room.


all

home and

authors,

In the preparation of the work

foreign,

have been freely consulted.


In the first few chapters
discussion of each principle.

whose works were

available,

elementary examples follow the


In the subsequent
chapters

sets

of examples appear at intervals throughout each chapter,


and are so arranged as to partake both of the nature of a
review and an extension of the
At the
preceding
principles.

end of each chapter general


examples, involving
extended application of the
are
principles deduced,

the benefit of those

who may

a more

placed for

desire a higher course in the

subject.

The author takes pleasure


cussion

of

Surfaces,"

Mathematics

in

appears as the

He

by

in calling attention to a

and Lee University,


chapter in this work.

Washington

final

"Dis

A. L. Nelson, M.A., Professor of

which

takes pleasure also in


acknowledging his indebtedness

PREFACE.

Prof. C. S. Venable, LL.D., University


of
Prof. William Cain, C.E., University

of

to

and

Virginia, to

North

Carolina,

of Pennsylvania,
to Prof. E. S. Crawley, B.S., University

for assistance rendered in reading

and revising manuscript,

and for valuable suggestions given.


E.

LEXINGTON, YA.
January, 1893.

W. NICHOLS.

CONTENTS.
PAET

I.

PLANE ANALYTIC GEOMETRY.


CHAPTEK

I.

CO-ORDINATES.

PAGES

ARTS.
1-3.

4-6.

The Cartesian or Bilinear System.


The Polar System. Examples

CHAPTER

Examples

....

II.

LOCI.
7.

Locus of an Equation.

8.

Variables.

9.

10-16.
17, 18.

The Equation

Constants. Examples
Relationship between a Locus and

of a

...

9
10

its

Equation

Discussion and Construction of Loci.


Methods of Procedure. Examples

CHAPTER

Locus

....

11

...

11

Examples

23

III.

THE STRAIGHT LINE.

21.

The Slope Equation. Examples


The Symmetrical Equation. Examples
The Normal Equation

22.

Perpendicular Distance of a Point from a Line.

19.

20.

25
29
32

Ex

26.

amples
Equation of Line, Axes Oblique. Examples
General Equation, Ax + B?y + C =
Equation of Line passing through a Point. Examples
Ex
Equation of Line passing through Two Points.

27.

Length

23.

24.

25.

37
.

38
39

amples
of

33
35

Line joining

Two

Points.

Examples

...

41

CONTENTS.

vi
ARTS.
29.
30.

Intersection of

Two

PAGES

.......
= ......

28.

Examples

Lines.

Ax + By + C + K(A a5 + BV + e )
Examples.
Angle between Two Lines.

Ex

General

.................

amples

CHAPTER

IV.

TRANSFORMATION OF CO-ORDINATES.
31.
32.
33*

34 35

Objects

of.

............

Illustration

Examples
to a Parallel System.
an Oblique System. Rectangular
Rectangular System to
also Rectangular. Examples.
System to Another System
Polar
a
to
System. From a Polar
Rectangular System
Examples. General
System to a Rectangular System.

From One System

Examples

....... .........
CHAPTER
THE

V.

CIRCLE.

Equation of Circle

38.

Generation of Circle.
General Equation of

39.

Polar Equation of Circle

36, 37.

~rv

ou

Circle.

....

Concentric

Circles.

59

Ex

.................
............
Supplemental Chords .............
Tangent. Sub-tangent ............
Normal. Sub-normal .............
General Equations
Tangent and Normal. Examples
Tangent ..............
Length
amples

40.

41.
42.
43.
44.

45, 46.

of

of

Radical Axis.

Radical Centre.

Examples

.....

47.

Condition that a Straight Line touch a Circle.

48.

Equation of Tangent
Chord of Contact
Pole and Polar

49, 50.
51.

^7

............
...............
.....

Conjugate Diameters.

Examples.

CHAPTER

General Examples

VI.

THE PARABOLA.
52, 53.

70

Slope

Generation of Parabola.
tions

Equation of Parabola.

Defini

................
...........
Parabola

54.

Construction of

55.

Latus-Rectum.

Examples

77

CONTENTS.

Vil

PAGES

ARTS.
57-59.

Polar Equation of Parabola


Tangent. Sub-tangent. Construction of Tangent

60, 61.

Normal.

Sub-normal

62.

Tangents

at Extremities of

63.

ici

56.

= +

88
.

89

90

Latus-Rectum

91

92

64.

Examples
y%
Tangent Line makes Equal Angles with Focal Lines and
Axis
Condition that a Straight Line touch the Parabola.

95

65.

66.

67.

68.
69.
70.

71, 72.
73.

c$

Slope

Equation of Tangent
Locus of Intersection of Tangent and Perpendicular from
Focus
Locus of Intersection of Perpendicular Tangents
Chord of Contact
Pole and Polar
Conjugate Diameters
Parameter of any Diameter. Equation of a Diameter

...

at the Extremities of a Chord.


General Examples

Tangents

CHAPTER

95
96
97

97
98
98
100

Examples.
101

VII.

THE ELLIPSE.
74, 75.

76. 77.

Generation of Ellipse. Definition. Equation of Ellipse


Focal Radii

78.

Construction of Ellipse

Latus-Rectum.

Polar Equation of Ellipse

81.

Supplemental Chords
Tangent. Sub-tangent
Tangent and Line through
Centre

85.

86,87.
88.

89.

90.

91.
92.
93.

94.

108
Ill

79.

84.

106
109

80.

82,83.

Eccentricity.

Examples
.

114
115

Point

of

Tangency and

H8

...

Methods of constructing Tangents


Normal. Sub-normal. Examples
Normal bisects Angle between the Focal Radii
Condition that a Straight Line touch the Ellipse.

...

118
119

....

122

Slope

Equation of Tangent
Locus of Intersection of Tangent and Perpendicular
from Focus
Locus of Intersection of Perpendicular Tangents
Equation of Chord of Contact
Pole and Polar
Conjugate Diameters
.

123

124
125
125

126
126

CONTENTS.

via
ARTS.
95, 96.

97.
98.
99.

PAGK8
Equation of a Diameter.
Conjugate Diameter
2
a"

a*

Co-ordinates of Extremities of

129

b*

130

...

Parallelogram on a pair of Conjugate Diameters


Relation between Ordinates of Ellipse and Circles on

Axes
100, 101.

131

133

Construction of Ellipse.
General Examples

Area

CHAPTER

of Ellipse.

Examples.
134

VIII.

THE HYPERBOLA.
102,103.
104, 105.

100, 107.
108.

109.
110.
111.

112,113.
114.

Generation of Hyperbola. Definitions.


Equation of
Hyperbola
Focal Radii
Eccentricity.
Construction of Hyperbola. Latus-Rectum
Relation between Ellipse and Hyperbola

....

116, 117.
118.

119.

120.

121.

122.
123.
124.
125.

126, 127.

128.
129.

130.
131.

144

146

Conjugate Hyperbola. Examples


Polar Equation of Hyperbola

146

Supplemental Chords
Tangent, Sub-tangent
Tangent and Line through Point of Tangency and

150

149

Centre
115.

141

143

Method
Normal.

151

of constructing

Tangents

151

Sub-normal.

Examples
Angle between the Focal Radii
Condition that a Straight Line touch the Hyperbola.
Slope Equation of Tangent
Locus of intersection of Tangent and Perpendicular
through Focus
Locus of intersection of Perpendicular Tangents
Chord of Contact
Pole and Polars
Tangent

bisects

Conjugate Diameters
Conjugate Diameters lie in the same Quadrant
Co-ordinates of
Equation of Conjugate Diameter.
Extremities of Conjugate Diameter
2 =
a12
a2
b2
Parallelogram on a pair of Conjugate Diameters. Ex

...

150

b"

amples
Asymptotes
Asymptotes as Axes.
Vertex

152
154
155
155
155

156
156
156
157
157

158
159
160

Rhombus on

Co-ordinates of
162

CONTENTS.

ARTS.
132.

PAGES

Tangent Line, Asymptotes being Axes. The Point


of Tangency
Intercepts of a Tangent on the Asymptotes
Triangle formed by a Tangent and the Asymptotes
Intercepts of a Chord between Hyperbola and its

.............

133.
134.

135.

....

Examples.

General Examples

CHAPTER

165

165

...

165

Asymptotes.

164

IX.

GENERAL EQUATION OF THE SECOND DEGREE.


136, 137.

The General Equation.

138.

First Transformation.

139.

Second Transformation
a = 0. c -

140, 141.
142.
143.

144.
145.

146.

.......
....
..........

Discussion

Signs of Constants

...........
Summary .............
..............
=
...........
............ .....
General Summary. Examples .........

..."

6-<4ac

62

62

4 ac

>

4 ac

CHAPTER

170
171

172
173
175

175
176
177
173

X.

HIGHER PLANE CURVES.


147.

148.

149.
150.
151.

152.

Definition

EQUATIONS OF THE THIRD DEGREE.


The Semi-cubic Parabola
Duplication of Cube by aid of Parabola
The Cissoid
Duplication of Cube by aid of Cissoid
The Witch

...........
......
............
................

188
190
1 91

193
194

EQUATIONS OF THE FOURTH DEGREE.


153.
154.

155.
156.

.........
.....
The Limacon ............
The Lemniscate ...........
The Conchoid

Trisection of an Angle by aid of Conchoid

196
198

JQQ

201

TRANSCENDENTAL EQUATIONS.
158.

The Curve
The Curve

159.

The, Cycloid

157.

...........
...........
........

of Sines
of

Tangents

203
204
206

CONTENTS.

PAGES

ARTS.

SPIRALS,
160.

Definition

161.

The
The
The
The
The

162.
163.
164.

165.

PART

208

Spiral of

Archimedes

208
210
212
213

Hyperbolic Spiral
Parabolic Spiral
Lituus

214

Examples

Logarithmic Spiral.

SOLID ANALYTIC GEOMETRY.

II.

CHAPTER

I.

CO-ORDINATES.
166.

The Tri-planar System.

167.

Projections

168, 169.

Length

of

Line

joining

170.

Angles. Examples
The Polar System

171.

Relation

217
219

Examples

Two

Points.

Directional

220
222

between Systems.
ordinates.
Examples

CHAPTER

Transformation of Co
223

II.

THE PLANE.
226

172.

Equation of Plane

173.

Normal Equation

174.
175.

Symmetrical Equation of Plane


General Equation of Plane

176.

Traces.

177.

178.

Perpendicular from Point on Plane


Plane through three Points

179.

Any Equation between

227
229

of Plane
.

Intercepts

229
.

three Variables.

230
231

232

Discussion.

233

Examples

CHAPTER

III.

THE STRAIGHT LINE.


180.
181.

Equations of a Straight Line


Symmetrical Equations of a Straight Line

236
237

CONTENTS.

xi

ARTS.

PAGES

182.

To

183.

184.

Planes
Line through One Point
Line through Two Points.

185.

Intersecting Lines

242

Angle between Two Lines


Angle between Line and Plane

243

247

191-193.

Transformation of Co-ordinates
The Cone and its Sections

194, 195.

Definitions.

186, 187.
188.
189, 190.

find

where a given Line pierces the Co-ordinate


238
239

Examples

246

Equation of a Conic.

CHAPTER

239

Examples

250
254

IV.

DISCUSSION OF SUHFACES OF THE SECOND OKDER.


General Equation of the Second Degree involving three
Variables. Transformations and Discussion
.

The Ellipsoid and varieties


The Hyperboloid of One Sheet and varieties
The Hyperboloid of Two Sheets and varieties
The Paraboloid and varieties
Surfaces of Revolution.

Examples

259
262

....
.

265
267

269
273

PLANE ANALYTIC GEOMETRY.

CO-ORDINATES.

PAST

CHAPTElt

I.

THE CARTESIAN OR BILINEAR


SYSTEM.

of objects are determined by


other
some
objects whose positions are
referring
assumed as known. Thus we speak of Boston as situated
Here the ob
west.
in latitude
north, and longitude
is referred are the equator and the
Boston
which
to
jects
meridian passing through Greenwich. Or, we speak of Bos
ton as being so many miles north-east of New York. Here the
1.

THE

relative

them

positions

to

reference are the meridian of longitude through


and New York itself. In the first case it will be
each
observed, Boston is referred to two lines which intersect
other at right angles, and the position of the city is located
when we know its distance and direction from each of these
objects of

New York

lines.

In like manner, if we take any point such as P x (Fig. 1) in


the plane of the paper, its position is fully determined when
we know its distance and direction from each of the two lines
which intersect each other at right angles in
and

This method of locating points is known by the


that plane.
name of THE CARTESIAN, or BILINEAR SYSTEM. The lines of
1

PLANE ANALYTIC GEOMETRY.


X,
Y, are called CO-ORDINATE AXES, and, when
read separately, are distinguished as the X-AXIS and the
Y-AXIS. The point 0, the intersection of the co-ordinate

reference

axes, is called

the ORIGIN OF CO-ORDINATES, or simply the

ORIGIN.

x and y which measure the distance of the


from
the Y-axis and the X-axis respectively, are
point P!

The

lines

FIG.

1.

the distance (x ) from


called the co-ordinates of the point
the Y-axis being called the abscissa of the point, and the dis
f
tance (y ) from the X-axis being called the ordinate of the
point.

Keferring to Fig. 1, we see that there is a point in each


of the four angles formed by the axes which would satisfy
the conditions of being distant x from the Y-axis and distant
2.

y from the X-axis.

This ambiguity vanishes when we com


In the case

bine the idea of direction with these distances.

of places on the earth s surface this difficulty is overcome by


using the terms north, south, east, and west. In analytic geome
are used to serve the same
try the algebraic symbols -f- and

purpose.

All distances measured to the right of the Y-axis

CO-ORDINA TES.
are

called positive

abscissas

measured

those

3
to

the

left,

negative ; all distances measured above the X-axis are called


all distances below, negative.
With this
positive ordinates
the
co-ordinates
of
the
become
understanding,
(V, y )
point Pj.
;

of

2,

(-

of

P (- x f

3,

of

P4

-y

(x

).

3. The four angles which the co-ordinate axes make with


each other are numbered 1, 2, 3, 4. The first angle is above

the X-axis, and to the right of the Y-axis the second angle
above the X-axis, and to the left of the Y-axis the third
;

is

angle is below the X-axis, and to the left of the Y-axis the
fourth angle is below the X-axis and to the right of the
;

Y-axis.

EXAMPLES.
1.

Locate the following points

(2.

1,

(0, 1),

(-

1,

2), (3,

What

(-

4, 0), (0,

(0, 1),

3).

are the lengths of its sides ?

Ans.

Vl3,

The ordinates of two points are each


them situated with reference

the line joining

Ans.

line

0),

Locate the quadrilateral, the co-ordinates of whose ver


(2, 0), (0, 3),

5.

2,

- 4).

tices are,

4.

(-

1),

Locate the triangle, the co-ordinates of whose vertices

are,

3.

- 1), (1, 2), (2, 3), (3,


- 4).
(0, 0), (3, 0), (0,

The commoji
joining them

abscissa of two points


situated ?

is

5, 5,

V 13.

how

is

to the X-axis ?

Parallel, below.

how

is

the

6. In what angles are the abscissas of


points positive ?
In what negative ?

In what angles are the ordinates of points negative ?


7.
In what angles positive ?

PLANE ANALYTIC GEOMETRY.

In what angles do the co-ordinates of points have like


In what angles unlike signs ?
signs ?
8.

9.

The base

X-axis and
the origin

its

of an equilateral triangle coincides with the


is on the Y-axis at the distance 3 below

vertex

required the co-ordinates of its vertices ?


Ans.
(i- V12, 0), (0,
3), (-i V"l2,

ordinate

is

and how

is it

situated ?

Ans.

straight line passing through the


an
ing
angle of 45 with the X-axis.
11.

0).

moves that the ratio of its abscissa to its


=
1, what kind of a path will it describe,
always

If a point so

10.

The extremities

of a line are the points

origin,

and mak

1,

(2, 1), (

2)

construct the line.

the ordinate of a point is


0, on which of the
If the abscissa is
?
co-ordinate axes must it lie ?
12.

If

13.

Construct the points

the line joining them


14.

Show

is

2,

3), (2, 3),

bisected at

that the point (m, n)

and show that

(0, 0).
is

distant

Vm +
2

?i

from

the origin.
15. Find from similar triangles the co-ordinates of the
middle point of the line joining (2, 4), (1, 1).

Ans.

($,$).

THE POLAR SYSTEM.


4. Instead of locating a point in a plane by referring it to
two intersecting lines, we may adopt the second of the two
methods indicated in Art. 1. The point P 1? Fig. 2, is fully

determined when we know its distance


PI
X (= 0) from some given point
tion P!
to
X. If we give all values from
line
values from

to 360

to

(=
in
oo

and direc
some given
to r, and all
r)

easily seen that the position


be located.

0, it is

of every point in a plane may


This method of locating a point

is

called the

POLAR SYSTEM.

CO-ORDINA TES.

The point
is called the POLE
the line
X, the POLAR
Axis, or INITIAL LINE the distance r, the EADIUS VECTOR
;
the angle 0, the DIRECTIONAL or VECTORIAL
;

The
POLAR CO

ANGLE.

distance r and the angle


ORDINATES of a point.

0,

(r, 0),

are called the

In measuring angles in this


system, it is agreed (as in
trigonometry), to give the positive sign (+) to all angles meas5.

FIG.

ured round to the


sign

(-)

to those

2.

left from the


polar axis, and the opposite
measured to the right. The radius vector

considered as positive
(+) when measured from the
pole toward the extremity of the arc (0), and
negative (-)
when measured from the pole
away from the extremity of the
(r)

is

arc

(0).

few examples will make

points clear.
If r
2 inches and

this

method

of locating

45, then (2, 45) locates a point


PI 2 inches from the pole, and on a line
making an ano-le
of -f 45 with the initial line.
If r

=-2

inches and
45, then (- 2, 45) locates a
2 inches from the
pole, and on a line making an
angle of 45 with the initial line also but in this case the
point is on that portion of the
boundary line of the angle
which has been produced backward
the
point

If

=2

inches and 9

= - 45,

through
then (2,

pole.

- 45)

locates a

PLANE ANALYTIC GEOMETRY.

below
inches from the pole, and out on a line lying
point P 4 2
it.
with
45
of
the initial line, and making an angle
45 )
2,
45, then (
2 inches and 6
If r
the
to
pole),
locates a point P 2 directly opposite (with respect
the point

P4

=-

=-

- 45).

(2,

in analytic geometry oi express


6. While the usual method
it
and seconds (,
minutes,
in
is
degrees,
ing an angle
of
terms
in
convenient to express angles
frequently becomes
radius of the
is equal in length to the
arc
whose
the angle
UNIT.
CIRCULAR
is called the
measuring circle. This angle
ot
centre
the
at
that angles
from
,

We

same

know

"),

geometry

each other as the arcs included between


and ff be two central angles, we
hence, if

circle are to

their sides;

have,

arc

_.

&
Let

ff

= unit

angle

~"arc

then arc

=r

(radius

of

measuring

circle).

_
Hence

= 360,

Hence,

circular unit
.

If

arc

-.

r 6

= arc

common
r

360

circular unit.

measure, then arc

=2

TT

= 2irr.

circular unit.

Therefore the equation,

= 2 X circular

360

TT

unit,

...

(1)

between the two units of measure.


expresses the relationship

j
EXAMPLES.
an
What is the value in circular measure of
From (1) Art. 6, we have,

1.

= 30 X 12 = 2 circular unit.
30 = - circular unit.
TT

360
..

angle of 30 ?

CO-ORDINA TES.
2.

What

are the values in circular measure of the


following

angles ?

1, 45, 60,
3.

What
3

90,,

120, 180, 225, 270, 360.

are the values in degrees of the


following angles?

4"

*
4"

4.

What

5.

Locate the following points

is

8"

the unit of circular measure ?

Ans.

(2,

(-

40),

4,

57, 17

45.".

90), (3,

135).,

^3,

(-

1,

180),

- 1, - *),
6.

2,

Locate the triangle whose vertices


are,
/
3
\
\
5
1
-7T \, M, -TT
4
-

7.
The base of an equilateral
triangle (= a) coincides with
the initial line, and one of its vertices is at
the pole
re
quired the polar co-ordinates of the other two vertices.
;

(a>

8.

The polar

co-ordinates

of

a point

are ( 2,
\

three other ways of


locating the

same

point,

~\
4

()).

Give

using polar

co-ordinates.

Ans.

9.

a.,

Construct the line the co-ordinates of whose extremities

PLANE ANALYTIC GEOMETRY.

8
10.

How

being ( 3,

is

-\

the line, the co-ordinates of two of


f 3,

its

points

situated with reference to the initial

Ans.

Parallel.

Find the rectangular co-ordinates of the following points


11-

13

4,

LOCL

CHAPTER

II.

LOCI.

THE Locus OF AN EQUATION

7.

generatrix as

it

moves

is

the

path described by

its

in obedience to the laiv expressed in the

equation.

The EQUATION OF A Locus


the

law subject

to

the algebraic expression of

is

which the generatrix moves in describing that

locus.

If we take any point P 3 equally distant from the X-axis


and the Y-axis, and impose the condition that it shall so move
,

FIG.

that the ratio of

its

ordinate to

3.

its

abscissa shall always be

equal to 1, it will evidently describe the line


algebraic expression of this law is
It

and

is

The

1,

or y

Pi-

= x,

called the Equation of the Locus.


line
is called the Locus of the Equation.

P^

The

Again

PLANE ANALYTIC GEOMETRY.

10

we take the point P 4 equally


make it so move that the ratio of

if

distant from the axes, and


ordinate to its abscissa

its

at any point of its path shall be equal to


1, it will describe
In this case the equation of the locus is
the line P 4 P 2
.

y~

1,

and the
8.
first,

line

P4 P 2

or y

x,

the locus of this equation.

is

be observed in either of the above cases (the


for example), that while the point P 3 moves over the

It will

line P 8 PU its ordinate and abscissa while always equal are


yet in a constant state of change, and pass through all values
For this reason y and x are
from
oo, through 0, to -|- GO.

called the

VARIABLE

or

GENERAL CO-ORDINATES

of the line.

we

consider the point at any particular position in its


x
as
at P, its co-ordinates (
?/) are constant in
path,
of
to
this
the point, and to
and
value,
position
correspond
If

this position alone.

The

variable co-ordinates are represented

by x and

of the moving
y, and the particular co-ordinates
point for any definite position of its path by these letters
with a dash or subscript or by the first letters of the
Thus (x ?/), (x lt yi), (a, ), (2, 2)
alphabet, or by numbers.
correspond to some particular position of the moving point.
;

EXAMPLES.
1.

Express in language the law of which y

algebraic expression.
Ans. That a point shall so

move

= 3x

-f-

is

the

in a plane that its ordinate

shall always be equal to 3 times its abscissa plus 2.


2.

3.

The sum

a quantity a is
point so moves that its ordinate
b
to
its
abscissa
a
; required the
quantity
always equal
of
the
law.
expression
algebraic
b.
Ans. y -\- a
\x
a

of the squares of the ordinate and abscissa of


a 2 what is the
is always constant, and

moving point

equation of

its

path

Ans.

x2

+y

==

a2

LOCI.

11

Give in language the laws of which the following are

4.

the algebraic expressions

^-2,*= -6.

2y--.|.
y

4 z2

z.

2 x 2 -f 3

2
?/

= 6.
if

= 2px.

a^f

5 y2

z2

= 18.
=a6
2

9. As the relationship between a locus and its equation


constitutes the fundamental conception of Analytic Geometry,

important that it should be clearly understood before


We have
entering upon the treatment of the subject proper.
it

is

been accustomed in algebra to treat every equation of the


x as indeterminate. Here we have found that this
form y

equation admits of a geometric interpretation ; i.e., that it repre


sents a straight line passing through the origin of co-ordi
nates and making an angle of 45 with the X-axis. We shall
find, as

we

dental,

which does not involve more than three variable quan

tities, is

proceed, that every equation, algebraic or transcen

susceptible of a geometric interpretation.

We

shall

geometric forms can be expressed alge


braically, and that all the properties of these forms may be
find, conversely, that

deduced from their algebraic equivalents.


Let us now assume the equations of several loci, and let us
locate and discuss the geometric forms which they represent.
10. Locate the geometric figure whose algebraic &wwdent is

We
its

know

= 3x+2.

that the point where this locus cuts the Y-axis has
in the equa
0.
If, therefore, we make x

abscissa x

we

shall find the ordinate of this point.


Making the
substitution we find y
2.
Similarly, the point where the

tion,

locus cuts the X-axis has

for the value of its ordinate.

Mak-

t+T*

PLANE ANALYTIC GEOMETRY.

12

in the equation, we find x


ing y
the axes and marking on them the points

we

will have

two points of the required

successively equal to
1, 2, 3,

1,

2,

- 3,

f.

locus.

Drawing now

Now make

etc.

in the equation, and

find the corresponding values of y.


the result thus
let
us
tabulate
convenience

For

Values of x

Corresponding

2
3

"

"

_2

Values of y
5
8
11

FIG.

"

4.

a line through them we


Locating these points and tracing
have the required locus. This locus appears to be a straight

LOCI.

13

We

line
and it is, as we shall see hereafter.
shall see also
that every equation of the first degree between two variables
The distances Oa and Ob
represents some straight line.

which the

line cuts off on the co-ordinate axes are called


INTERCEPTS. In locating straight lines it is usually sufficient
to determine these distances, as the line drawn
through their
extremities will be the locus of the equation from which their

values were obtained.

EXAMPLES.
1.

Locate the geometric equivalent of

I,-..-l_2*
Solving with respect to y

y=-2x +
The extremities

we

in order to simplify,

have,

2.

of the intercepts are


(0,2),

(1,0).

Locating these points, and drawing a straight line through


them, we have the required locus.
Construct the loci of the following equations

5.

6.

12.

2y
Is

cx

-cl

= 3x.

10.

i-

11.

the point (2, 1)


2x
3 ? Is
y

-y=-3 y

_2x.

on the line whose


(6, 9) ?

NOTE.
If a point is on a line,
the equation of the line.

its

Is (5, 4) ?

equation
Is (0,

co-ordinates

must

is

- 3) ?
satisfy

PLANE ANALYTIC GEOMETRY.

14
13.

Which

of the following points are on the locus of the

2
equation 3 x

(2, 1),

14.

+2y = 6?
2

(V2,

(0,V3),

0),

2x

1,

Construct the lines y

show by similar

11.

-2x = 3y

0), (2,

V3)

line

6.

Construct the polygon, the equations of whose sides are

y
16.

(- V2,

1, 3),

Write six points which are on the


- y

15.

(-

Discuss

= sx

a;,

= 5.

and y

-f-

saj

+ 4,

and

triangles that they are parallel.

and
x*

construct the equation

+ y* =

Solving with respect to

y =

=t

16.

y,

we

Vie - X

have,
2-

The double sign before the radical shows us that for every
we assume for x there will be two values for y, equal

value

and with contrary

This is equivalent to saying that


signs.
for every point the locus has above the X-axis there is a cor
responding point below that axis. Hence the locus is symmet

with respect to the X-axis. Had we solved the equation


with respect to x a similar course of reasoning would have
shown us that the locus is also symmetrical with respect to the
Y-axis. Looking under the radical we see that any value of x
less than 4 (positive or negative) will always give two real
values for y ; that x
0, and that any
J- 4 will give y =
value of x greater than -j- 4 will give imaginary values for y.
Hence the locus does not extend to the right of the Y-axis
x
4.
farther than x
-f- 4, nor to the left farther than
rical

Making

= 0,
=
0,
y

we have y
"

"

=
=

_j_

-j-

4.

LOCI.

15

Drawing the axes and constructing the points,


- 4), (4, 0), ( 4, 0), we have four
(0, 4), (0,
points

of

the locus

i.e.,

B,

A, A!.
Y
B

FIG.

Values of x

5.

Values of y

Corresponding

+ 3.8 and 3.8


+ 3.4 and - 3.4
+ 2.6 and - 2.6

2
3

-1
-2
-3
-4

+ 3.8 and - 3.8


+ 3.4 and 3.4
+ 2.6 and - 2.6

Constructing these points and tracing the curve,

we

find

it

to be a circle.

This

might readily have been inferred from the form

of the equation, for

we know

that the

sum

of the squares

PLANE ANALYTIC GEOMETRY.

16
the

(OC) and ordinate (CP^ of any point Pj


equal to the square of the radius (OPj).
We might, therefore, have constructed the locus by taking
the origin as centre, and describing a circle with 4 as a radius.
NOTE, x
for any assumed value of y, or y
-JL
_j_ 0,
of

abscissa

in the circle

is

any assumed value of x always indicates a tangency. Re


ferring to the figure we see that as x increases the values of
y decrease and become -j- when x = 4. Drawing the line
for

represented by the equation x


to the curve.

We

= 4,

we find that it is tangent


we proceed that any two

shall see also as

coincident values of either variable arising from an assumed


or given value of the other indicates a point of
tangency.
12.

Construct and discuss the equation


9 x2
16 y 2
144.

Solving with respect to

we have

?/,

144

9 x2

16

=
y =

"

Drawing the axes and laying


four points of the locus

Values of x
1

2
3
4
1

3
4

=
=

gives y

i.e.,

B,

off

-j-

these distances,

A,

Corresponding
"

4-4:.

Values of y

4 2.9 and - 2.9

"

+2.6

((

_i_

"

we have

Fig. 6.

+2.9

"

-r-

2.6

"

"

2.9

"

"

"

-J-

Locating these points and tracing the curve through them,


the required locus.
Referring to the value of y we
see from the double sign that the curve is symmetrical with

we have

respect to the X-axis.

The form

of the equation (containing

LOCI.

17

of the variables), shows that the locus


only the second powers
with
Looking
is also
respect to the Y-axis.

symmetrical

FIG.

6.

under the radical we see that any value of x between the


4 will give two real values for y ; and that
4 and
limits
will give imaginary values for
any value beyond these limits
Hence the locus is entirely included between these limits.
y.
with which we shall have more to do hereafter,
This

is

curve,
called the ELLIPSE.

13.

Discuss and construct the equation


2
2/

Solving,

we have

is symmetrical with respect to the


of
the
as
and
equation contains only the first power
X-axis,
As
to the Y-axis.
with
not
it
is
that
respect
symmetrical
x,
will always give real values for y,
every positive value of x
the locus must extend infinitely in the direction of the posi
tive abscissae and as any negative value of x will render y

We

see that the locus

PLANE ANALYTIC GEOMETRY.

18

imaginary, the curve can have no point to the left of the


hence the curve
Making x
0, we find y
-j- 0;
passes through the origin, and is tangent to the Y-axis.

Y-axis.

Making y

= 0,

we

find

hence the curve cuts the

X-axis at the origin.

Values of x

Corresponding

"

2
3
4

From
is

Values of y
2 and

+2.8

+ 3.4
+4

these data

we

-2
- 3.4
-4
-2.8

easily see that the locus of the equation

represented by the figure below.

FK;.

This curve
14.

is

called the

7.

PARABOLA.

Discuss and construct the equation


4 z2
9 ?2
36

Hence

We see from the form of the equation that the locus must
be symmetrical with respect to both axes. Looking under

19

LOCI.

the radical, we see that any value of x numerically less than


3 will render y imaginary. Hence there is no
_j_ 3 or
We see also that
point of the locus within these limits.

any value of x greater than


values for

The

y.

3 or
3 will always give real
locus therefore extends infinitely in the

direction of both the positive


3.
limits x

=i
Making x = 0,

we

find

and negative

-J-

abscissae

from the

hence, the curve

does not cut the Y-axis.

Making y

= 0,

we

find

the X-axis in two points

Value of
4

x.

(3, 0), (

-j-

hence, the

Corresponding.

Values of y

+
+ 2.6
+ 3.4
+ 1.7
+ 2.6

-6

4-3.4

1.7

5
6

-4

FIG.

This curve

is

called the

curve cuts

3, 0).

8.

HYPERBOLA.

and
"

"

"

- 1.7
- 2.6
- 3.4
- 1.7
-2.6

"

- 3.4

PLANE ANALYTIC GEOMETRY.

20
15.

We

have in the preceding examples confined ourselves

RECTANGULAR equations
of equations whose loci were referred to rectangular
Let us now assume the POLAR equation
axes.
to the construction of the loci of

i.e.,

and discuss and construct

Assuming values

for

6 (1

cos 0)

it.

6,

we

find their cosines

convenient table of Natural Cosines.


values,

we

Substituting these

find the corresponding values of

Values of 6

Values of cos
1.

.86

.50

6
6

160

- .50
- .94

180

200
240

1.

.94
.50

270

(1
(1
(1

6 (1

30

90

r.

Values of r

60

120

from some

.86)
.50)
)

+ .50)
+ .94)
6 (1 + 1
6 (1 + .94)
6 (1 + .50)
6 (1 6

(1

6 (1

300

.50

6 (1 -- 50)

330

.86

6 (1

.86)

=
= .84
= 3.
= 6.
= 9.
= 11.64
= 12.
= 11.64
= 9.
= 6.
= 3.
= .84

as the
the initial line OX, and assume any point
a
series of lines, making the
draw
this
point
Through
pole.
assumed angles with the line OX, and lay off on them
the corresponding values of r.
Through these points, tra
the
have
we
required locus.
cing a smooth curve,

Draw

LOCI.

FIG.

This curve, from


16. Discuss

and

its

9.

heart-like shape,

is

called the CARDIOID.

construct the transcendental


equation

y
NOTE.

21

= log x.

transcendental .equation

is one whose decree


power of analysis to express.
Passing to equivalent numbers we have
= x, when 2 is
the base of the system of
logarithms selected.
As the base of a system of
logarithms can never be nega
tive, we see from the equation that no
negative value of x can
Hence the locus has none of its
satisfy it.
to

ti-anscends the

&

the left
points
the other hand, as
every positive value of
x will give real values for
y, we see that the curve extends
the direction of the
infinitely
of the Y-axis.

If

y = 0,

On

positive abscissae

then

PLANE ANALYTIC GEOMETRY.

22
If

0,

then
2*

The

.-.

= log

.-.

co.

at a unit
locus, therefore, cuts the X-axis

distance on

the positive side, and continually approaches the Y-axis with


out ever meeting it. It is further evident that whatever be
the base of the system of logarithms, these conditions must
x.
hold true for all loci whose equations are of the form a?

Values of x
1

Corresponding

Values of y

"

"

"

"

.5

"

.25

traced through them will


Locating these points, the curve

be the required locus.

FIG.

10.

This curve is called the LOGARITHMIC Curve,


being taken from its equation.

its

name

LOCI.

23

The preceding examples explain the method


employed

17.

in constructing the locus of


any equation. While it is true
that this method is at best
be made
approximate, yet it

may

sufficiently accurate for all practical purposes by assuming


for one of the variables values which differ from each other

by very small quantities. It frequently happens (as in the


case of the circle) that we
may employ other methods which
are entirely accurate.
In the discussion of an equation the first
step, usually,
it with
respect to one of the variables which enter

18.
is

to solve

into

it.

The question

of which variable to select

is

immate

rial in
principle,

yet considerations of simplicity and conven


ience render it often times of
The sole
great importance.
difficulty, in the discussion of almost all the higher forms of
If this difficulty can
equations, consists in resolving them.
be overcome, there will be no trouble in
tracing the locus and
discussing it.
If, as frequently happens, no trouble arises in
the solution of the
equation with respect to one of the vari

then that one should be selected as the


dependent
variable, and its value found in terms of the other.
If it is
equally convenient to solve the equation with
to either
ables,

respect

of the variables

selected

which enter into

it,

whose value on inspection

then that one should be


will afford the

discussion.

EXAMPLES.
Construct the loci of the
following equations
1.

2.

+ l-0.

2jr-4
y

-.,
2

3.

2y

4.

4aj 2

10.

= 10.

+ 5s
-9 =-36.
2

5.

fi+4

6.

**+y-

7.

8.

35

?/

= ^ cos
= logy.
2

simpler

PLANE ANALYTIC GEOMETRY.

24

Construct the loci of the following


9
10.
11.

si.y^O.
x + 2 ax + a = 0.
x - a* = 0.
2

12

yi_9 = 0.

13.

7/

2 xy

+ x - 0.
2

14.

x*-x-6 = 0.

15.

or -j-

6 ==

+4x - 5

0.

16.

17.

x*-7x + l2 = 0.

18.

x2

0.

+ 7 z + 10 =

Factor the first member: equate each factor


NOTE.
and construct separately.

0.

to 0,

THE STRAIGHT LINE.

CHAPTER

25

III.

THE STRAIGHT

LINE.

19. To find the equation of a straight line, given the angle


which the line makes with the X-axis, and its intercept on the
Y-axis.

FIG.

11.

Let C S be the line whose equation we wish to determine.


Let SAX
and OB
b.
Take any point P on the line

=
PM

to OY and BN
to OX.
Then (OM, MP) = (x, y) are the co-ordinates of P.
From the figure PM = PN + OB = BNtan PEN +

and draw

||

||

BN = OM = x,
..

and tan PBN = tan SAX = tan


Substituting and letting tan a = s, we have,
(I)

b,

but

PLANE ANALYTIC GEOMETRY.

26

Since equation (1) is true for any point of the line SC, it
true for every point of that line hence it is the equation of
the line. Equation (1) is called the SLOPE EQUATION OF THE
is

STRAIGHT LINE

COROLLARY

If

= tan
b =
in

sx

1.

(1),

called the slope.

is

we

have,

(2)

a line which passes through the

for the slope equation of


origin.

COR.

If s

2.

y
which

is,

as

it

we have

in (1),

=b

ought to

be, the

equation of a line parallel to

the X-axis.

COR.

3.

If

oo,

then

90, and the

line

becomes

parallel to the Y-axis.

Let the student show by an independent process that the


equation of the line will be of the form x = a.

SCHOLIUM. We have represented by the angle which the


makes with the X-axis. As this angle may be either
acute or obtuse, s, its tangent, may be either positive or nega

line

tive.

The

line

may

also cut- the Y-axis either above or below

the origin; hence, b, its Y-intercept,


From these considerations
negative.

=-

sx

-{-

it

appears that

+b

represents a line crossing the


== sx

be either positive or

may

first

angle

represents a line crossing the second angle

y =

sx

represents a line crossing the third angle

= sx

represents a line crossing the fourth angle.

THE STRAIGHT

LINE.

27

EXAMPLES.

The equation of a
and intercepts.

1.

slope

line

Solving with respect to

Comparing with

we

y,

22
1

x H

(1)

Art.

Making y

Y-intercept.

is

2y

+x=3

required

its

have,

3
.

19,

we

and

= 3 = X-intercept.

find s

in

the

equation,

we have

3.
Construct the line 2 y -\- x
The points in which the line cuts the axes are

2.

O,

?V

and

(3,

3).

-*/

Laying these points

off

on the axes, and tracing a straight

through them, we have the required

line

thus

~F-+fLay

off

OB =

draw

BN OX and make it = 2, also


NP OY and make it = + 1.
||

||

The

line

through

P and B

the required locus.


T^I^T

-*

For

locus.

solving the equation with respect to

tan

BAX.
.-.tan

BAX

is

y,

we

Or otherwise
have,

PLANE ANALYTIC GEOMETRY.

28
3.

Construct the line 2 y

we

Solving with respect to y,

have,

Lay
||

to

also

3.

off

OX
NP

||

BO = 5 = |

and make it = 2
to OY and make

straight line

BK

Draw

draw

it

1.

P and B

through

will be the required locus.

= - = tan PBN = tan

For

BAX =

Hence, in general,

s.

to the left of

BN

is

laid off to the right or

according as the coefficient of

is fiositive

or negative.

Give the slope and intercepts of each of the following


and construct

lines

4.

Ans.
5.

&

= - 2, a = - -

1,

^^

x-

6.

* ==

- 12,
8.

~
3

+2x = 1 -

y.

9.

a and 6 in the answers above denote the X-intercept


NOTE.
and the Y-intercept, respectively.

THE STRAIGHT
What

14.

29

angle does each of the following lines cross ?


10.

= 3x +

11.

= - x + 2.

12.

1.

=2

a;

1.

\<^

Construct the figure the equation of whose sides are

2y
15.

LINE.

+ s-l = 0, 3^=2x + 2, y = - x -

I.

Construct the quadriiftfc&rai the equations of whose sides

are

= 3,

l,

= 2,

= 0.

20. To find the equation of a straight line in terms of

its

intercepts.

FIG.

Let S C be the

line.

Then OB

OA =

12.

= b = Y-intercept,
= X-intercept.

The slope equation of


Art. 19. equation (1),
y

a line

sor

and

we have determined

/>.

to be

PLANE ANALYTIC GEOMETRY

30

From

the right angled triangle

tan

GAB =

tan

AOB, we

BAX =

have,

= OB
OA

.-.-.--*.
a

Substituting in the slope equation,

This

is

called the

we

have,

SYMMETRICAL EQUATION

of the straight

line.

COR.

1.

+ and b +, then we have,


-= 1, for a line crossing the

If a

ni

rvt

\-

first

j-

and

If a

+, then

---f-^ = lisa line crossing the


a

If

and

-f-

and

--

=1

is

a line crossing the third angle.

then

----- *-

second angle.

then

If

angle.

=1

is

a line crossing the fourth angle.

EXAMPLES.
1.

Construct

NOTE.
Y-axis.

1.

Lay off 3 units on the X-axis and


Join their extremities by a straight

2 units on the
line.

Across which angles do the following lines pass

THE STRAIGHT

LINE.

31

Give the intercepts of each, and construct.

+ |=1.

2-

/v,

Q
o

7/
if

4.

i*^
*

^
^

57^ =

"Q

r>

-i
-

^^

AXV-~)C7^-

z--<r^-

K"

Write the slope equations


construct

-f~V=l.

the

of

following lines, and

6.

r-IS-7.
3

s.

y
2

T ?.6 = _
+

i.

y-ix-2.
O
.

-J

O
J

5
10.

Write y

+ 6 in a symmetrical
."

7-

form.

1
1.

/-t

A
O*^"

/>

Given the following equations of straight


their slope and symmetrical forms

lines, to write

11.

+ 3z-7 = z + 2.
^^ = ~ 3

2y

23

13.

a?

12.

14

2?
x

43

_2x-l

PLANE ANALYTIC GEOMETRY.

32

To find the equation of a straight line in terms of the


perpendicular to it from the origin and the directional cosines
21.

of the perpendicular.

The Directional cosines of a line are the cosines


it makes with the co-ordinate axes.

NOTE.

the angles which

FIG

Let OS
Let

be-

the

line.

AOP =
triangles AOP and BOP, we
OA = -OP- OB = OP

OP = p, BOP

From

the

13.

==

7,

have

COS 7

COS

that

is,

7.

cos w

cos 7

Substituting these values in the symmetrical equation,


Art. 20. (1),

+ y-b =

x cos

which

is

1,

we

have, after reducing,

+ y cos 7 = p

the required equation.

(1)

of

THE STRAIGHT
Since

90

This form

and

COR.

and the
COR.

NORMAL EQUATION

(1)

of the straight line.

becomes parallel to the Y-axis.

line

= 90,

If

2.

= 0, then
x =p

If

1.

more frequently met with than that given in

is

called the

is

= sin hence
+ y sin = p
(2)

cos y

cos

cc

33

LINE.

then

y*=p
and the

If x

22.

becomes parallel to the X-axis.

line

+ y sin = p be the equation of a given line,


+ y sin a = p -- d is the equation of a parallel

cos

then x cos

For the perpendiculars p and p j- d coincide in direction


line.
hence the lines
since they have the same directional cosines
to which they are perpendicular are parallel.
;

COR.

Since

1.

p
it

is

evident that d

therefore, (x

the origin

is

-J-

-p =
is

the distance between the lines.

_j_

If,

be a point on the line whose distance from


d,

we have

+ y sin = p
=
x cos
d
y

x cos u
.-.

"

-\-

-J-

d.

sin

(1)

a point (x
y ) from the line
found
-\- y
by transposing the constant
term to the first member, and substituting for x and y the co
Let us. for example, find the
ordinates x y of the point.
distance of the point ( y~3, 9) from the line x cos 30
y sin

Hence the distance


sin a

x cos a

=p

of

is

30

5.

From

(1)

= V3 cos 30

-f 9 sin 30

PLANE ANALYTIC GEOMETRY.

34

From
.*.

Fig.

sin= / =

-2

_L

t=

<2

|-

^b

+6

Vtt 2

the expression for the distance of the point (x

line

J/

Hence
is

13 we have cos

whose equation

is

of the form -

+b=

?/)

from a

1.

Let the student show that the expression for d becomes

VA + B
2

when the equation

of the line

See Art. 24, Equation

given in

is

general form.

its

(1).

EXAMPLES.
1.

line

The perpendicular Kit


makes an arfgle

= 5 and

equation of the

fall

from the origin on a straight


with X-axis required the

of 30

line.

V3

Ans.

sc

+ =
//

2. The perpendicular from the origin on a straight


makes an angle of 45 with the X-axis and its length =

required the equation of the

line.

Ans.
3

What

is

the distance of the point

(2.

Ans.

Find the distance of the point from the


following cases

x -\-y

4) from the

From

(2,

5.

From

(3, 0)

to

6.

From

(0, 1)

to 2

7.

From

(a, c) to

5) to

1.

?/

=
- x = 2.
1.

SJT

= 2.
line

line in each of the

|
|

line

V2

i.

4.

10.

b.

^^

THE STRAIGHT LINE.

35

23. To find the equation of a straight line referred to


oblique axes, given the angle between the axes, the angle which
the line makes with the X-axis and its Y-intercept.

NOTE.

Oblique axes are those which intersect at oblique

angles.

M
N>

FIG. H.
%

it

Let CS be the line whose equation we wish to determine,


being any line in the plane YOX.
Let YOX = ft SAX =
OB = b.
Take any point P on the line and draw
PM to OY and ON to SC
,

From

NP = OB = b.

==
,

the figure

From

||

||

PM = y, OM = a, NOX

then,

triangle

= MN + NP = MN + b

ONM, we

MN

sin

OM

sin

(1)

have,

NOM
MNO
sin

sin

Substituting the value of


(1),

we

drawn from

.M"N"

this equation in

have,

Sin

=
sin

(/?

"

x
)

+b

...

(2)

PLANE ANALYTIC GEOMETRY.

36

This equation expresses the relationship between the co


Bnt as the point
ordinates of at least one point on the line.
selected was any point, the above relation holds good for
every point, and is, therefore, the algebraic expression of the
law which governed the motion of the moving point in de
It is therefore the equation of the line.

scribing the line.


COR. 1. If b

=
?/

0,

then
sin

sin (p

(3)

a)

the general equation of a line referred to oblique axes


passing through the origin.
is

COR.

2.

If b

=
y

and

=o

then

0,

(4)

the equation of the X-axis.

COR.

3.

If

ft

and

/3
.

But tan
This
19

is

<t

=s

5.

= tan n x
y = sx

if

-J-

-\-

the axes are

made

rectangular,

ft.

ft-

See Art.

the slope equation heretofore deduced.

(1).

Cor.

then

(5)

the equation of the Y-axis.


COR. 4. If (3 = 90
i.e.,

then

If

/3

= 90 and = 0, then
y = sx. See Art. 19, Cor.
ft

1.

EXAMPLES.
makes an
two units

Find the equation of the straight line which


1.
angle of 30 with the X-axis and cuts the Y-axis
distant from the origin, the axes

making an angle

with each other.


Ans.
2.

had been assumed rectangular


what would have been the equation ?

If the axes

ple above,

Ans.

of

=x

in the

-JL-

60

-f-

2.

exam

+ 2.

THE STRAIGHT LINE.

37

The

co-ordinate axes are inclined to each other at an


of
angle
30, and a line passing through the origin is inclined
to the X-axis at an angle of 120,
required the equation of the
3.

line.

Ans.

24.
is

y
y

=
f\

,/q
vo

Every equation of the first degree between two variables

the equation of a straight line.

Every equation of the first degree between two variables


can be placed under the form

Ax
in

y,

+ By + C

which A, B, and C may be either


Suppose A, B, and C are not zero.

we

and whose slope

from which
If

(1)

finite or zero.

Solving with respect to

with

(2)

(1) Art.

the equation of a straight line


-

have,

Comparing equation
is

A = 0,

it

then

was derived
?/

is

.19,

we

see that

whose Y-intercept

hence

(1),

the

it

equation

the equation of a straight

line.

the equation of a line parallel to the X-axis.


If

= 0,

then x

the equation of a line


parallel to the Y-axis.
If

= 0,

then y

=-

A
B

X)

the equation of a line


passing through the origin.
Hence, for all values of A, B, C equation (1) is the equa
tion of a straight line.
^

PLANE ANALYTIC GEOMETRY.

38
25.

To find the equation of a straight

line

passing through

a given point.

Let (V, y

be the given point.


is to be straight,

its

Since the line

y = sx + b

which

in

and

...

equation must be

(1)

be determined.

b are to

of a line expresses the relationship which


of every point on it; hence
co-ordinates
the
exists between
when
the co-ordinates of any
satisfied
be
must
its

Now, the equation

equation
for the general co-ordinates x and
point on it are substituted
the
We
equation of condition.
have, therefore,
y.

=sx

But a straight

...

(2)

cannot in general be made to pass


cut off a given distance (b) on

line

through a given point (x /),


the Y-axis, and make a given angle (tan. = s) with the X-axis.
We must therefore eliminate one of these requirements. By
,

subtracting (2) from

which

is

COR.

(1),

we have,

=s

3)

the required equation.

1.

If

0,

then

= SX

(4)

on the
the equation of a line passing through a point
Y-axis.
COR. 2. If y
0, then

is

y
is

=s

(x

(5)

the X-axis.
the equation of a line passing through a point on

COR.

3.

If

=
y

and y

= 0,

then

= sx

the equation (heretofore


through the origin.
is

determined), of a line passing

THE STRAIGHT

LINE.

39

EXAMPLES.
1.

Write the equation of several lines which


pass through

the point
2.

is the equation of the line which


passes through
and makes an angle whose tangent is 2 with the

2),

(1,

(2, 3).

What

X-axis ?

Ans. y

3.

straight line passes through (


with the X-axis.
What

angle of 45

1,
is its

3),

2 x

4.

and makes an

equation ?
Ans. y

=x

2.

Required the equations of the two lines which contain


and 60, respectively
(a, b), and make angles of 30

4.

the point

with X-axis.
Ans.

y
26.

-b = ^.(x-a).

(x",

Since the line

which

and

we

line passing

through

be the given points.

straight its equation

= sx + b

b are to

Since the line


y"),

y")

is

(x",

V3

To find the equation of a straight

two given points.


Let (x r y ),

in

-b = ?L=JL

must be

(1)

be determined.

required to pass through the points (x


have the equations of condition.

y
y"

is

=
=

8X
#t"

+
+

y ),

(2)

...

(3)

As a straight line cannot, in general, be made to fulfil more


than two conditions, we must eliminate two of the four con
ditions expressed in the three
equations above.
Subtracting (2) from (1), and then (3) from (2), we have,
y
y

- y = s (x - x
- = s (x y"

)
x")

PLANE ANALYTIC GEOMETRY.

40

member by member, we

Dividing these,

tf-y"

Hence

-x"

^^

(x

(4)

OC

3C,

is

have,

~ xx

the required equation.


r
then
COR. 1. If y

=
-y =
y
y",

which

is,

COR.

tf>

it

2.

is

or y

should be, the equation of a line


then
If x

as

to the X-axis.

||

x",

which

>

0,

or x

the equation of a line

||

to the Y-axis.

EXAMPLES.
2, 8)
required both
Given the two points (
1, 6). (
line
the
passing through
the slope and symmetrical equation of
1.

them.
Ans.

vertices of a triangle are


required the equations of its sides.
2.

The

=(-

+ 4,

2 x

(-

2, 1),

|
3,

+|=

-4)

1.

(2, 0)

= 5 X + 11
oy = 8
-\4:X
(y + x = 2.
(

Ans.

Write the equations of the lines passing through the points


3.

Ans.
4.

?/

(1,4), (0,0)

Ans.
5.

6.

(-2,3),(-3,-l)

(0,2),(3,-1)
Ans.

= 4x-f-ll.
7.

7 y

2.

+ 2 x = 24.

(-3,0)

Ans. y
8.

+ =

(2,0),

= 0.

(-l,-3), (-2,4)
Ans.

(-2,4)

Ans.

= 4 x.
x

(5,2),

+ 7 x + 10 = 0.

THE STRAIGHT LINE.


27.

To find the length of a

line joining

FIG.

Let (x

i/),

(x",

y"}

41

two given points.

15.

be the co-ordinates of the given points

L = FT" = required length.


Draw
and P A to OY, and PC to OX.
We see from the figure that L is the hypothenuse of a

P,

P".

P"B

||

||

right

angled triangle whose sides are

PC
P"C

Hence,
PT"

= AB = OB - OA = - x
- EC = if - y
=
x"

P"B

=L=

1.

If

- xY

fa"

x
and y
the origin, and we have
Con.

2
V*"

for the distance of a point

and

= 0,

2
y"

(y

r::~y lY

the point

...

from the

(l)

coincides with

(2)

origin.

EXAMPLES.
Given the points (2, 0), (
2, 3)
required the distance
between them also the equation of the line passing through
them.
Ans. L = 5, 47/-f3;r =6.
1.

PLANE ANALYTIC GEOMETRY.

42
2.

what

The

vertices of a triangle are


are the lengths of its sides ?

(2, 1)

1,

2)

Give the distances between the following points


0)

(2, 3), (1,

(4,

5), (6,

(-

(0, 2),

Ans.
11.

What

is

Ans. Area
28.

To

(-

2,-l),

9.

(a,

ft),

10.

(-

2, 3),

(0, 1)

(2,

0)

d)

(c,

V5.

(-

a, 6).

2.

the expression for the area of a triangle whose


(V, y

),

(a",

= \ \x
./iTicZ

8.

0)_

0)

(0, 0), (2,

vertices are

0)

V20.
1,

Ans.
6.

(-

3,

-_!)

^ws.
5.

3,

VlO.

Ans.
4.

7.

2)

V8, VlOJ V2fr

Ans.

3.

(y"

")

(*

y")>

",

^Ae intersection

?
")

- /) +

f
*"

(y"

xf

"

(y

y")]-

of two lines given by their

equations.

Let

y
y

= sx
=s +6

-f- b,

and

a:

be the equations of the given lines.


Since each of these equations is satisfied for the co-ordinates
of every point on the locus it represents, they must -ttt-tke
samer-tim* be satisfied for the co-ordinates of their point of
Hence, for the
intersection, as this point is common to both.
So
the equations are simultaneous.
co-ordinates of this

treating them,

we

pointfind
b

s b

sb

for the co-ordinates of the required point.

EXAMPLES.
1.

Find the intersection of y

= 2x + 1

and 2 y
Ans.

= x - 4.
(-2, -3).

THE STRAIGHT LINE.


2.

The equations

43

of the sides of a triangle are

required the co-ordinates of

its vertices.

(MH-i,
\o
oj

Ans.

Write the equation of the

=
]

which shall pass through


and 3 y
x
8=0,
and make an angle with the X-axis whose tangent is 4.
3.

the intersection of 2 y

3x

-f-

line

-f-

Ans.
4.

What
the

x -f

2,

= 3 x -f 2,
Ans.

5.

x2

The equation
2

-f-

?/

= 4 x -|- 10.

are the equations of the diagonals of the quadri


equations of whose sides are y
x-\-l =Q,

lateral

10

is

+2x+2 =

and y

23 y

;c

+ 2 = 0, 3y

A*

and

a;

6.

whose equation is
required the length of the chord.

of a chord of the circle

=x

-f-

Ans.
29. If

30 x

+ By -f C =
+By+C =

(1)
.

(2)

be the equations of two straight lines, then

Ax -f By + C

+K

(A a;

+ B y +C =
)

...

(3)

(K being any constant quantity) is the equation of a straight


line which passes through the intersection of the lines
repre
sented by (1) and (2).
It is the equation of a straight line
because

it

variables.

an equation of the first degree between two


See Art. 24.
It is also the equation of a straight

is

which passes through the intersection of (1) and (2).


it is obviously satisfied for the values of x and
y which
simultaneously satisfy (1) and (2).
Let us apply this principle to find the equation of the line
which contains the point (2, 3) and which passes through the
intersection of y = 2 x -\- 1 and 2 y + a; = 2.
line

since

PLANE ANALYTIC GEOMETRY.

44

From

(3)

we have y

2a

+ K(2y +

sc

2)=0

for the equation of a line which passes through the intersec


But by hypotheses the point (2, 3) is
tion of the given lines.

on

this line

hence 3

4-1 + K
K

Substituting this value for

we

(6

+ 2-2) =0

have,

or,

1 ==

Let the student verify this result


for the required equation.
by finding the intersection of the two lines and then finding
the equation of the line passing through the two points.
30.

To find the angle between

given by their

lines

tivo

equations.
M,

Yrf

FIG.

Let

y
y

16.

= sx +- and
= sx+ V
/>.

be the equations of
s

SC and MN,

= tan

and

respectively

tan

then

THE STRAIGHT LINE.


From

45

the figures

=
.

(f

(f

-\-

a.

From trigonometry,
tan

.-.

cp

=
tan

= tan -

1 -f ss

COR.

1.

If s

=
<p

tan a

(2).

then

= tan -1

= 0.

.. qp

the lines are parallel.

COR.

2.

If 1

+ =
ss

(f

/.

= tan

substituting;

Or,

.*,

cp

then

0,

= tan -

oo

.-.
g>

= 90

the lines are perpendicular.

These results may be obtained geometrically.

SCHOL.

If the lines are parallel, then, Fig. 16,

If they are perpendicular

= 90
.-.

= s = tan

tan a

.-.

+
(90

+ ss = 1 tan
1-1 = 0.
-{-

-f

tan

= -

cot

tan a

1 4- tan

tan

EXAMPLES.
1.

What

and 2y

is

the angle formed by the lines y

+ 2cc + l=0?

= 90.

PLANE ANALYTIC GEOMETRY.

46

2. Required the angle formed by the


8 = 0.
6x
and 2 y

3.

(2,

Ans.

3x
y
Perpendicular to 2 ?/

(b)

= 0.

= 0.

3x

5=0.
2 y

= 8,

(5)

3y

-j-

2x

= 1.

Given the equations of the sides of a triangle


a -f- 2 and y =
3 required.
l,
y = 2#

+ y=

(a)

q>

Required the equation of the line which passes through


1) and is

Ans. (a) 3 x
4.

y -\-ox

(a) Parallel to 2
(b)

lines

The angles of the triangle.


The equations of the perpendiculars from

vertices to

sides.
(c)
5.

The lengths

What

relation exists

y
y
y
y
6.

What

of the perpendiculars.

between the following

lines

= SX +
= SX 3.
= SX -\- 6.
= SX + m.
b.

relation exists between the following:

y
y

= sx -[= sx

b.

-j- c.

Find the co-ordinates of the point in which a perpen


1 = 0.
2 a;
dicular through (
2, 3) intersects y
7.

,.
8.

Find the length of the perpendicular

origin on the line

2y-\-x

(,r

let fall

from the

= - V80.

= 4.

Ans.

be the equations of three straight lines, and /,


m, and n be three constants which render the equation

B ty-j-C"

THE STRAIGHT
I

+ By + C) + m

(Ax

C")

LINE.

47

+ Wy -f

By
n (Af x
CO
(Afx
an identity, then the three lines meet in a point.

Find the equation of the bisector of the angle between


Ax + By + C = and Afx -f B y -f C = 0.
Ax + By
= __ (A x +j^y + CQ

10.

the two lines

+_C
"

VA + B
2

VA + B

GENERAL EXAMPLES.

1.

straight line makes an angle of 45 with the X-axis


off a distance
2 on the Y-axis what is its equation

and cuts

when

the axes are inclined to each other at an angle of 75 ?


Ans. y
-Y/2 x -j- 2.

2.

Prove
3

y
3.

a;

-f-

that

If

the

lines

=x
(1,

-{- 1,

3 intersect in the point

(x

and

tremities of a line,

ordinates of

its

(x",

y"}

show that

=2x+2

and

0).

are the co-ordinates of the ex


I

t~,

S-

1_2.

a rf the co
j

middle point.

of the sides of a triangle are y = x -f- 1,


x
1 required the equations of the sides of
the triangle formed by joining the middle points of the sides
4.

a;

The equations

= 4,

of the given triangle.

(y
Ans.

lyo
^

5.

=x
,

JL

+4
4

q
O.

Prove that the perpendiculars erected at the middle

points of the sides of a triangle meet in a

NOTE.

Take the origin

common

at one of the vertices

point.

and make

the X-axis coincide with one of the sides. Find the equations
of the sides and then find the equations of the
perpendiculars
;

at the

middle points of the

sides.
The point of intersection
of any two of these perpendiculars ought to
satisfy the equa
tion of the third.

PLANE ANALYTIC GEOMETRY.

48
6.

Prove that the perpendiculars from the vertices of a


meet in a point.

triangle to the sides opposite


7.

Prove that the line joining the middle points of two of


is parallel to the third side and is equal

the sides of a triangle


to one-half of it.

8. The co-ordinates of two of the opposite vertices of a


square are (2, 1) and (4, 3) required the co-ordinates of the
other two vertices and the equations of the sides.
Ans. (4, 1), (2, 3) y
4.
2, x
1, y
3, x
;

9.

Prove that the diagonals of a parallelogram bisect each

other.

Prove that the diagonals of a rhombus bisect each other

10.

at right angles.
11.

Prove that the diagonals of a rectangle are equal.

12.

Prove that the diagonals of a square are equal and

bi

sect each other at right angles.

The distance between the points (x, y) and


the
algebraic expression of the fact.
give
13.

Ans. (x

The points

14.

1)2

15.

3) are equi-distant

points

(x",

at an angle

2), (

What

16.

2)

= (x -

2)

(y

circle circumscribes the triangle

(3, 4), (1,

Ans.

I)

=4

- 2) = 4
2

(y

2
.

from the point

Express the fact algebraically.

(x, y).

(x

(1, 2), (2,

(1, 2) is

is

y"),

(3

1,

2)

3)

or,

whose

+ y = 4.

vertices are

required the co-ordinates of its centre.


Ans. (2, 1).

the expression for the distance between the


inclined
(x ?/ ), the co-ordinate axes being
,

- xj +
2

V<V

(y"

- yj + 2

-x
(x"

(y"

-y

cos

THE STRAIGHT LINE.

49

17. Given the perpendicular distance (p) of a straight line


from the origin and the angle () which the perpendicular
makes with the X-axis required the polar equation of the line.
;

Ans.
cos (0
18.

origin on the line


19.

Required the length of the perpendicular from the

What

the point

is

+|=

2 4

AnSt

the equation of the line which passes through


and makes an angle of 45 with the line

2),

(1,

1.

whose equation

is

-\-

2x

1 ?

Ans.

- x

+_

of two lines passes through the points (1, 2),


the other passes through the point (1,
3), and
makes an angle of 45 with the first line required the
20.

One

4,

3),

equations of the lines.


Ans.

= 1.
x + 1, and y =
3, or x
y
in the normal equation of a line, through
21. If p =
what point does the line pass, and what does its equation
Ans. (0, 0) y = s x.
become ?
;

22.

r sin

Required the perpendicular distance of the point (r cos $,


Ans. r
from the line x cos
y sin = p.
p.

0),

Given the base of a triangle


the squares of its sides == 4 c 2
the vertex is a straight line.
23.

of

= 2 a,
Show

and the difference


that the locus of

24. What are the equations of the lines which pass through
the origin, and divide the line joining the points (0, 1), (1, 0),
into three equal parts.
2 y
x.
Ans. 2 x
?/,

25.

If (x

y ) and

show that the point

(x",

y")

be the co-ordinates of two points,

+ nx m + "/^divides the
m+n
m n

mx

"

y"

ii

ne

-\-

joining

n.

them

into

two parts which bear to each the

ratio

PLANE ANALYTIC GEOMETRY.

50

CHAPTER

IV.

TRANSFORMATION OF CO-ORDINATES.
IT frequently happens that the discussion of an
equa
and the deduction of the properties of the locus it
represents are greatly simplified by changing the position of
the axes to which the locus is
the
referred, thus
31.

tion,

simplifying

equation, or reducing

some desired form. The operation


accomplished is termed the TRANSFORMATION
it

to

by which this is
OF CO-ORDINATES.

FIG.

The equation

of the line PC, Fig. 17,

when
axes

= sx

and

its

is

-f b

referred to the axes

17.

and X.

If

we

refer

equation takes the simpler form

= sx

it

to the

TRANSFORMATION OF CO-ORDINATES.
we

If

refer

it

to

Y"

and

X",

51

the equation assumes the yet

simpler form
y"

appears that the position of the axes materially

it

Hence,

affects the

- 0.

form of the equation of a locus referred to them.

The equation

NOTE.

of a locus

which

referred to rec

is

of
tangular co-ordinates is called the KECTANGULAR EQUATION
the locus when referred to polar co-ordinates, the equation is
called the POLAR EQUATION of the locus.
;

To find the equation of transformation from one system

32.

of co-ordinates to a parallel system, the origin being changed.

FIG.

18.

and Y as axes,
be any plane locus referred to
be any point on that locus. Draw PB to OY
then from the figure, we have,
of P when referred
(OB, BP)
(x, y) for the co-ordinates

CM

Let

and

let

||

to

and

= (# ,/) for the

(O A, AP)
to

Y
DO

and

(OD,
origin.

co-ordinates of

P when

referred

(a,

b)

for the

co-ordinates

of

the

new

PLANE ANALYTIC GEOMETRY.

52

From

OB = OD

the figure

hence

-f

DB

x and y

=b

BP = BA

and

-\-

+ AP

are the desired equations.


As these equations express the relations between x, a, x y,
on the locus they express the relations
b, and y for any point
f

Hence, since the

between the quantities for every point.

CM

Q
expresses the relationship between
equation of th locus
the co-ordinates of every point on it if we substitute for

x and y in that equation their values in terms of x and y


the resulting or transformed equation will express the rela
tionship between the x and y co-ordinates for every point
on it.

EXAMPLES.
1.

What

origin

is

does the equation y

removed

+1

3x

become when the

to (2, 3) ?

Ans. y

Construct the locus of the equation 2 y


fer the origin to (1, 2) and re-construct.
2.

3.

The equation

of a curve

is

2
?/

2
-f x

(2,

origin

2) ?

What

does the equation

become when the origin

is

2
?/

- 2 x* -

removed

to
(

The equation

of a circle

is

x*

2
?/

4.

taken at

+ y = 16.
+ 6x - 3 =

2y
1

?
)

Ans. 2
5.

is

-\-

Trans

x2

Ans.
4.

= 2.
4 x

4 y

-\-

what does the equation become when the

= 3x

2
?/

=a

4 x2

when

1.

referred

does this
rectangular axes through the centre. What
left-hand
the
at
equation become when the origin is taken

to

extremity of the horizontal diameter

TRANSFORMATION OF CO-ORDINATES.
33.

53

To find the equations of transformation from a rectangu-

lar system to

an

oblique system, the origin being changed.

D
FJG.

19.

Let P be any point on the locus CM.


Let O Y O X be the new axes, making the angles y and
with the X-axis. Draw PA to the Y -axis also the lines
,

||

O D, AL, PB
AF, O K to
||

From

the figure,

we

to the Y-axis,
the X-axis.
||

and

have,

OB = OD
PB = DO

+ O N + AF,
+ AN + PF.

and

OB = x, OD = a, O N == x cos 6, AF =
PB = y, DO = b, AN = x sin 0, PF =

But

hence, substituting,

we

cos
g>,

sin
<p

have,

=a+x
=
b
x
y
x

+ y cos
+ y sin

cos
sin

-}-

,^

q>

<p

for the required equations.

COR.

1.

If a

= 0,

have,

and

= x! cos
= x sin B

= 0,
-f-

-f

y cos
y sin

coincides with 0, and

<p

go

...

we

(2)

for the equations of transformation from a rectangular system


to an oblique
system, the ori-gin remaining the same.

PLANE ANALYTIC GEOMETRY.

54

= 90 + 6, coincides with
2. If a = 0, b = 0, and
and the new axes X and Y are rectangular. Making these
substitutions, and recollecting that
COR.

we

g>

cos

(p

sin

(f

= cos (90 + 0) = sin


= sin (90 + 0) = cos

0,

and

9,

have,

=x
y = a

y sin 9 \

cos

+ ?/ cos

sin

,^

one rectangular system


/or &e equations of transformation from
the
to another rectangular system,
origin remaining the same.
If we rind the values of x and y in equations (2)
NOTE.
in terms of x and y we obtain the equations of transforma
tion from an oblique system to a rectangular system, the

origin remaining the same.

EXAMPLES.

16 become when the


does the equation x*
if
?
45
of
an
angle
axes are turned through
Ans. The equation is unchanged.
2.

tion

What

1.

1
required the equa
The equation of a line is y = x
axes
to
referred
when
making angles of
line
same
the
of
;

45 and 135

with the old axis of

x.

Ans.

3.

when

-\A-

does the equation of the line in Example 2 become


an
referred to the old Y-axis and a new X-axis, making

What

angle of 30

with the old X-axis.


Ans.

2 y

= (V3 -

1)

2.

TRANSFORMATION OF CO-ORDINATES.
34.

To find the equations of transformation from a


rectangu

lar system to a polar system, the


origin

FIG.

Let
angle

55

(a, b)
g>

be the pole and

with the X-axis.

point on

it.

From

non-coincident.

20.

S the

initial line,

CM

Let

the figure,

and pole

we

making an

be any locus and


have,

any

OB = OD
BP = DO
But

OB = x,
BP = y,

+ O F,
+ FP.
=
OD a, O F = O P cos PO F = r cos (B +
DO = FP = O P sin PO F = r sin (0 +

g>)

b,

<p)

hence, substituting,

we

=a
=
b
y
x

have,

-f-

-(-

r cos

qp))

+?

r sin (B

for the required equations.

COR.

1.

If the initial line

usual to so take

it)

x
y

cp

= 0,

is

parallel to the X-axis

(it is

and

= a + r cos
= b + r sin

)
)

become the equations of transformation.


COE.
line

2.

If the pole is taken at the


origin 0,

made coincident with

the X-axis a

and the

initial

= 0, b = 0, and = 0.
g>

PLANE ANALYTIC GEOMETRY.

56

Hence, in this case,

s-rcosfl)
(3)

= r sin 6

will be the required equations of transformation.

35.

system

1.

To find the equations of transformation from a polar


to a rectangular system.

When

From

and

the pole

initial line coincides

equations

adding

by division

tan

origin are coincident,

and when the

with the X-axis.


2

cc

we

Art. 34,

(3),

+y

2
;

have, by squaring and

and,

for the required equations.

We

have, also, from the same

equations,

Vx +

2.

When

the pole

and

Vx + y

origin are non-coincident,

and when

the initial line is parallel to the X-axis.

From

a simi
equations (2) of the same article, we have, by

lar process,

= (x- ay +
tan. 6 = y^~~-

r*

x
cos

=x

sin0

=_y

,,

(y

also

b)

V(*
for the required equations.

ay

(y

TRANSFORMATION OF CO-ORDINATES.

57

EXAMPLES.

The rectangular equation of the circle is x + y = a


what is its polar equation when the origin and pole are coin
2

1.

cident and the initial line coincides with the X-axis ?


Ans.

The equation

2.

of a curve

is (x

+y

=a

= a.

(x

re

and initial being taken as


quired its polar equation, the pole
in the previous example.
a 2 cos 2 0.
Ans. r 2

Deduce the rectangular equation of the following curves,


assuming the origin at the pole and the initial line coincident
with the X-axis.

= a tan sec
^ws. o^ = a? y.
sec
4. r = a tan
= a y.
2

3.

;c

r2

=a

6.

sin 2
2

Ans.

5.

(x

a (cos
2
Ans. x 2
y
r ==

2
?/

=2a

scy.

sin 0)

=a

y}

(x

GENERAL EXAMPLES.
Construct each of the following
the origin to the point indicated, the
to the old, and reconstruct

straight lines, transfer

new axes being

parallel

1.

2.
3.
4.

= 3 x + 1 to (1, 2).
to (- 1, 2).
2 y - x - 2 =
_
i
=
to
4
(- 2, - 1).
y
y + x _f_i = o to (0, 2).

_|_

What do
referred to

5.

6.
7.

8.

= sx + b to d).
- 2).
y + 2 = to (2,
= mx to w).
y -4o; + c = 0to(d,0).

(c,

a;

?/

(Z,

the equations of the following curves become when


a parallel (rectangular) system of co-ordinates

passing through the indicated points ?


9.

10

"

11.

3 x2

+2y =
2

= 4 *(1

9 y2

6, (

0).

>>-

- 4x = 2

V2

12.

36

(3, 0).

/_ 2

PLANE ANALYTIC GEOMETRY.

58

What

13.

X-axis

is

Y-axis

is

does the equation x 2 -f- y 2


4 become when the
turned to the left through an angle of 30 and the
turned to the right through the same angle ?

14. What does the equation x


axes are turned through an angle of
2

15.

What

,y

=a
45

become when the

the polar equation of the curve y 2

is

pole and origin being coincident, and the


ing with the X-axis ?
16.

The polar equation

of a curve

is

2 px, the

initial line coincid

=a

(1

-f-

2 cos 0)

required
rectangular equation, the origin and pole being
coincident and the X-axis coinciding with the initial line.
its

Ans.

-f

(x

2 ax) 2

a 2 (x 2

-f

).

Required the rectangular equation of the following curves,


the pole, origin, initial line, and X-axis being related as in

Example
17.

16..

20.

asec 2 ?-.
2

cos 2

Ans. x 2
18.

19.

= a sin
r = aO.

0.

=a

2
.

21.

22.

r
r

= a sin 2 0.
-2r (cos + V3 sin

Find the polar equations of the


equations are
23.
24.

whose rectangular

= y (2 a - x).
4a x = y (2 a
x).
xs

loci

25.
26.

= a x - x\
x* -f y* = a*.
2

a*y

= 5.
6>)

THE CIRCLE.

CHAPTEE

59

V.

THE CIRCLE.
36.

THE

generated by a point moving in


remain at the same distance from a
will be observed that the circle as here de

circle is a curve

the same plane so


fixed point.

fined

is

It

as to

the same as the circumference as defined in plane

geometry.
37.

Given the centre of a

circle

and

radius

its

to

deduce

its

equation.
Y

Let C

(^ $)

be the centre of the

point on the curve.

OX

then

circle,

PM

to
||

and

OY

let

and

be any

CN

^^

= (*(, &
MP) = (x, y)

(OA, AC)

(OM,

Draw CA and
)

are the co-ordinates of the centre C.

are the co-ordinates of the point P.

||

to

PLANE ANALYTIC GEOMETRY.

60

Let CF

From

a.

CN
ON
NP

But

have,

+ NP = CP ... (1)
= (OM - OA) = -^,
= (MP - AC) = (y -^) and
=a
2

(a;

CP 2

Substituting these values in


2

(x

we

the figure,

-1^)

we

(1),

have,

-$) = a
2

(y

for the required, equation.


For equation (2)
relation existing between the co-ordinates of

the circle

(2)

expresses the

any point (P) on


between the co

hence it expresses the relation


ordinates of every point.
It is, therefore, the equation of the
;

circle.

If in (2)

we make x
a*

+ ,f
CC

or,

symmetrically,

=
=a

- 4-

= 0,

and y
.

=1

?/

have,

(3)

(4)

CL

we

for the equation of the circle when referred to


rectangular
axes passing through the centre.

Let the student discuss and construct equation

(3).

See

Art. 11.

COR.
factor,

1.

we

If

we transpose x z

have,
2
?/

i.e.,

(a

+ x)

in (3) to the second

x)

(a

in the circle the ordinate

is

member and

a mean proportional between

the segments into which it divides the diameter.


COR. 2. If we take L, Fig. 21, as the origin of co-ordinates,
and the diameter
as the X-axis, we have,

LC

H
= x = a and y = 0.

These values of x and y


(x

a)

in (2) give
2
?/

=a

2
,

2
2
2 ax
reduction, x
y
(5)
for the equation of the circle when referred to rectangular

or, after

THE CIRCLE.

61

axes taken at the left hand extremity of the horizontal


diameter.
38.

Every equation of the second degree between two varia


which the coefficients of the second powers of the
variables are equal and the term in xy is missing, is the
equa
bles,

in

tion

of a

circle.

The most general equation


these conditions obtain

ax 2

of the second degree in

which

is

+ ai/ + cx+dy+f=Q.

(1)

Dividing through by a and re-arranging, we have,


a
If to both

members we now add


c

4 a2
the equation

may

d*

4 a2

be put under the form

4 a2
this with (2) of the preceding article,

Comparing
that

it is

we

see

the equation of a circle in which


c
~a

d
~2~o

are the co-ordinates of the centre and

COR. 1. If ax 2 -{be the equation


-f ex
dy -f- m
of another circle, it must be concentric with the circle
repre
sented by (1) for the co-ordinates of the centre are the same.
ay"-

Hence, when the equations of

circles

have the variables in

PLANE ANALYTIC GEOMETRY.

62

their terms affected with equal coefficients, each to each, the

Thus

circles are concentric.

are the equations of concentric circles.

EXAMPLES.

What

is

of the

the equation

circle

when

the origin

is

taken.

What

1.

At D,

Fig. 21 ?

Ans.

x2

2.

At K,

Fig. 21 ?

Ans.

x2

3.

At H,

Fig. 21 ?

Ans.

x2

+ f - 2 ay = 0.
+ if + 2 ay = 0.
+ if + 2 ax = 0.

are the co-ordinates of the centres, and the values of

the radii of the following circles ?


4.

Ans.
5.

a; a-t-

2 ,*

7.

+2

2 z2

X2

?/

2, 3),

= 4.

= 2.

= 3.

4) a

= 5.

+ 4a;-6y-3=0.
(-

^tns.

(1, 1),

= 0.

8x

?/

Ans.

(2,0),

_ 6 x = o.
(3, 0),

8.

xz

?/

4 x -f 8

?/

= 0.
.

9.

x 2 H-

10.

x2

11.

x2
2

12.

13.

x2

2
?/

mz

+^ +G=
2/

+ = m.
4* = + =c +^
cz +
=/
2

?/

?/

2/

7/

wy.

(2,

THE CIRCLE.
Write the equations of the

circles

63
whose

and whose

radii

centres are
14.

= 3,

18.

(0, 1).

x*+y -2y = 8.
15. a -2, (1, - 2).
19.
Ans. x + if -2x + 4 y + 1 =0.
20.
16. a = 5, (- 2, - 2).
+ y -f 4 + 4 = 17.
17. a = 4, (0, 0).
21.
= 16.

= m,

= 5,

(b, c).

Ans.

b, (c,

d).

^4rcs.

ic

The radius

of a circle
-\-

is

what

is its

24.

(2, i).

equation

x 2 -f

common

centre the point

Find the equation of a

(2,

circle passing

?/

Write the equations of two concentric

have for their

A;,

if it is

4ic=2?

Ans.

?/

concentric with x 2

23.

&).

cc

22.

(7,

a;

?/

cc

circles

1).

= 21.
which

through three

given points.
39.

To deduce the polar equation of the

circle.

B
FIG.

The equation

of the circle
(X

-XY+

22.

when

(y

referred to

_ y Y = a\

OY,

OX

is

PLANE ANALYTIC GEOMETRY,

64

To deduce the
curve, then

polar equation let

be any point of the

(OA, AP) = (x, y)


- (* y )
(OB, BO )
= (r, 0)
(OP, POA)
,

= (/,
= r cos

(00 O OB)
,

From

A=

the figure,

OB =
hence, substituting,
(

a;

we

=r

cos

AP = y = r sin
BO = y = r sin

(r sin

- / sin

Squaring and collecting, we have,

+ sin 0) + r (cos + sin


=a
sin
r + r - 2 r/ cos (0 -

+ sin

ff

2
.

- 2 r/ (cos

cos

&e polar equation of the

is

i.e.,

^(cos

0,

0,

have,

r cos

r C os 6

a*

(1)

circle.

This equation might have been obtained directly from the


triangle*

COR.
tre

OO P.

1.

If

= 0, the initial line OX passes through the cen

and the equation becomes


r2

COR.

2.

Tf

+/

= 0,

2 rr cos

and /

= a,

=a

2
.

the pole lies on the circum

ference and the equation becomes

= 2 a cos
= 0, and / = 0,

COR.

3.

If

0.

the pole

is

at the centre

and

the equation becomes


r

40.

= a.

To show that the supplemental chords of the

each other.
perpendicular
The supplemental chords of a

circle are

to

circle are those chords

diameter and
pass through the extremities of any
other on the circumference.

which

intersect each

65

FIG.

23.

Let PB, PA be a pair of supplemental chords.


prove that they are at right angles to each other.
The equation of a line through B ( a, o) is

For a

line

through
y = s

Multiplying these,
if

(a, o),

We

wish to

we have

a).

(x

member by member, we have


a 2) ... (a)
ss (x*

for an equation which expresses the relation between the


co-ordinates of the point of intersection of the lines.

Since the lines must not only intersect, but intersect on the
circle

whose equation

this equation

above

is

must subsist

hence, dividing,

same time with equation

at the

(a)

we have

"1

or,

+ ss =

...

(1)

Hence the supplemental chords

of a circle are perpendicular

to each other.

Let the student discuss the proposition for a pair of chords


passing through the extremities of the vertical diameter.

PLANE ANALYTIC GEOMETRY.

66

To deduce the equation of the tangent

41.

FIG.

to the circle.

24.

Let CS be any line cutting the


Its equation is

circle in the points

(#

),

P"

(x",

y").

(x

Since the points (a/, y ),


the equations of condition
/**

(x",

/"I

~~

(Art. 26, (4)

are on the circle,

y")

/2

).

).

we have

These three equations must subsist at the same time hence,


subtracting (2) from (1) and factoring, we have,
;

(x

+
y
x

x")

(*

x")

y,

_|_

y"

.x"

(y

y")

(y

Substituting in the equation of the secant line

If

point

we now revolve the secant

when the

will approach

secant

;/

CS becomes

line

y")

"

it

becomes

upward about

P"

the

will finally coincide with it


tangent to the curve. But when

and

THE CIRCLE.
P

coincides with

in (3)

we

P",

x"

and

?/

?/";

hence, substituting

have,

y
or, after

67

y"

=-

(*

(*)

*")>

reduction,

+ yy _;..;..
"

a-x"

or,

(5)

symmetrically,

for the equation of the tangent.

SCHOL. The SUB-TANGENT for a given point of a curve is the


distance from the foot of the ordinate of the point of tangency
to the point in
in Fig. 24,

which the tangent intersects the X-axis

AT

its

value

we

have,

P".

make y

OT =
But

thus,

the sub-tangent for the point


To find
in the equation of the tangent (5) and

is

AT = OT - OA =

x"

x"

= an ~ x = V,//2
-2

sub-tangent

"2

To deduce the equation of the normal to the


The normal to a curve at a given point is a
dicular to the tangent drawn at that point.
42.

The equation
24, is

of

any

y"

through the point

line

= s (x -

x")

...

circle.

line

perpen

P"

(x",

y")

Fig.

(1)

In order that this line shall be perpendicular to the tangent


P"T, we must have
1

But

Art. 41, (4)

-f
s

88

= 0.

~
y"

hence,

we must have

=V
x"

PLANE ANALYTIC GEOMETRY.

68

Therefore, substituting in

y"

we

(1),

-*(*-

have,
(2)

*")

or, after

reduction,

yX

-X

"

y"

...

(3)

for the equation of the normal.


see from the form of this equation that the
the circle passes through the centre.

We

normal to

SCHOL. The SUB-NORMAL for a given point on a curve is


the distance from the foot of the ordinate of the point to the
In the circle,
point in which the normal intersects the X-axis.

we

see

from Fig. 24 that the


Sub-normal =

By methods

43.

the last two articles,


(x

to be

-x

x" .

precisely analogous to those developed in


we may prove the equation of the tangent

- x )* + (y- y Y = *

- x ) + (y - y )
)
and that of the normal to be
- x ) -(x(y -y")
(x

(x"

(y"

- y } = a*

-y
Let the student deduce these equations.
x")

(x"

(y"

...

(1)

(2)

EXAMPLES.
1.

dy

What is

+f=

0,

the polar equation of the circle ax 2


ex
ay*
the origin being taken as the pole and the X-axis

as the initial line ?

Ans.

r2

+ (- cos
\a

2.

x2

What

is

+ f = 25

equation of
(3,

- sin
a

\r
J

What

+ /a = 0.

the

tangent to the circle


of the subAns. 3 x
4y
25 J^.

The value

4) ?

tangent ?
3.

the

at the point

-f

the equation of the normal to the circle


37 at the point (1, 6) ? What is the value of the

x 2 -f if =
sub-normal

is

?
--

Ans. y

= 6x

1.

THE CIRCLE.
What

4.

x2

circle

are the equations of the tangent and normal to the


20 at the point whose abscissa is 2 and ordi-

if

-f-

69

Give also the values of the sub-tangent and


sub-normal for this point.
nate negative ?

Ans.

cc

- 4 = 20
=8
?/

Sub-tangent

?/

+4 =
= 2.
cc

sub-normal

Give the equations of the tangents and normals, and the


values of the sub-tangents and sub-normals, to the
following
circles
5.
6.

7.

+ ?f = 12, at (2, + V8). = 25, at (3, - 4)!


x +
= 20, at (2, ordinate +).
x +
x -f if = 32, at (abscissa +,
4).
x + y = a at
y = m, at (1, ordinate +).
= &, at (2, ordinate
x
+ = 18, at (m. ordinate -f).
= 45 and the
Given the circle x +
x*
z

?/

?/

8.

9.

10.

a?

-|-

11.

(6, c).

-j- ?/

12.

cc

).

7/

13.

+ =

line 2 y
x
2
required the equations of the tangents to the circle which are
parallel to the line.
?/

Ans

What

14.

x~

2
2/

f3*

{3*

+ 6y = 45.
+ 6y=-45.

are the equations of the tangents to the circle

= 45 which are perpendicular to the line


,

2y

-f-

=2?

(3y- 6x^45.

- 3 y = 45.
x + y = 9

16 x

The point

1
2
(3, 6) lies outside of the circle
the
of
the
required
equations
tangents to the circle which
pass through this point.
X
3
Ans.
A
4
3x
15.
y

16.

PLANE ANALYTIC GEOMETRY.

70

What

17.

(x

2)

equation of the tangent to the circle


the point (4, 4) ?
Ans. 2 x -f y
12.

the

= 5 at

3)

(y

The equation

18.

the

is

x2

circle

-\-

y*

of one of

=9

is

two supplementary chords of


x -|- 2, what is the equation

of the other ?

?/

-f 3

.x

== 9.

Find the equations of the lines which touch the circle


2
2 =
r* and which are parallel to y = sx -f- c.
a) -\- (y
b)

19.

(x

x 2 + ?/ 2
4 x + 4 ?/ = 9
of
the
normal
at
the
the
point whose
required
equation
abscissa == 3, and whose ordinate is positive.
Ans. 4 x
y = 10.

The equation

20.

of a circle

is

44. To find the length of that portion of the tangent lying


between any point on it and the point of tangency.

Let (xu
this point

be the point on the tangent. The distance of


1/1)
from the centre of the circle whose equation is

(x
V(a>!

-f-

(y

-xY+

<*?

- y y.

(y,

is

evidently

See Art. 27,

(1).

But

this distance is the hypothenuse of a right angled tri


whose
sides are the radius a and the required distance
angle

d along the tangent


d*

COR.

1.

If

=
d2

as

it

hence
(x,

-xy+

and y

= 0,

(2/1

then

= x^ + ^ - a

- yY - ^
(1)

...

(i)

becomes

2
.

(2)

ought.

45.

To deduce the equation of the radical axis of two given

circles.

The RADICAL AXIS OF TWO CIRCLES is the locus of a point


from which tangents drawn to the two circles are equal.

THE CIRCLE.

71

FIG.

Let

(x
\

+ (v
+ (y

x")

y/

\*/

(x

25.

}
/

y")

=a
=b

2
,

be the given

Let P (XD yi) be any point on the radical axis


the preceding article, we have,

d2 =
d2 =
.:

6/1

y")

(x"

(y"

Calling, for brevity,

w iH

y")

y)

-f (y t

hence, reducing,

- x x,+2

-f-

(#15 2/i)

+ Vl - y + yi 2

x")

( Xl
2

(x,

by definition
2

-x )

(x,

y ) y,
2
a2
b

we

then from

a2
b2

a2

( Xl

x")

have,
2

x"

circles.

-y

2
y"

the second

member m, we

see that

satisfy the equation.


2

(x"

-x )x+2

(y"

=m

(1)

But (xu 1/1) is any point on the radical axis hence every
It is, therefore, the re
point on that axis will satisfy (1).
;

quired equation.

COR.
then,
is

1.

If c

and

be the equation of two circles,

the equation of their radical axis.

PLANE ANALYTIC GEOMETRY.

72

2. From the method of deducing (1) it is easily seen


the two circles intersect, the co-ordinates of their points
of intersection must satisfy (1) hence the radical axis of two

COR.

that

if

intersecting circles is the line joining their points of intersection,

PA,

Fig. 25.

Let the student prove that the radical axis of any two
circles is perpendicular to the line joining their centres.

To show that the radical axes of three given


a common point.

46.

circles in

tersect in

Let

= 0,

= 0,

and

c"

be the equations of the three circles.


Taking the circles two and two we have for the equations of
their radical axes
c
c

-c =
- =
_c =o

c"

(1)

...

(2)

"

(3)

evident that the values of x and y which simultaneously


will also satisfy (3) hence the proposition.
and

It is

satisfy (1)

(2)

The intersection of the radical axes of three given circles

THE RADICAL CENTRE

called

is

of the circles.

EXAMPLES.
Find the lengths of the tangents drawn to the following
circles
1.

(x

2)

(y

3)

16 from

(7, 2).

Ans.
2.

x>

(y

+ 2)

10 from

4.
5.

_ a y + if = 12 from
x 2_f2/ 2_2^+4?/-2 from
x + y = 25 from (6, 3).

VlO.

d=^/3.

(b, c).

(a-

(3, 0).

Ans.

(3, 1).

Ans.

V20.

THE CIRCLE.
6.

7>

g.

+y 2

x2

(3

a )2

+y

2 x

10 from

73

(5, 2).

= 3.

Ans. d

(^

6)2

4y

rom

c f

10 rom
f

(d,

/).

(0, 0).

Give the equations of the radical axis of each of the follow


ing pairs of circles

9.

10.

13.

+_ y
3)2
| (x
(

x2

2
|X

_^_

*
|x

_|_ (

^2

4y
^2

_|_

_
_

2 y

3)2

0.

16

+- y = 16.
=
I) +

^s.

o.

-4?w.

= 2 y.

a?

a?

i-

= 0.

1 (x

= 0.
_9 _

2/

a?.

14.

Find the co-ordinates of the radical centres of each of the


following systems of circles
15.

= 9.
- 2 = 25.
2

16.

+ if - 4 x + 6 y - 3 =
-4* = 12.
=
6
| x
=
(.T +
= 9.
J
I) + f
(^
__ 2
4. 4 y = 10.
( ^2 + yj
x + if - k* =
-j= d.
//
(

x-

_j_

?/

_|_

y>

7.

7/

a,

18.

G,

^rcs.

(1,

3).

0.

x8 4-3^
.2

17.

-4w.

ft

?/

?H

-f-

//

I)-

PLANE ANALYTIC GEOMETRY.

74
47.

To find the condition that a straight

= sx + b
+ =

line

2
a2
must fulfil in order that it may touch the circle x
y~
In order that the line may touch the circle the perpendicu
lar let fall from the centre on the line must be equal to the
.

radius of the circle.

From

= bcosy =
p

r2 (1

hence,
is

we have

Art. 21, Fig. 13,

= VI

+ tan.

=r=

+s =b
2

(1)

the required condition.


COR. 1. If we substitute the value of b

the equation y

-==

sec y

= sx +
y = sx

b,

AVC

_j_

drawn from

(1) in

have

Vl +s

...

(2)

for the equation of the tangent in terms of its slope.

48. Two tangents are drawn from a point without the circle ;
the points oftangency.
required the equation of the chord joining

FIG.

26.

y ) be the given point, and


tangents through it to the circle.
Let

(x

let

PT",

P P,

be the

THE CIRCLE.

75

deduce the equation of


The equation of a tangent through
It is required to

PP

/X
.

is

P"

y")

(#",

\yy^_

xx"

Since

on this

is
(a/, ?/)

the equation

line, its co-ordinates

must

satisfy

hence

The point

rt

(x",

1/1/ =
1
+ ^~
a

x"

2
aT

?/"),

therefore, satisfies the equation

it is a point on the locus represented by


similar
(1).
source of reasoning will show that P is also a point of this
locus.
But (1) is the equation of a straight line hence, since
.-.

it

is satisfied

for the co-ordinates of both

P"

equation of the straight line joining them.


the required equation.
49.

chord of a given

and P,
It

circle is revolved

is,

it is

the

therefore,

about one of

its

points ; required the equation of the locus generated by the


point of intersection of a pair of tangents drawn to the circle at
the points in which the chord cuts the circle.

Let

Fig. 27, be the point about which the chord


It is required to find the equation of the
locus generated by Pj (x l5 yj, the intersection of the tangents
revolves about P
1?
1? as the line P

P AB

(x

),

revolves.

AP BP
From

AB

the preceding article the equation of the chord

X^X

?/!?/ =

-,

^"""^

Since

(a/,

y)

on this

is

a2

xx
hence
Cb

we have

line,

a"

-\

y y == l
1
%
Cf

/1

(1J

AB

is

PLANE ANALYTIC GEOMETRY.

76
is satisfied

for the co-ordinates of

the locus represented by

(1).

any pair of tangents drawn

But

(x lt

y-^)

is

hence

lies

on

the intersection of

to the circle at the points in

hence (1)
position, cuts the circle
will be satisfied for the co-ordinates of the points of intersec

which the chord, in any

tion of every pair of tangents so drawn.

the equation of the required


(1) is, therefore,
observe that equation (1) is identical with (1) of
the preceding article hence the chord PP" is the locus whose

Equation

locus.

We

equation

we

sought.

The point P

(**

+& =

THE POLAR
1-1-21
(i

(x

y)

is

and the

of the point

called
line

THE POLE

PP"

(x>,

y>)

of the line

(^ + ^f

1\

is

PP"

called

with regard to the circle

THE CIRCLE.

77

the
principles here developed are perfectly general,
the
circle.
within
or
be
without^ on,
pole may
Let the student prove that the line joining the pole and the
centre is perpendicular to the polar.

As the

The terms pole and polar used


no connection with the same terms used
NOTE.

I.

co-ordinates, Chapter

If the polar of the point P (x y ), .Fig. 27, passes through


XD yi), then the polar of P (x 1? yi) will pass through P
1

50.

PI

The equation

of the polar to

xx
~a*

In order that

yu

But
f

(x

(x

) is

y^ may be on

(aj 1?

this line,

we must

have,

a2

this is also the equation of condition that the point


f
y ) may lie on the line whose equation is

^ _i_M

"

a2

a2

But

^~

a2

in this article have


in treating of polar

this is the equation of the polar of P! (x^ y-^

hence

the proposition.

To ascertain the relationship between the conjugate diam

51.
eters

of the

A pair

circle.

to be conjugate when they are


the curve is referred to them as axes its
equation will contain only the second powers of the variables.

of diameters are said

so related that

when
x2

Let

2
?/

ascertain

OY OX
,

a2

(1)

its centre and axes.


becomes when referred to
axes making any angle with each other, we must

be the equation of the

To

what

circle,

referred to

this equation

substitute in the rectangular equation the values of the old

PLANE ANALYTIC GEOMETRY.

78

co-ordinates in terms of the new.

have

=x
=x

x
y

+y
+y

cos 9
r

sin

From

the

+ 2 x y cos

((f

FIG.

1,

we

cos 9
sin cp

equations of transformation.
values in (1) and reducing, we have,

for

Art. 33, Cor.

Substituting these

+x =a
2

0)

2
.

(2)

28.

diameters
Now, in order that OY OX may be conjugate
the term containing x y in (2)
that
related
so
be
must
they
must disappear; hence the equation of condition,
,

- 6) =
= 90, 0*9-e
9

cos
...

(cp

The conjugate diameters of the


As there
dicular to each other.

circle are therefore

are an infinite

perpen

number

of

the circle which satisfy the condition of being


pairs of lines in
the circle there
at right angles to each other, it follows that in
diameters.
are an infinite number of conjugate

THE CIRCLE.

79

EXAMPLES.
1.

-|-

Prove that the line y = V3 x -+- 10 touches the circle


2
= 25, and find the co-ordinates of the point of tangency.
?/

_ y
f5
__

2.

What must be the value of b in


2
may touch the circle x +

=2x +b

order that the line

?/

16

^7i.
3.

What must be the value of


4 may touch the circle x

= sx

4.
_|-

The
y

V80.

-L,

in order that the line

=2?

-f if

Ans.

#2

5
3,

-L_

V7.

slope of a pair of parallel tangents to the circle


16 is 2 required their equations.
;

Ans.

^j

= 2x -

V80.

Two tangents are drawn from a point to a circle required


the equation of the chord joining the points of tangency in
each of the following cases
;

5.

From

6.

From

7.

From

8.

From

(4, 2)

(3,

to x

4) to or

(1, 5) to

+ if = 9.

+ if =
+

4x

+2y =

^4ws.

3x

+ 4 y = 8.

8.

=c

?/

Ans.

16.

Ans. x
(a, 6) to

cc

7/

9.

10.

Of

= 16.

2
.

v4?is.

What

5 y

-\-

9.

ax

by

-{-

=c

are the equations of the polars of the following points


(2, 5)

Of

(3,

with regard to the

4)

circle

with regard to the

circle

+ y- = 16 ?

cc

5 y

-,

16+16
x + if = 9 ?
^715. 3 x + 4 y = 9.
2

PLANE ANALYTIC GEOMETRY.

80
11.

Of

(a, b)

with regard to the

circle

x2

-\-

What
12.

are the poles of the following lines

Of 2 x

-f 3 y

= m?

y-

Ans.

ax

= 5 with regard to the circle x

-f

Ans.
13.

Of ~

+ y = 4 with regard to the


Jg

14.

Of y

2
-

= 25 ?

(10, 15).

circle

+ fg = 1 ?

= sx + 6

= m.

by

-\-

2 4 )>

with regard to the circle

Ans

(-^r>i

Find the equation of a straight line passing through


x 2 -\- if
3 x -\- 4 y = 0.
0) and touching the circle
Ans. y = - x.

15.
(0,

GENERAL EXAMPLES.
1.

Find the equation of that diameter of a


drawn parallel to y = sx + b.

circle

which

bisects all chords

Ans.
2.

line

sy

+#=

0.

Required the co-ordinates of the points in which the


x-|-l = intersects the circle

2y

3. Find the co-ordinates of the points


drawn through (3, 4) touch the circle

in

which two

lines

and the
[The points are common to the chord of contact
circle.]

THE CIRCLE.
4.
(4,

The centre

0)

required

which touches the Y-axis

of a circle
its

81

equation.
2

Ans.

(x

4)

is

at

+ y = 16.
2

Find the equation of the circle whose centre is at the


= x -|- 3 is tangent.
origin and to which the line y
Ans. 2 x2 + 2 y* = 9.
5.

+y =

6.

Given x 2

2
16 and (x
4 required the
if
5)
equation of the circle which has their common chord for a
diameter.

7.

Required the equation of the circle which has the dis


(3, 4) from the origin as its diameter.
Ans. x* -j- ?/ 2
4 y = 0.
3x

tance of the point

8. Find the equation of the circle which touches the lines


x.
0, and y
3, y
represented by x

9.

Find the equation of the

points

(-

(1, 2),

(-

2, 3),

circle

- 1).

1,

which passes through the

10. Required the equation of the circle which circumscribes


the triangle whose sides are represented by y
4 x,
0, 3 y
4x
and 3 y
6.
Ans. x 2 -f ?/ 2
0.
| x
1| y

=-

Required the equation of the circle whose intercepts


are a and b, and which passes through the origin.
11.

12.
is

The

points

in the line

13.

circle
14.

The point
x2

ax

(4,

on a

whose centre

its

required
Ans. 2 x 2

2) is the

(3.

6) lie

circle

by

equation.

+2

2
?/

17 x

= 30.

middie point of a chord of the

= 16 required the equation of the chord.


= 16 and the chord y 4 x = 8.
x +

-\- if-

Given

and

(1, 5)

=x

= 0.

x 2 -f y z

Ans.

?/

Show

that a perpendicular from the centre of the circle bisects the


chord.
15.

Find the locus of the centres of

pass through

(2, 4), (3,

2).

all

the circles which

PLANE ANALYTIC GEOMETRY,

82
16.

Show

that

if

the polars of two points meet in a third


is the pole of the line
joining the first

point, then that point

two points.
17.

= 8,

Required the equation of the


and whose sub- normal = 2.

circle

whose sub-tangent

= 20.

x 1 -f y 2

Ans.

Required the equation of ihe circle whose sub-normal


= 2, the distance of the point in which the tangent intersects
the X-axis from the origin beinr = 8.
18.

x2

Ans.
19.

ax 2

Required the conditions

in

order

that

-f- ?/

the

= 16.

circles

+ aif + cx + dy + e = Q and ax + af + kx + ly + m =

may

be concentric.
Ans.

20.

k,

I.

Required the polar co-ordinates of the centre and the

radius of the circle


r2

2 r (cos

^/3 sin

0)

= 5.

Ans.
21.

line of fixed length so

moves that

60)

(2,

its

= 3.

extremities

remain in the co-ordinate axes required the equation of the


circle generated by its middle point.
;

22.

Find the locus of the vertex of a triangle having given


= 2 a and the sum of the squares of its sides = 2 b 2

the base

Ans.

x2

-j-

&

a2

Find the locus of the vertex of a triangle having given


the base = 2 a and the ratio of its sides
23.

Ans.

circle.

n
24. Find the locus of the middle points of chords drawn
from the extremity of any diameter of the circle

THE PARABOLA.

CHAPTER

83

VI.

THE PARABOLA.
THE

parabola is the locus generated by a point moving


same plane so as to remain always equidistant from a
fixed point and a fixed line.
The fixed point is called the Focus the fixed line is called
the DIRECTRIX the line drawn through the focus perpendic
ular to the directrix is called the Axis the point on the axis
midway between the focus and directrix is called the VERTEX
52.

in the

of the parabola.
53.

To find the equation of the parabola, given the focus and

directrix.

Let

EC

be the directrix and let

the axis of the curve, and the tangent

be the focus.

OY

drawn

Let OX,

at the vertex

PLANE ANALYTIC GEOMETRY.

84

0, be the co-ordinate axes.


to OY, PB

and draw

PA

||

(OA, AP)

From

Take any point P on the curve


OX and join P and F. Then

to

||

y) are the co-ordinates of P.

(x,

FAP, we have
2
... (1)
generating the curve, we have

the right angled triangle

= AP = FP - FA
2

if

But from the mode of

FP = BP = (AO
2

and from the

+ OD) =

FA = (AO - OF) =
2

Substituting these values in

or, after

reduction,

As equation

true for every point

then

= 2 px

hence

(x

(x

OF)

OD = OF =

true for

is

(3)

(x

2
,

OF)

2
.

we have

(1),

+ OD) -

DF =p,

Let

+ OD)

we have

figure,

if

2
.

(2)

hence

(3)

of the parabola it
the equation of the curve.

any point

it is

is

COB. 1. If (x y ) and (x
are the co-ordinates of any
two points on the parabola, we have,
1

hence

2
?/

2
:

y"

\:x

y")

x"

i.e., the squares of the ordinates of any two points on the


bola are to each other as their abscissas.

para

SCHOL. By interchanging x and y, or changing the sign of


the second member, or both in (3), we have
2
y
2px for the equation of a parabola symmetrical

=~

with respect to X and extending to the left of Y;


x 2 = 2 py for the equation of a parabola symmetrical with
respect to Y and extending above X.
x2 =
2 py for the equation of a parabola symmetrical
with respect to Y and extending below X.
Let the student discuss each of those equations.
See
Art. 13.

THE PARABOLA.
54.

85

To construct the parabola, given the focus and

FIG.

First Method.

Let

DR

directrix.

30.

be the directrix and

let

be the

focus.

From F

FD on the directrix it
Take a triangular ruler ADC
and make its base and altitude coincide with the axis and
Attach one end of a string, whose
directrix, respectively.
the perpendicular

let fall

will be the axis of the curve.

is AD, to
the other end to a pin fixed at F.
Place
the point of a pencil in the
loop formed by the string and
stretch it, keeping the point of the
pencil pressed against the

length

base of the triangle. Now, sliding the


triangle up a straight
edge placed along the directrix, the point of the pencil will
describe the arc OP of the parabola for in
every position of
the pencil point the condition of its
being equally distant
from the focus and directrix is satisfied. It is
easily seen, for
;

instance, that

FP = PD

when

the triangle

is

in the position

ADC

that

Second Method.

Take any point C on the

axis

and erect

PLANE ANALYTIC GEOMETRY.

86

the perpendicular P CP.


as a centre and DC (=

With F

Measure the distance DC.

as a radius describe the arc of a

FP)

P CP in P and P P and F will be points of


the parabola. By taking other points along the axis we may,
by this method, locate as many points of the curve as may be
cutting

circle,

desired.

55. To find the Lotus-rectum, or parameter of the parabola.


The L AT US-RECTUM, or PARAMETER of the parabola, is the

double ordinate passing through the focus.


The abscissa of the points in which the latus-rectum pierces

the parabola

is

2i

Making

this substitution in the equation

= 2px
*
= 2^y* = 2 p
2 y = 2 p.
2

?/

we have
Hence
COR.

1.

Forming a proportion from the equation


if

i.e.,

any

= 2px,
(

we have

x:y:\y. 2p;

the latus-rectum of the parabola is a third proportional to


abscissa and its corresponding ordinate.

EXAMPLES.
Find the latus-rectum and write the equation of the parab
ola which contains the point
1.

5.

8,

?/-

Ans.
4.

What
is it

is

?/

x.

(_,

= b- x.
a

2).

-,?/=a
parabola x* =

Ans.

the latus-rectum of the

defined in this case ?

b*

8,

(a, I).

x.

(-2,4).
Ans.

How

3.

(2,4).

Ans.
S.

-x.
a

THE PARABOLA.

87

6. What is the equation of the line which


passes through
the vertex and the positive extremity of the latus-rectum of
any parabola whose equation is of the form if
2px ?

Ans. y
7.

The focus

of a parabola

vertex of the curve

what

is

is its

when symmetrical with

(a)

"

"

at 2 units

= 2 x.

distance from the

equation
respect to the X-axis ?

"

"

"

(b)

Ans.

(a)

"

Y-axis?

= 8x,

(b)

x*

y.

Construct each of the following parabolas by three differ


ent methods.
8.
9.

12.
2
?/

10.

x2

11.

x2

= 6 y.
= - 10 y.

What

are the co-ordinates of the points on the parabola


where the ordinate and abscissa are equal ?
Ans. (0, 0), and (6, 6).

=6x
13.

x2

= Sx.
- 4 x.
if =
if

Required the co-ordinates of the point on the parabola


whose ordinate and abscissa bear to each other the

=4y

ration 3

Ans.

2.

What

(6, 9).

the equation of the parabola when referred to


2
the directrix and X-axis as axes ?
Ans. y 2
2px p
14.

is

Find the points of intersection of the following


15.

if

16.

x2

= 4 x and 2 y

Ans.

17.

18.

= 0.
(0, 0), (16, 8).

= 6 y and y x 1 = 0.
8 x and
y* =
+ 3 = 0.
=
= 8.
2x and x +
if
a:

?/

Ans.
19.

x*

20.

x2

(2, 2), (2,

= - 4 y and 3 x + 2 if = 6.
= 4 y and if = 4 x.
2

- 2).

PLANE ANALYTIC GEOMETRY.

88

To deduce the polar equation of

56.

the parabola, the focus

being taken as the pole.

The equation
To

we have

FX

= + r cos
= r sin

0.

0.

Substituting these values in

(1),

we have

= p~ + 2pr cos
sin
= 1 cos 9
= p -\-2pr cos 9 + r cos 9 = (p + r cos 0)
r cos
r =p
2

9.

?-

But

sin

r2

.-.

.-.

2
,

9,

-\-

solving,
1

cos B

the required equation.

We
Let

might have deduced this value directly as follows


P (r, 9) Fig. 29 be any point on the curve then
;

= D A = DF -f FA = p + r cos 9
r=p + rvosO.
r =
cos 9
1
9 = 0, r = oo
e = 90, r = p.
9 = 180, r =
|
9 = 270, v = p.
9 = 360, r =

FP
i.e.,

Hence
COR.

*-

If
If
If

If
If

An

-,

1.

is

and the pole F

1,

is

Fig. 29,

for the equation of transformation, Art. 34,

or,

OY, OX,

(1)

refer the curve to the initial line

Cor.

of the parabola referred to

f = 2px

oo.

these results.
inspection of the figure will verify

THE PARABOLA.

89

To deduce the equation of the tangent to the parabola.


be the points in which a secant line cuts
(V, y ),

57.
If

y")

(x",

the parabola, then

y-y =

,~

will be its equation.

parabola,

(-**)

",

(!)

Ou

OC-

Since (V,

?/),

(#",

y")

are points of the

we have
y

2
/2

7/

= 2px
= 2X

...

(2)

3)

These three equations must subsist at the same


hence, subtracting (3) from (2) and factoring, we have
y
x

i.e.,

y"

time

2p

==
f

-x"

+y"

Substituting this value in (1), the equation of the secant

becomes

When

the secant, revolved about


y ) coincides with

(x",

to the parabola (x

or,

y".

Making

y"},

this substitution in (4),

simplifying, recollecting that


yy"

=p(x+x")

2
y"

=2

...

becomes tangent
hence x
r

(x",

y")

we

px".

x",

have,

we have

(6)

for the equation of the tangent to the parabola.

To deduce the value of the sub-tangent.


Making y = in (6), Art. 57, we have
= OT, (Fig. 31)
x = -

58.

x"

which the tangent intersects


But the sub-tangent CT is the distance of this

for the abscissa of the point in

the X-axis.

point from the foot of the ordinate of the point of tangency;


i.e., twice the distance just found; hence

Sub-tangent

=2

x"
;

PLANE ANALYTIC GEOMETRY.

90
i.e.,

the sub-tangent

is

equal

to

double the abscissa of the point

of tangency.
59.

The preceding

principle affords us a simple method of


constructing a tangent to a parabola at a given point.
Let
be any point of the curve. Draw the ordinate P"C, and measure 00.
Lay off OT = 00.
P"

(a;",

y")

FIG.

A
at

line joining

T and

P"

31.

will be tangent to the parabola

P".

60.

To deduce the equation of the normal

The equation

of any line through

= s(x-

y-y"

We have found Art. 57,


s

to the

P"

(#",

x")

y"}

Fig. 31,

(1)

hence, for the slope of the normal

,P

is

(5) for the slope of the tangent

~7"

.__*:.

parabola.

P"N,

we have

P"T

THE PARABOLA.
Substituting this value of

-y"

for the equation of the

=-

we have

s in (1),

^ (*

normal

91

(2)

*")

to the

parabola.

To deduce the value of the sub-normal.

61.

Making y

in (2) Art. 60,

we

have, after reduction,

x=*p+af = ON; Fig. 31,


Sub-normal = NC = p
.-.

Hence
to the

-\-

the sub-normal in the parabola

is

x"

x"

constant

= p.

and equal

semi-parameter FB.

62.

To

shoiv that the tangents

drawn

at the extremities of

the latus rectum are perpendicular to each other.


The co-ordinates of the extremities of the latus-recturn are
?
,

for the upper point,

and

for the lower point.


j

Substituting these values successively in the general equa


tion of the tangent line, Art. 57 (6), we have

yp

= plx

-yp=plx +
or, cancelling,

x+|

(1)

y=-*-f

(2)

for the equations of the tangents. As the coefficient of x


in (2) is minus the reciprocal of the coefficient of x in
(1), the
lines are perpendicular to each other.

COR.
that x

1.

Making y =
?.

in (1)

and

(2),

we

find in each case

hence, the tangents at the extremities of the

PLANE ANALYTIC GEOMETRY.

92

and the
same point.

latus-rectum
in the

The values

directrix meet the axis


of the parabola

of the coefficients of

these tangent lines

make

x in

angles of 45

(1) and (2) show that


with the X-axis.

63. To deduce the equation of the parabola when


referred to
the tangents at the extremities of the latus-rectum as axes.

FIG.

The equation

of the parabola
if = 2px

We

wish to ascertain what

curve

is

Let

DY DX

referred to
(x

when

referred to

the figure,

OC

CP )

we

OY, OX,

is

(1).

this equation
as axes.

becomes when the

be any point of the curve

i/)

(OC,

From

32.

= (x,

y),

and (DC

then, Fig. 32
,

CP

have,

= DC - DO = DK + C M - DO;

(a*

tf).

THE PARABOLA.

93

but

DK = x

= r^C, C M = /cos 45 =

cos 45

V2

hence

V2

V2

Substituting the values of x and


(2/

- *T -

V2

?/

DPF, we

we

in (1),

(C

In order to simplify this expression


the triangle

= MP - C K

..,

|
*

DO =|

-\

CP

have, also,

V2

p
--^=-- 5r
2
V2

JK

V2

We

-X-,

let

-^

2/

have,
2

DP = a

then from

(2)

have,

DF =p =

a cos 45

-^=-

V2<

Substituting this value of

by

2,

we

have,

x )2

(/

?/

or,

Adding 4

a?

ic

= 2a
2

a:

.-.

transposing, x
...

or,

is

a2

4- ?/)

= 4;ry,

<X+/-a)

(V

and multiplying through


,

/+

a2

= 0.

both members, the equation takes the form

?/ to

or

in (2)

+y
-[-

^x

2x *y *

yx

-J-

-+- ?/

a;

?/

=a
...

symmetrically, dropping accents,

the required equation.

^
;

(3)

PLANE ANALYTIC GEOMETRY.

94

/EXAMPLES.

What is the polar equation of the parabola, the pole


1.
being taken at the vertex of the curve ?
Ans. r
2p cot cosec 0.

Find the equation of the tangent to each of the following


value of the subtangent in each case
parabolas, and give the
2
Ans. y = x + 1 2.
= 4 x at (1, 2).
2.
:

3.

x2
2

4.

v/

5.

x2

= 4 y at (- 2, 1).
= - 6 x at
6, ord
= 8 y at (abs +,
= 4 ax at (a, 2 a).
=
at (m, m).
= ^y at (abs +,
(

6.

?/

7.

8.

a;

9.

x*

y + 1 =
=3
Ans. 2 y +
^rcs. x + y = 2

Ans.

a;

oj

+).
2).

2.

12.

4.

??ia;

= 2py

at

(abs-,
\

p).

p8

Write the equation of the normal to each of the following


parabolas
10.
11.

12.

13.

= 16 x at (1, 4).
To x = - 10 y at (abs +, - 2).
mx at
To if =
m, m).
To x = 2 m?/ at ofo To

if
2

\
\

*
of a parabola is x* _L_ y
?
curve
the
of
are the co-ordinates of the vertex
14.

The equation

Ans.

-a, -- a
4
V4
I

= 4x

and the line y


which is,
the
of
the
tangent
equation
required
15.

Given the parabola

if

what

to the line,
(a) parallel
(b)

perpendicular to the line.


x
Ans. (a) y
1, (b) y

= +

+ + 1 = 0.
a;

THE PARABOLA.

95

16. The point (1, 2) lies outside the parabola


if = 6x;
what are the equations of the tangents through the point to

the parabola ?

The point (2, 45) is on a parabola which is symmetri


with respect to the X-axis required the equation of the
parabolk, the pole being at the focus.
- 2 V2) x.
Ans. if
(4
17.

cal

18. The subtangent of a parabola


10 for the point (5, 4)
required the equation of the curve and the value of the sub
normal.

Ans.

64.

The tangent

focal line

drawn

to the

= ^x;*.
5

>

parabola makes equal angles with the

the point of tangency

to

y*

and

the axis of the

curve.

From

We

Fig. 31

we

have,

FT

= FO + OT =

4-

x".

have, also,

= DC = DO + OC = f +
FT =

FP"

.-.

The

triangle

x".

FP".

FP"T is
FP"T

therefore isosceles and


FTP".

65.

To find

the condition that the line

in order to touch the parabola y 2

= 2 px.

= sx + c must fulfil

Eliminating y from the two equations, and solving the


resulting equation with respect to x, we have,

- SC

(C8

- PY ~ C S

2 2

for the abscissae of the points oflntersection of the


parabola
and line, considered as a secant.
When the secant becomes

PLANE ANALYTIC GEOMETRY.

96

a tangent, these abscissas become equal but the condition for


in the numerator of
equality of abscissas is that the radical
;

(1) shall

be zero

hence

c s

= 0,

= --

or, solving,

2^

(cs

the condition that the line must

is

fulfil in

order to touch the

parabola.

COR.

we

1.

Substituting the value of


sx
c,
y

= +
V = sx +

have,

c in

the equation

(2)

^-S

for the equation of the tangent in terms of

its slope.

To find the locus generated by the intersection of a tan


to it from the focus as the point of
gent, and a perpendicular
the curve.
around
moves
tangency
66.

The equation

of a straight line through the focus

I?

is
)

In order that this line shall be perpendicular to the tangent

we must
hence

have,

= sx + L
=

(2)

+ ^-

(3)

the equation of a line through the focus perpendicular to


the tangent. Subtracting (3) from (2), we have
is

or,

= 0,
=

is the
But x
equation of the required locus.
the perpendiculars from the
Y-axis
the
of
hence,
equation

for the

THE PARABOLA.
focus

to the

97

on the
tangents of a parabola intersect the tangents

Y-axis.

67.

To find

the locus generated by the intersection of two tan


to each other as the point of tan-

gents which are perpendicular


curve.
gencij moves around the

The equation

of a tangent to the parabola

The equation

= 8X +
f-g

-l*-fA

...*

=-

we

(1),

|.

is

...(2)

Subtracting (2) from

Art. 65 (2),

(1)

of a perpendicular tangent

is,

have,

(3)

the equation of the required locus. But (3) is the equa


hence, the intersection of all perpendicu
lar tangents drawn to the parabola are points of the directrix.

is

tion of the directrix

68. Two tangents are drawn to the parabola from a point


without ; required the equation of the line joining the points of

tangency.

be the given point without the parabola, and let


f
Since (x y is
X
be the points of tangency.
( 2, 2/2)
(X
on both tangents, its co-ordinates must satisfy their equations

Let (V,
>

?/)

2/")?

~)

hence, the equations of condition,

y"

2/2/2

The two points

yy
r

yy

+X"\

+x

(x

of tangency

satisfy

or

=p(x

=P

=p
=p

(*
(x

+
+x

z ).

(x",

y"),

(x 2

2/ 2 )

must therefore

),

(1)

Since (1) is the equation of a straight line, and is satisfied


for the co-ordinates of both points of tangency, it is the
equation of the line joining those points.

PLANE ANALYTIC GEOMETRY.

98
69.

To find the equation of the polar of the pole

(x,

with

2 px.
parabola if
The polar of a pole with regard to a given curve is the line
generated by the point of intersection of a pair of tangents
drawn to the curve at the points in which a secant line through

regard

to the

the pole intersects the curve as the secant line revolves about the
pole.

a course of reasoning similar to that of Art. 49,


prove the required equation to be

By

As the reasoning by means


fectly general, the pole

may

we may

which (1) is deduced is per


be without, on, or within the

of

parabola.

COR.

1.

If

we make,

hence, the directrix

is

in (1), (x

r
,

= | 0\
I
\z
(

we have

the polar of the focus.

70. To ascertain the position and direction of the axes,


other than the axis of the parabola and the tangent at the
be referred its equation will
vertex, to which if the parabola

remain unchanged in form.

FIG.

33.

THE PARABOLA.
Since the equation

to retain the

is

99

form

= 2px ... (1)


y*
y = 2p x ... (2)
f

let

when referred to the


we are now seeking.

be the equation of the parabola

axes,

It is
whose position and direction
be
the
whatever
that
obvious at the outset
position of
may
must be
-axis
Y
new
the
each
to
the axes relatively
other,
on the
must
be
new
the
and
the
to
origin
curve,
tangent
0, a
curve for, if in (2) we make x = 0, we have y =
;

which we can only account for by assuming the Y -axis


and the new origin in the positions indicated. This conclu
verified by the analysis which
sion, we shall see, is fully
result

follows.

Let us refer the curve to a pair of oblique axes, making


in the
any angle with each other, the origin being anywhere

The equations

plane of the curve.


Art. 33

(1),

= a -j- x cos 6 + y cos


= b x sin + y sin
r

-f-

q>

cos
<p)

Now,

+2

(1),

(b

sin

-f-

we have,
x

sin 2

(9

+2

(a)

sin

(b)

sin 2

(d)

= 0, then sin

and

(b sin

sin
g>

= 0.
2 pa = 0.

b sin

g>

sin
g>

q>

satisfied

= 0.
cos

<p

= 0.

and sin 2 = 0; i.e., conditions


assumed value of 6. But is
-axis makes with the old X-axis

(b) are satisfied for this

the angle which the new


hence, these axes are parallel.
If (a, b) be a point of the parabola ?/ 2
pa is an analytical expression of the fact

= 2px,

that the

we must have the following conditions

(d)

(a)

(1),
q>

cos 0) x -f b
2 pa
(3)
in order that this equation shall reduce to the same

form as

If

q>

g>.

Substituting these values in


2
2 x y sin
sin
sin 2

?/

transformation are,

of

new

orisrin lies

on the curve.

then

hence

(c)

=2

shows

PLANE ANALYTIC GEOMETRY.

100
S

1!L^

If

cos

= tan

= P, then

<f

gp

But

(d) is satisfied.

the

is

whose ordinate is b, Art.


slope of the tangent at the point
is the slope of the new Y -axis
and tan.
hence, the
new Y -axis is a tangent to the parabola at the point whose
ordinate is b .-. at (a, b) .% at the new origin.

57, (5),

<p

COR.

1.

Substituting

= cos =

that cos

and (d) in (3), recollecting


after dropping accents,

(a), (&), (c),

we have,

1,

-^S-*sin* y
% =p

letting

or,

sin

we have

"

cp

= 2p x

(4)

for the equation of the parabola when referred to O


The form of (4) shows that for every value
Fig. 33.

y has two

for x,

OX

Y OX

values, equal but of opposite sign

bisects all chords,

drawn

parallel to

OY

and

line

which

is

assumed
;

hence,

therefore a

diameter of the parabola.

A DIAMETER of a curve
NOTE.
tem of parallel chords.

is

bisects

a sys

\l

To show that the parameter of any diameter

71.

to

the focus
four times the distance from
that diameter cuts the curve.
Draw the focal line FO and the normal

to

Since the triangle


2

O FN

Since
61.

O FT

Hence

is

normal

at

in the triangle

AO = FO

AO N =

<y>

and

sin 2

g>

= FO

NO A,
AO = AN cot v =
FO

2 sin

cp

AN = p,

FO A
2 sin

In the triangle

hence

cos

equal

O N, Fig. 33.
Art.
64, the angle
isosceles;

is

<r.

is

the point in which

<JP

=p

sin

cp

cos

cp

(jp

cos

<JP.

Art.

THE PARABOLA.
.-.

FO =

2 sin 2

q>

i* -4
Sin*
2p = 4 FO

But

101

go

.-.

72.

To find the equation of any diameter in terms of the

and

slope of the tangent

The equation

of

= AO =

y
But from the triangle
b

the semi-parameter.

any diameter as

AO N,

= AX cot =

*L_
tan

is

Fig. 33,

is

we have,

<p

hence

OX

b.

CP

<:

(1)

the required equation.

To show that the tangents drawn at the extremities of


chord
meet in the diameter which bisects that chord.
any
73.

FIG.

Let

(x

/),

P"

(x",

y")

FP";

then

34.

be the extremities of the chord

y"

n (_;*>... (1)

PLANE ANALYTIC GEOMETRY.

102
its

is

The equation

equation.
are

P"

of the tangents at

(V,

y")

0",

^-p(* +
y/ -*(*
Eliminating x from

(2)

and

2)
(

*")

3
>

(3) by subtraction,

we

have,

intersection of the tangents.


for the ordinate of the point of
~~

But

(see (1)

"

is

the reciprocal of the slope of chord

Hence, since the chord

).

are parallel,

we

P"

and the tangent

P",

YT

have,

-y"

Substituting in (4)

it

becomes

-*.value of y with (1) of the preceding


is on the diameter.
the
see that
point of intersection

Comparing
article,

we

this

EXAMPLES.
1.

y
J

=
2.

What must be the value of c


4 x _L_ c may touch the parabola

?/

What

=3x +
3.

in order that the


2

Ans.

is

the line
the parameter of the parabola which
2 touches ?
is

to the parabola y
slope of a tangent
?
the
of
tangent
the equation

The

What

line

=8x?

6x

is

= 3 ^ _L

3.

on a tangent to a parabola required


the equation of the parabc
the equation of the tangent and
,=
the slope of the tangent
- 16 *.
l;
4a
4

The point

(1,

3) lies

4.^

^=

THE PARABOLA.

8 x

is

In the parabola y z
diameter whose equation
5.

what

the parameter of the

is

W=

103

Ans.

136.

6. Show that if two tangents are drawn to the


parabola
from any point of the directrix they will meet at right angles.

From

2
the point
8 x
5) tangents are drawn to ?/
required the equation of the chord joining the points of
Ans. 5 ?/
4^
8
0.
tangency.

7.

8.

What

(2,

the

are

equations of the tangents to

which pass through the point

2,

+ =
= 6x
2

?/

4) ?

Find the equation of the polar of the pole in each of the


following cases
9.

Of (-

3)

1,

with regard to y 2

= 4*.
Ans.

10.

Of

(2, 2)

with regard to if

11.

Of

12.

Given the parabola

(ay b)

with regard to y

?/

=x

Q.

2y

+ 2a; + 4 = 0.

= 4 x.
Ans.

+2=

x.

Ans.
2

2x

3y

2x

by

and the point

2 a
4,

= 0.

10)

to

find the intercepts of the polar of the point.

Ans.
13.

The latus-rectum of a parabola


8x

of the line y

4,

=4

= 0.

= 4:,b=-.

5
required the pole

Ans.

Given

(i,i).

10 x and the tangent 2 y


x
10 required
the equation of the diameter passing through the point of
tangency.
Ans. y
10.
14.

if-

GENERAL EXAMPLES.
1.

Assuming the equation

point on the curve


directrix.

is

of the parabola, prove that every


equally distant from the focus and

PLANE ANALYTIC GEOMETRY.

104

Find the equation of the parabola which contains the

2.

points

(0, 0), (2, 3),

2, 3).

Ans.

What

3.

(-

are the parameters of the parabolas

through the point

3 x2

which pass

(3, 4) ?

Ans.

Find the equation of that tangent


4 = 0.
2 x
parallel to the line y
4.

Ans.

The parameter

5.

of a parabola

is

?/

8y

Y, and

f.

= 9x

which

is

16 a

0.

required the equation

perpendicular to the line


tangent
2 x -\- 2. Give also the equation of the normal which is

which

line

the

of

to

= 4 y.

is

line.
parallel to the given

2
4 x makes an angle of 45 with the
tangent to ?/
X-axis required the point of tangency.
Ans. (1,2).

6.

Show

that tangents

drawn

at the extremities of a focal

chord
7.

Intersect on the directrix.

8.

Meet

9.

That a

focus
10.
11.

is

at right angles.

line joining their point of intersection with the


perpendicular to the focal chord.

Find the equation of the normal

in terms of its slope.

Show that from any point within


may be drawn to the curve.

the parabola three

normals
12

Given the parabola

to construct the tan-

+ cos 6
vectorial angle =
1

60, and to find the


gent at the point whose
line.
initial
angle which the tangent makes with the
60.
Ans. 61

13.

Find the co-ordinates of the

extremity of the latus-rectum being

pole, the
its polar.

normal

at one

THE PARABOLA.

In the parabola y 2
4 x what
chord which the point (2, 1) bisects ?
14.

is

105
the equation of the

Ans.
15.

The polar

of

any

point in a diameter

is

=2x

3.

parallel to the

ordinates of that diameter.


16.

The equation

of a chord of y*

10 x

is

=2x

required the equation of the corresponding diameter.


17.

Show

that a circle described on a focal chord of the

parabola touches the directrix.


18.

The base

of a triangle

gents of the base angles


vertex is a parabola.

b.

=2a

and the sum of the tan

Show

that the locus of the

Required the equation of the chord of the parabola


=2px whose middle point is (m, n}.
n = x
m
A

19.

y-

Ans.

p
20.

angle

chord of the parabola if


2 px makes an
with the X-axis required its length.

focal
cp

Ans.

-?-.

Show

that the focal distance of the point of intersec


to a parabola is a mean
proportional to
the focal radii of the points of tangency.
21.

tion of

two tangents

Show that the angle between two tangents to a parab


one-half the angle between the focal radii of the
points
of tangency.
22.

ola

is

23.

The equation

is y = a

of a diameter of the parabola y 2


2 px
required the equation of the focal chord which this

diameter bisects.
24. The polars of all points on the latus-rectum meet the
axis of the parabola ?/ 2
2 px in the same point required the
co-ordinates of the point.

Ans.

PLANE ANALYTIC GEOMETRY.

106

CHAPTER

VII.

THE ELLIPSE.
74.

THE ellipse

that the

sum

is

the locus of a point so moving in a plane

of its distances from

constant and equal to a given


called the Foci of the ellipse.

two fixed points

line.

If

is

always

The
the

fixed points are


points are on the

the given
given line and equidistant from its extremities, then
line is called the TRANSVERSE or MAJOR Axis of the ellipse.
75.

To deduce

the equation of the ellipse, given

the,

foci

and

the transverse axis.


Y

FIG.

Let F, F! be the

OY

J_ to

AA

foci

at its

co-ordinate axes.

and

35.

AA

the transverse axis.

Draw

OX

as the

middle point, and take OY,

THE ELLIPSE.
Let
also

PD

Let

FF

AA =

V?/

From

the

+ (x

c)

mode

y) are the co-ordinates of P.

20F

201\

hence

and

FPD

Vy +

=2

=2a
Vy + (x - c) + V y +
/

and FjPD, we

have,,

(#-{- c)

we

(a)

have,

+ c) = 2 a
2

(x

(1)

and reducing,

clearing of radicals,

+ x -c
2

(if

and

a2

FP =

c,

of generation of the curve,

r -f

or,

(x,

the right angled triangles

2a,

= /.

From

draw

OY.

to

Then (OD, DP)

FP

Draw PF, PFi

be ary point of the curve.


||

107

x2

a 2 (a 2

c )

...

(2)

As

this equation (2) expresses the relationship between the


co-ordinates of any point on the curve, it must express the

relationship between the co-ordinates of every point


the required equation.

hence

it is

Equation

(2)

elegant form.

may be made, however, to assume


Make x = in (2), we have,

2
?/

a2

for the square of the ordinate of the point in

curve cuts the Y-axis


this distance

by

we

b,
2

.-.

(= OB

which the

/2

).

Eepresenting

2
?

-b* ...

Substituting this value of

or,

have,

=a _
c = a
2

OB

i.e.,

a more

in (2)

(3)

and reducing, we have,

symmetrically,

for the equation


axes.

of ellipse

when

Let the student discuss equation

referred to

(4).

its

centre and

See Art. 12.

PLANE ANALYTIC GEOMETRY.

108
COR.

we make

If

1.

x -f y

which

is

COR.

=a
=a
2

the equation of a

circle.

we interchange a and

If

2.

we have,

in (4),

for the equation of an ellipse


lies along the Y-axis.

COR. 3. If (z y
we have from (4)
,

and

*L

(z",

(^

-z

we

b in (5),

have,

whose transverse axis

y")

- a/

are

(=2 a)

two points on the curve,

and

= -^ (a -

y"*

z"

x ) (a
(a
) (a
ordinates
the
of any two points on the
of
i.e., the squares
other as the rectangles of the segments in
ellipse are to each

hence, y

::

(a

y"

which they divide the transverse


COR.

4.

= x a and y
and dropping accents,
^x 2

tf ,f

_|_

2 atfx

for the equation of the ellipse,


of co-ordinates.

The

line

BB

Fig. 35,

x")

axis.

By making x

after reduction

76.

x"}

in

(4),

we have

(7)

being taken as the origin

is

called

the

CONJUGATE

or

and A
from the figure that
bisects all lines drawn through it and terminating
the point
the CENTRE of
is called
For this reason
in the curve.

MINOR

axis of the ellipse

VERTICES of the

ellipse.

the points

are called the

It is evident

the ellipse.

The
is

~ ** - -?

ratio

called the

this ratio is

ECCENTRICITY
always

<

See

1.

(3)

of the ellipse.
of c

The value

ures the distances of the foci F,

F from
x

Art. 75

...

(1)

It is evident that

= i Va - b
2

the centre.

meas

THE ELLIPSE.
If a

=b

then

in (1),

If b

=1

in (1), then e

when

the ellipse becomes a

zero.
i.e.,

when

the ellipse becomes a

becomes unity.

straight line the eccentricity

77.

i.e.,

becomes

circle its eccentricity

109

To find the values of the focal

radii,

/, of a point on

r,

the ellipse in terms of the abscissa of the point.

The FOCAL RADIUS of a point on the ellipse


of the point from either focus.

From equations

(a),

Art. 75,

= Vy +

we

from the equation of the

ellipse, Art.

^_ ( a 2
a2

have,

cf

(x

the distance

is

x 2\

^2

we

75

(4),

&_ X 2

have,

a2

hence, substituting
*

o
2 ex

/
t/V -

2 ex

=
r

Similarly

we

78.

c-

,.-

See

ex.

(1) Art.

76 ... (1)

find

<Z-

a
hence

-(X

= a + ex

(2)

Having given the transverse

axis

and the

foci of

any

ellipse, the principles of Art. 75 enables us to construct the


ellipse by three different methods.

First Method.

verse axis

AA

Take a cord equal


Attach one end of

in length to the trans


at F, the other at F

it

Place the point of a pencil in the loop formed by the cord


and stretch it upward until taut. Wheeling the pencil around,
while keeping, the point on the paper and tightly pressed

PLANE ANALYTIC GEOMETRY.

110

against the cord, the path described will be an arc of the


After describing the upper half of the ellipse, re
ellipse.
move the pencil and form the loop below the transverse axis.

By

a similar process the lower half

FIG

may

be described.

It is

36.

evident during the operation that the sum of the distances of


the point of the pencil from the foci is constant and equal to
the length of the cord i.e., to the transverse axis.
Take any point C on the transverse axis
Second Method.
;

and measure the distances

CA

A C,

AC.

With F

as a radius describe the arc of a circle

as a centre

also with

and
as a

CA as a radius describe another arc. The points


these arcs intersect are points of the ellipse.
which
in
B/
R,
the radii two other points P, P may be
interchanging
By
A smooth curve traced through a number of
determined.
centre and

points thus located will be the required ellipse.

Let the axes AA = 2


on any straight edge
= OA = a and DL = OB

Third Method.

Lay
given.
will do)

off

KD

MX

a,

BB =

2 b be

(a piece of paper
== b.
Place the

the position indicated in the


straight edge on the axes in
and L slide along the axes, the point
Then as
figure.

THE ELLIPSE.
describe the

will

DKE

For from the

ellipse.

are similar triangles

DK =
--

Ill

DL
lji

-a

_-

i.e.,

_
b

-//

DLH

figure

(x

and

and y being

the co-ordinates of D).

Hence, squaring, clearing of fractions, and transposing, we


have
2
z
a?tf _j_ &2X = a b ^

That is the locus described by D is an ellipse.


ment based upon this principle is commonly used

An
for

instru

drawing

the ellipse.
79.

To find the

The

latus rectum or

parameter of an ellipse.
parameter of an ellipse is the double
or dinate passing through the focus.
The abscissas of the points in which the latus rectum
2
b
-JSubstituting either
pierces the ellipse are x
of these values on the equation of the ellipse
latus rectum, or

we have
Hence

a2

(a

vV

Latus rectum

Forming a proportion from

(a

- b )) =
2

=2y=

a2

9 A2

...

a
(1)

this equation there results,

2y:2b::b:a ;
hence
i.e.,

2y

2 b

::

2 a

the latus rectum is a third proportional to the two axes.

EXAMPLES.
Find the semi-axes, the eccentricity,
of each of the following ellipses

and the latus rectum

1.

2.

3 x 2 -f 2 y 2

= 6.

^,^ = L

3.

4>

x2
4

+ 3 if = 2.
o __
6 =
2
2

3.2

PLANE ANALYTIC GEOMETRY.

112

60y
c

-\-

x-

a.

o.

Write the equation of the


9.

The transverse

V
-^-

-f-

?i.

having given
the distance between the foci

ellipse

= 10

axis

x-

or

Ans.

25
10.

Sum

= 18

of the axes

difference of axes

36

Transverse axis

10

-4^

the conjugate axis

l.

= 6.

Ans.
11.

$-

l-

= 1.

= y2

the

transverse axis.

x*

A
12.

tween

Transverse axis

= 20

conjugate axis

distance be

foci.

Ans.

13.

=1

4y2
25

25

Conjugate axis

= 10

distance between foci

Ans.

-j-

= 50.

= 10.

^+

y*

= 25.

Given 3y 2 + 4 a? 2 = 12 required the co-ordinates of the


point whose ordinate is double its abscissa.
14.

15.

Given the

ellipse 3

2
?/

+ 2 x = 12, and the line y


2

to find the co-ordinates of their points of intersection.

X1
(--=3-1, and the abscissa of a
64
15
.-2

16.

Given the

ellipse

point on the curve

required the focal radii of the point.


8T
Ans. r
7 T%, /

= V

THE ELLIPSE.
80.

113

To deduce the polar equation of the

ellipse, either focus

being taken as the pole.

FIG.

Let us take F as the

From

the figure,

i.e.,

and

pole,

the co-ordinates of any point


r
a
ex
(1) we have,
.

37.

P
.

-\-

or,

reducing,

P FA)

From

(r, (9)

be

Art. 77

(ae

-f-

(1),

we have

r cos 0),

we have
,

*L

_
1

+ e cos

for the polar equation of


being taken as the pole.
From Art. 77 (2),

FP

We

r cos 0.

Substituting this value of x in

=a

(FP

(1)

OD = OF + FD

= ae
r

let

of the ellipse.

...

(2)

the ellipse, the right-hand focus

= / = a + ez

readily determine from this value

cos

for the polar equation of the ellipse, the left-hand focus being
taken as tlje pole.

PLANE ANALYTIC GEOMETRY.

114
COR. If

= a (1
= FA,
=
/ a (1 + = F A.
=a-a
r = a (1 - e
r

0,

0)

e)

= 90,

If

= (1 + = FA
= a (1 - = F A
FM
r = a (1 - e

ft

/
If

= 270,

If

= 360,

To deduce

e)

e)

/
81.

= a (1 - e) = FA,
= a (1 + = F A.
e)

the equation of condition for the supplemental

chords of an ellipse.
Y

FIG. 38.

Let AP,

AP

The equation

be a pair of supplemental chords.


of a line through A (a. o) is
y

= a(l-e )=a-a- -=-^- = - = FN.


= 180,

If

= FM.

=
r

s (a;

r/).

THE ELLIPSE.
The equation

V =

Where

of a line through
s

(x

?/

= ss

a, 6) is

+ a).
we must have

these lines intersect


2

115

a 2)

(x

(1)

In order that the lines shall intersect on the ellipse their


equations must subsist at the same time with the equation of
the ellipse

Dividing

*- 5 (a ^^
(1)

or

(2),

ss

=-

for the required condition.


COR. If a
the ellipse
b,

ss

2)

we have

by

comes

= -

(3)

becomes

-a circle

and

(3)

be

1,

a relationship heretofore deduced. Art. 40 (1).


SCHOL. The preceding discussions have developed a remark
able analogy between the ellipse and circle.
As we proceed
we shall find that the circle is only a particular form of the
ellipse and that all of the equations pertaining to it may be
deduced directly from the corresponding equations deduced
for the ellipse

82.

Let
P"S

by simply making a

To deduce the equation of the tangent


P"

(x",

y"),

cuts the ellipse.

As

= b in those equations.

= i!_(V2

2
2/"

the ellipse.

we must have

the points are on the ellipse,


,

to

(x ?/ ) be the points in which a secant


Its equation is, therefore,

= 4 (a- a-

AA

3/2}

2
*"

...

(3)

PLANE ANALYTIC GEOMETRY.

116

FIG.

39.

These three equations must subsist at the same time


subtracting (3) from (2) and factoring, we have

2/0

(/
r

hence

y
x

+ 2/0 =

P_

becomes

v^alue in (1) it
/

/,2

Eevolving the secant

_|_

line

"

becomes a tangent and x


we have

"-

x",

y"

for the equation of the tangent

(x",

(5)

P"

?/")

and

secant

hence, substituting,

,*;...
1
2

P"

xx
b

P (x y ) will approach
When this occurs the

i.e.,

a2

X)

upward about the point

the other point of intersection


will finally coincide with it.

or

-^-7T7

>-

hence

- i/\
-x

Substituting this

(4)

THE ELLIPSE.
COR.

If b

= a,

117

we have
_i_

yy
n

a*
for the equation of the tangent to the circle.
SCHOL. If we make x and y successively
tion of the tangent (5),

we have y

See Art. 41

and x

values of the variable intercepts

,,

hence

y"

IP

OT
d2
x

..

and x

These values

(6).

equa

= ^ for
x

the

"

y
,

in the

OT, Fig. 39

in the equation
2

2
*"
.

^~"

?/

give, after reduction,

for the equation of the ellipse, the


intercepts of its tangents

on the axes being the variables.


83.

To deduce the value of the sub-tangent.

Making y

.-.sub-tangent

in (5), Art. 82,

= DT = ^1 -

we have

x"

COR. If
.

a*

~x
x

x"

b = a,

"*

ff

then from Art. 41, Schol. a 2

sub-tangent in the circle

2
x"

rr<2

SCHOL. The value of the sub-tangent being


independent of
the value of the minor axis
(2 b) it follows that this value is
the same for every ellipse which is concentric with the
given

ellipse,

and whose common transverse axis

is

a.

PLANE ANALYTIC GEOMETRY.

118

of condition that a line shall pass through


the centre of the ellipse and the point of tangency is, Fig. 39,

84.

The equation

tx",

y"

.-.

the slope of this line

t==

The

is

^7

slope of the tangent at

a2

(x",

y")

is,

Art. 82,

y"

Multiplying,

member by member, we have

But

(3)

Art. 81

...

i.e.,

ss

the tangent to the ellipse

and

the line joining the centre

and

the point of tangency enjoy the property of being supplemental


I)

chords of an ellipse whose semi-axes bear

is

COR. If

parallel

to

then s = t
a diameter of the

t,

parallel to the tangent


diameter.

chord

85.

is

to

each other the ratio

-.

chord
if one supplementary
ellipse, the other supplementary
i.e.,

drawn

at the extremity of that

84 afford us two different


principles of Arts. 83,
a given
of constructing a tangent to the ellipse at

The

methods
point.

First Method.

Art. 83, Schol. Let


draw the ordinate

P",

Through
point.
until it meets the circle described
P"

of the ellipse
Join
at F.

(AA )

P"

and

in

F; draw
P"T

Fig. 40, be the given

P"D

and produce

it

upon the transverse axis


tangent to the circle

FT

will be the required tangent.

THE ELLIPSE.

FIG.

P"

||

86.

from

RA

40.

Draw

Art. 84 and Cor.

Second Method.
centre, and
to

draw

AR

119

||

to

P"R

will be tangent to the ellipse at

To deduce the equation of the normal

The equation

of

any

line

through

P"R

P"T

P".

to the ellipse.

P"

(x",

through the

drawn through

y"),

Fig. 39, is

S (X
y
(1).
In order that this line and the tangent at
shall
be perpendicular their slopes must satisfy the condition
?,"

X")

P"

We

+ S8 = o

(x" ,

y")

(2).

have found Art. 82 for the slope of the tangent

/==

"5-^
hence, the slope of the normal

Substituting this value of

for the equation of the

is

s in (1),

normal

to

we have

the ellipse.

PLANE ANALYTIC GEOMETRY.

120
COR.

1.

If

then

b,

"

yx

which

xy"

becomes, after reduction,


0,

normal

line to the circle.

To deduce the value of the sub-normal

Making y

...

is the equation of the

87.

we

(3)

in the equation of the normal, (3), Art. 86,

ON = x

have, Fig. 39,

*x",

Sub-normal
COR.

If

1.

b,

then

Sub-normal for the

circle

x".

EXAMPLES.
Deduce the polar equation of the ellipse, the pole being
with the X-axis.
the centre and the initial line coincident

1.

at

A
Ans.

-.
2

sin

+b

cos 2

the

of
following
Write the equation of the tangent to each
of the sub-tangent in each case.
value
the
and
give
ellipses,

2z

2.

El

3.

+ 4y = 38at
2

+ 2.

(1, 3).

Ans.

!.

+ 6 y = 19

18.

at (1, ordinate positive).

1,

An,.

4.

._l,

at (2,0).
0.

^ + 3^- 11

J/l

_
o

1,

at

(2,-

at (0,

1)

V)

4as

_ 3y . 11;i

THE ELLIPSE.

8.

9.

+ te - 2,
2

m +

2
?/

- V2 -

at (1,

at (afo

1,

+,

121

6).

.5).

Write the equation of the normal to each of the following


the sub-normal.
ellipses, and give the value of
7/

+ 4 x = 39,

at

if

+2

= 44,

at

10.

11.

12.

^!_

13.

-+{
o
o

14.

^1

_|_

4-

ic

7/2

2
?/

-f- ?

1,

at (1,2).

at

^ ord

=mn
2

1,

2
,

ord negative).

ord

at

1?

ic

2,

1.

a,

15.

(3, 1).

).

_|_^

at (m, o).

2x
6
of a chord of an ellipse is y
the equation of the supplementary chord, the axes of
the ellipse being 6 and 4 ?
Ans. y
f
f x
16.

what

The equation

17.

is

Given the equation

j-

MID

= 1,

and

?/

re-

quired the equation of the tangents to the ellipse at the points


in which the line cuts tne curve.
18.

Given the ellipse

= 0;
(a)

= 1,

and the

line

required

The equation
"

(b)

(-

"

of a tangent to the ellipse

||

"1

to the line.
"

"

"

-{-

PLANE ANALYTIC GEOMETRY.

122
19.

The point

(4, 3) is

1G

+ 1P

outside the ellipse

1;

required the equations of the tangents to the ellipse which


pass through the point.
88. The angle fanned by the focal lines drawn to any point
of an ellipse is bisected by the normal at that point.

FIG.

41.

Let P"N be a normal at any point

We

have found, Art.

From

Art. 76

P"
(a;",

?/ ).

= OF = ae;
NF = OF OX = ae NF = OF + OX = ae +
NF NF
r

FP"

FP"

.-.

::

NF NF

::

P"F

hence

e?x"

But

P"F,

87, that

we have OF

.-.

Draw

(a

ex")

::

e*x"

(a

ex")

(a

FP"

ex")

= e (a - ex
= e (a +
+

ex")

ex")

Art. 77, (1)

P".

ff

and

(2)

THE ELLIPSE.
The normal,

P"F

into

cent sides.

therefore,

divides the base of the

triangle

two segments which are proportional to the adja

Hence
==

FP"jST

SCHOL.

123

1.

If

P"T

P"N.

be a tangent drawn at

P"C

FP"T

P",

we must have

between a
FP"N
and
the
the
angle
Hence,
right angle
(= FP"N).
tangent to the ellipse makes equal angles with the focal radii
for each of these angles is equal to the difference

drawn

to

SCHOL.

method

the point of tangency.


2.

The

principles of this article afford us another

drawing a tangent to the ellipse at a given point.


be a point at which we wish to draw a tangent.
Let
Pro
to R, making P"R = FP" join F and R.
duce F
A line
J_ to FR will be tangent to the ellipse
P"T, drawn through
of

P"

P"

P"

at

P".

89. To find the condition that the straight line y


must fulfil in order that it may touch the ellipse

4 + y. =
a

If

= SX
O

a2

we obtain the

-f-

i.

we consider the line


y

= sx

as a secant

and combine the equations

-\- C,

lr

co-ordinates of the points of intersection.

Eliminating y from these equations, we have

sa 2 c

ab

for the abscissas of the points of intersection. Now, when the


secant line becomes a tangent, these abscissas become equal.

Looking

at (1)

we

see that the condition for equality of ab-

PLANE ANALYTIC GEOMETRY.

124

numerator shall disappear

scissas is that the radical in the

hence

sV

or
is

s*a

-f-

=c

0,

2
.

(2)

the required condition.

COR. If we substitute the value of


equation of the line, we have

= SX -.

V*

+&

a2

for the equation of the tangent

drawn from

...

(2) in the

(3)

the ellipse in terms of

to

its

slope.

To find the

90.

gent

to

a tan
a
from focus as

locus generated by the intersection of

and a perpendicular

the ellipse

to it

the point of tangency moves around the curve.


The equation of a straight line through the focus (ae, o)

is

s (x
ae).
y
In order that this line shall be perpendicular to the tangent

its

SX
y
be
must
equation

J-

V* 2

= - i (x -

+b

ae)

(1),

(2)

we now combine (1) and (2) so as to eliminate the slope


the
resulting equation will express the relationship be
(s),
tween the co-ordinates of the point of intersection of these
hence it will be the
lines in every position they may assume
If

equation of the required locus.


Transposing sx to the first member in (1),
of fractions

and clearing

(2)

and transposing, we have


y
sy

= Vs a + b
x = ae.

SX
-}-

-t-

2
.

Squaring these equations and adding, remembering that


2
a z e 2 Art. 76, we have,
ab

or

THE ELLIPSE.

125

hence, the circle constructed on the transverse axis of the ellipse

and

the locus of the intersection of the tangents


let fall from the focus on them.

is

the perpen

diculars

This circle

known

is

(See Fig. 45.

ellipse.

as the Major-Director circle of the

To find the locus generated by the intersection of two


tangents which are perpendicular to each other as the points of
tangency move around the curve.
91.

The equation

of a tangent to the ellipse

= SX +

The equation

a2

+b

is

2
.

(1)

of a tangent perpendicular to (1)

is

hence, by a course of reasoning analogous to that of the pre

ceding

article,

we have
xz

+ ?/ = a

2
-f b

(3)

The required locus is, therefore, a circle


2
ellipse and having its radius equal to Va
92.
out

concentric with the


-f-

2
.

Two tangents are drawn to the ellipse from a point with


required the equation of the line joining the points of

tangency.

Let

?/), Fig. 42, be the given point, and let


be
the points of tangency.
Since
2/2)
(x y ) is a
point common to both tangents, its co-ordinates must satisfy

?2

(a:

P"

(*2>

their equations

(x" ,

hence,
x"

a2

x xz
a2

Hence

(x",

y")

and

~W_
y yz

(a; 2

y z ) will satisfy the equation

y"),

PLANE ANALYTIC GEOMETRY.

126

As (1) is the equation of a straight line, and is satisfied for


the co-ordinates of both points of
tangency, it must be the
equation of the straight line which joins them.
93.

regard

To find the equation of the polar of the pole (V, /), with
to

the ellipse
2
~

1--1

FIG.

42.

the aid of Fig. 42, and a course of reasoning similar to


PiP", the polar to P
may be
shown to be

By

that of Art. 49, the equation of

xx
a2
COR.
PI

(^i>

,y_y_

1.

b*

If the polar of the point P (x f ?/) passes


yi)j then the polar of P l (x^ T/J) will pass

(V, /).

through
through

(See Art. 50.)

94. To deduce the equation of the ellipse when referred to a


pair of conjugate diameters as axes.
A pair of conjugate diameters of the ellipse are those diam-

THE ELLIPSE.

127

which if the ellipse be referred its equation will contain


the
second powers of the variables.
only
eters to

The equation
axes

of the ellipse

when

referred to its centre and

is

we

refer the ellipse to a pair of oblique axes


at
the centre, we have, Art. 33, Cor. 1,
origin
If

x
y

=x
=x

having the

+ i/ cos
+ y sin y

cos

(p

sin

for the equations of transformation.

Substituting in

(1),

we

have

2
2
(a sin
2
2 (a sin

+ W cos 0) x + (a sin + tf cos ij


sin
+ 6 cos cos a y = aW ... (2)
2

q>

q>)

(?)

q>

for the equation of the ellipse referred to oblique axes.


But,
by definition, the equation of the ellipse when referred to a

pair of conjugate diameters contains only the second powers


of the variables
hence
;

a- sin

sin

cp

-f b cos

cos y

the condition that a pair of axes must


conjugate diameters of the ellipse.
is

Making the

co-efficient of

(3)

fulfil in

x y equal to zero in

order to be

(2),

we have

after dropping accents


2
2
(a sin

+6

cos 2 0) x 2 -f (a 2 sin 2

<p

+6

cos 2
<p)

=a6
2

... (4)

for the equation of the ellipse when referred to a


pair of con
jugate diameters. This equation, however, takes a simpler
form when we introduce the semi-conjugate diameters. Mak

ing y

and x

= 0,

successively, in (4),

a2

sin 2

(p

-M

cos 2

+6

cos 2

<p

we have

PLANE ANALYTIC GEOMETRY.

128
r

which a and
have
(5). we

in

ft

From

represent the semi-conjugate axes.

a? sin

|72

2
ft

cos 2

= ^L
_

^2

2
ft

in
Substituting these values of the co-efficients

(4),

we

have,

after reduction,

for the required equation.

COR. As equation (6) contains only the second powers of


the variables, it follows that each of the two diameters to
which the curve is referred will bisect all chords drawn
parallel to the other.

SCHOL. The equation of condition for conjugate diameters


be put under the forms
(3) may

= --7 2

tan 6 tan

(7)

<p

we

see that the

81,
Comparing this expression with (3)
same result was obtained for the supplementary chords of an

Art.

if
ellipse; hence, Fig. 40,

ary chords, then

RP",

A R R A be

PR",

a pair of supplement

drawn through the centre

parallel

Again
to these chords, will be a pair of conjugate diameters.
we
see that the same relation
Art.
with
84,
(1)
comparing (7)
for a diameter and the tangent drawn at
ship was obtained
diameter and P"T
its
hence, Fig. 40, if P"R be a
:

extremity
be a tangent drawn at its extremity, then PR", drawn through
to RP".
the centre parallel to P"T, is the conjugate diameter
a
The equation of condition (7) being single equation con
;

unknown quantities (tan. 0, tan.


assume any value we please for one of them, and
will make known the value of the other hence,

taining two

there are

an

infinite

<r),

we may

the equation
in the ellipse

number of pairs of conjugate diameters.

THE ELLIPSE.

129

To find the equation of a conjugate diameter.

95.

Let

P"R,

RP

be a pair of conjugate diameters.

to find the equation of

R/P

We

wish

FIG.

The equation
P"
y"}

(*",

of the

wr"

By

tangent line

in/
yy

drawn through

_i

P R/

Art. 94, Schol., the diameter

equation must be the same

its

P"T,

is

xx
a2

hence

43.

is parallel to P"T
as that of the tangent,

the constant term being zero.

xx
.

(1)

or

...

is

the equation of a diameter expressed in terms of the co

(2)

ordinates of the extremity of its conjugate diameter.


COR. Let s represent the slope of the diameter P"R

from

(2)
_

bV
2

?/

jr

1
s

then,

PLANE ANALYTIC GEOMETRY.

130

^=^=

since

i
;

DP"
y"

hence we have

-~x
a

y=

(3)

for the equation of a diameter in terms of the


slope of its

conjugate diameter.

To find the co-ordinates of either extremity of a diam

96.
eter,

the co-ordinates of one extremity of

its

conjugate diameter

being given.

Let
Let

P"R

(x",

RT

and

y")

Fig. 43, be a pair of conjugate diameters.


be the co-ordinates of
We wish to find the
,

P".

terms of the co-ordinates of


The equation of condition that P (a/, y ) shall be on the
diameter P R is, Art. 95, (1)
co-ordinates (x

Since

(x

of

P."

X"

y ) is on the
V2
o/2

Eliminating y and

we

in

a;

ellipse,

we have

also

successively, from these equations,

find

These expressions, taken with the upper signs, are the co


ordinates of P
taken with the lower signs, they are the
;

co-ordinates of
97.

To show that the sum of the squares on any pair of

semi-conjugate diameters

on the semi-axes.
Let
P"

(x",

y"}

and

is

equivalent
f

(x

).

to the

sum of the squares

Fig. 43, be the extremities

THE ELLIPSE.

131

two semi-conjugate diameters. Let OP" = a


the triangles ODP", OD P we have,
from
then,
of any

OP =

r
;

...

"

and

But, Art. 96,

cc

=x +y

(1)

(2)

=~

and

hence

Adding

(1)

and

i/

2
x"

98.

hence,

_|_

P"R

The area

= 2a

),

is

+ -TT

//

==

-f

ft

(4)

PR

2ft

of the parallelogram

From

the figure
sin (180

),

to

the rectangle constructed

Fig. 44, be

OP"TR

is

OP"TR

0) )

= afb

OP"

=a

sin

sin
(<p

P"OR

sin
(>

(9)

<9)

any two conju

= area BB H H.

P"P.

P"P

(<p

area of

equivalent

To prove that area CTC T

OR X

.-.

To show that the parallelogram constructed on any two

gate diameters.

= a!

2_ = l

-f 6

conjugate diameters
on the axes.

Let

(3}

we have

(3),

70X

but

a2

(1)

PLANE ANALYTIC GEOMETRY.

132

c
Fm.

From

the triangles

OD P

sn

Hence

sin

44.

0DP",

we have
.y"

OF

(qp

Mb
ab

a fb

0)

sin

go

cos 6

cos

g>

sin

THE ELLIPSE.

133

Substituting this value in (1) and multiplying through by


4,

we have

area

OP"TB

area

CTC

X 4 = 4 ab
= area BB H H.
;

i.e.,

T"

To show that the ordinate of any point on

99.

the ellipse is

the ordinate of the corresponding point on the


circumscribing
circle as the semi-conjugate axis of the
ellipse is to the semitransverse axis.
to

Let

(*

DP

DP"

/) and

Since

Since

P"

be the ordinates of the


corresponding points

P"

(x",

(x

(x" ,

) is

y"}

y"}.

on the

is

ellipse,

on the

circle

we have

whose radius

is a,

we have

Dividing these equations, member by member, we have


/

2
,

(since

oj

a;"

PLANE ANALYTIC GEOMETRY.

134
Similarly

we may prove
x1

x.2

::

that
b,

any point on the ellipse, and x 2


the corresponding abscissa of a point on the inscribed circle.

where

Xi is the abscissa of

is

The

100.

method

principles of the preceding article give us a


of describing the ellipse by points when the axes are

given.

From

0, Fig. 45, as a centre with radii equal to the semiDraw any


OB describe the circles
RA, BCB

axes OA,
radius

draw

OB

MN

||

OA

Since MJST
a point of the ellipse.
have
we
the
of
O,
triangle
is

BD
D N D R OM OR
::

i.

e.,

of the larger circle, cutting the smaller circle in


in
to
cutting the or din ate let fall from

b
y"

is

parallel to the base

hence, the construction.

101.

To show that the area of the

the circumscribing circle as the


to its

semi-major

ellipse is to the

semi-minor axis of the

axis.

FIG.

46.

area of

ellipse is

THE ELLIPSE.
Inscribe in the ellipse any polygon
its vertices draw the ordinates

from

135

AEE E E E A
2

ED^D^

/
,

and

producing
etc.
Joining

them upward to meet the circle in P, P 1? P 2;


these points we form the inscribed
polygon

etc.,

APP^P^A

in

the circle.

Let (x, y ), (x yj,


y 2 ) etc., be the co-ordinates of
r
P, P 15 P 2 etc., and let (a;, y), (x , /),
be the co-ordi
y"), etc.,
nates of the corresponding points E,
1}
2 etc., of the ellipse.
,

(x",

(aj",

R R

Then
Area
Area RPD.R,
Area

hence

But, Art. 99,

Hence

AreaRDD^
Area

We

may prove in like manner that every corresponding pair


of trapezoids bear to each other this constant
ratio; hence,
by the Theory of Proportion, the sum of all the trapezoids in
the ellipse will bear to the sum of all the
trapezoids in the
circle the same ratio.
these sums
2,t and
Representing
2T, respectively, we have
2*

2T
As

this

by

relationship holds true for any

zoids, it holds true for the limits to

number

of trape

which the sum of the

trapezoids of the ellipse and the sum of the trapezoids of


the circle approach as the number of
trapezoids increase.

PLANE ANALYTIC GEOMETRY.

136

But these
the circle

limits are the area of the ellipse

area of ellipse
area of circle

Since the area of the circle

COR.

and the area of

hence

area of ellipse
TT

Since

a2

a2

TT

a 2 we have
,

area of ellipse
TT ab
TT ab

..
TT

is

=
:

TT

ab.

TT b*,

we

see that the area of the ellipse


tween the areas of the circumscribed

a mean proportional be

is

and

inscribed circles.

EXAMPLES.
1.

What must

be the value of

in order that the line

y = 2 x + c may touch the ellipse


! + JL == i ?
9
4
2

Ans.

= 5.

of an ellipse is 10 what must be


the value of the semi-conjugate axis in order that the ellipse
2.

may

The semi-transverse

touch the line 2 y

-\-

14

= 0?

Ans.
3.

What

~,2
x

_i_

whose inclination

drawn

,,,2
y

to X-axis

locus of

-4
a2

ellipse

= 45

the intersection of the tangents to the

=l
b

at the extremities of conjugate diameters

required

its

V24.

The

are the equations of the tangents to the ellipse

"*"

4.

is

an ellipse

equation.

Ans

=2.

THE ELLIPSE.
5.

Tangents are drawn from the point


a

frequired the

2
2/

137
(0,

8) to the ellipse

= i;
the

equation of

joining the points


Ans. 8 y
1

line

of

tangency.

Eequired the polar of the point


following ellipses
6.

*2

9.

What

with respect to the

= 9.

3/ 2

(5, 6)

0.

=1.

7.

are the polars of the foci ?

Ans.

-e

10.

What

is

the pole of y

= 3x

-f-

1 with respect to

(-

11.

The

line 3

y
T2

_^L

5x
2

_i_

7/
iL

is

_i

required the equation of the conjugate diameter.


Ans. 20 y
12.

12, 9).

a diameter of

+ 27 # = 0.

pair of conjugate diameters in the ellipse


?/ 2

T 2

_-f JL.^1

16

make

angles whose tangents are

with the X-axis


13.

What

is

^ and

^,

respectively,

required their lengths.

the area of the ellipse

?4

= 1?

10
.

TT

VlO.

PLANE ANALYTIC GEOMETRY.

138

The minor

14.

axis of an ellipse

to the area of a circle

and its area is equal


what is the length

is 10,

whose diameter

16

is

25.

Ans.

of the major axis ?


15. The minor axis of an ellipse is 6,
focal radii to a point on the curve is 16

axis, the distance between the

foci,

and the sum of the


required the major

and the

area.

GENERAL EXAMPLES.

What

1.

(2, 4)
2.

(6.

The major

4) is

3.

The

lines

18, and the point


the
equation of the ellipse.
required

axis of

on the curve

tal chords

an

ellipse is

13

+ 6 and y = ~ x -f ^ are supplemen

drawn from the extremities of the transverse

of an ellipse
4.

the equation of the ellipse which passes through


the centre being at the origin ?

is

2, 4),

The minor

axis of an ellipse

is

12,

and the

centre divide the major axis into four equal parts


the equation of the ellipse.
5.

sum
is

in

Assuming the equation


of the distances of

constant and
6.

of the ellipse

any point on the


for a point

an ellipse whose eccentricity

7.

What

foci

and

required

show that the

ellipse

from the

foci

to the transverse axis.

The sub-tangent

of the ellipse.

axis

required the equation of the ellipse.

is

whose abscissa
;

is

is

=6

required the equation


AnSf

^L

+ ]_ =

l.

are the equations of the tangents to

which form with the X-axis an equilateral triangle

that the tangents drawn at the extremities of any


chord intersect on the diameter which bisects that chord.
8.

Show

THE ELLIPSE.

139

9. What are the


equations of the tangents
extremities of the latus-rectum ?

drawn

at the

10. Show that the pair of diameters drawn


parallel to the
chords joining the extremities of the axes are
equal and
conjugate.

A chord

11.

of the ellipse
H

16

"~9~

passes through the point (2, 3) and is bisected by the line


x
required the equation of the chord.

What

12.

are the equations of the


pair of conjugate
2
9x 2
144 which are equal ?
?/

eters of the ellipse 16

diam

Show

that either focus of an


ellipse divides the major
two segments whose
rectangle is equal (a) to the
rectangle of the semi-major axis and semi-parameter
to
13.

axis in

the square of the semi-minor axis.


14.

Show

(&)

that the rectangle of the


perpendiculars let fall
is constant and
equal to the square

from the foci on a tangent


of the semi-minor axis.
15.

A system

of parallel chords which make an


angle whose
2 with the X-axis are bisected
tangent
by the diameter of
an ellipse whose semi-axes are 4 and
3; required the equation
of the diameter.

16. Show that the


polar of a point
parallel to the conjugate diameter.
17.

on any diameter

is

Find the locus of the vertex of a


triangle having given
= 2 a, and the
product of the tangent of the angles

at the base

^2

Ans.

PLANE ANALYTIC GEOMETRY.

140
18.

Find the locus of the vertex of a triangle having given


= 2 a, and the sum of the sides = 2 b.

the base

Ans
19.

Find the locus of the intersection of the ordinate of

the ellipse produced with the perpendicular let fall from the
centre on the tangent drawn at the point in which the ordi

nate cuts the ellipse.


20. Find the locus generated by the intersection of two
tangents drawn at the extremities of two radii vectores (drawn
from centre) which are perpendicular to each other.
a2
Ans. a^

bW =

21.

line of fixed length so

remain in the co-ordinate axes

by any point of the

moves that

its

extremities

required the locus generated

line.

The angle AOP"


y (Fig. 45) is called the eccentric
Show that (a; ?/)
P
the
of
(# tf) on the ellipse.
angle
point
and from these values of the co-ordinates
(a cos cp, b sin
22.

<p)

deduce the equation of the


23.

ellipse.

Express the equation of the tangent at

(a;",

y")

in terms

of the eccentric angle of the point.

Ans.

- cos

a
24.

If

(a/, ?/),

(x",

?/ )

4-

y~

sin

<p

1.

are the ends of a pair of conjugate


and qo show that
cp

diameters whose eccentric angles are

90.

q>

THE HYPERBOLA.

CHAPTER

141

VIII.

THE HYPERBOLA.
THE hyperbola is the locus of a point so moving in a
the difference of its distance from two fixed points
that
plane
The fixed
is always constant and equal to a given line.
102.

points are called the Foci of the hyperbola. If the points


are on the given line produced and equidistant from its
extremities, then the given line is called the TRANSVERSE Axis
of the hyperbola.

103.

and

To deduce the equation of the hyperbola, given the foci

the transverse axis.

FIG.

Let F,

be the

OY 1 to AA

at its

foci,

and

47.

AA

the transverse axis.

Draw

OX

as the

middle point, and take OY,

PLANE ANALYTIC GEOMETRY.

142

PF

draw

Let

co-ordinate axes.

PF.

also

PD

||

OY.

= (x, y) are
=
AA 2 a, FF = 2 OF

Then (OD, DP)

the co-ordinates of P.

Let

== 2

OF =

the right angled triangles

FPD

=r

Draw

be any point of the curve.


to

c,

FP =

and F P

r
.

From
r

= Vy +
2

From

the

and /

(x

mode

~c)

= Vy +

and

(x

of generating the curve,

F PD, we

+ c)

have

2
.

(a)

we have

= 2 a.

Hence, substituting,

Vy +

+ c) 2

or,

(x

V?/

(x

clearing of radicals and reducing,

_a
(c*

- a if = a

x2

(c

c)

= 2a;

(1)

we have

a 2 ) ...

(2)

for the required equation.


This equation, like that of the
ellipse (see Art. 75), may be put in a simpler form.

Let

This value in

or,

=b

a2

2
.

(3)

(2) gives, after

changing signs,

symmetrically,

4a~

b~

= l...(5)

for the equation of the hyperbola


and axes.

Let
Cor.

1.

the

student

If b

discuss

a in

x2

(5),

if

this

when

referred to

its

centre

(See Art. 14)

equation.

we have
a2

(6)

The curve represented by this equation is called the ^Equi


lateral Hyperbola.
Comparing equation (6) with the equation
of the circle

we
the

see that the equilateral hyperbola bears the

common hyperbola

same

that the circle bears to the

relation to

ellipse.

THE HYPERBOLA.
COR.

If (V,

2.

143

are the co-ordinates of two


y ) and
we have from (4)
(x" ,

y")

points on the curve,

hence

= -*! (x -

2
:
?/"

a 2) and if 2

::

a) (x

(x

+ a)

-*i

a2

(35"*

a)

(x"

^)

(x"

-f a)

the squares of the ordinates of any two


points on the
hyperbola are to each other as the rectangles of the segments in
which they divide the transverse axis.
i.e.,

COR.

= x a and y
and dropping accents,

By making x

3.

after reducing

V-

x2

+ 2 ab x =
2

for the equation of the hyperbola,

104.

From equation
b =

4-

...

(3) Art. 103,

=y

in

(4)

we have

(7)

being taken as origin.

we have

vr~

this distance off above

and below the origin on the


the points B, B Fig. 47, Art. 103.
The line
BB i called the CONJUGATE Axis of the hyperbola. The
points A and A are called the VERTICES of the curve.
The
Laying

Y-axis,

we have

bisects all lines drawn through it and


point
terminating in
the curve
for this reason it is called the CENTRE of the
;

hyperbola.

The

ratio

Vet 2

-I-

62
-

1
a

>

e.

See

(3) Art.

103 ... (r

called the ECCENTRICITY of the


This ratio is
hyperbola.
2
1.
The value of c
b 2 measures the
evidently
-t Va
distance of the foci F,
from the centre.
is

>

If b

=a

in (1),

we have

= V2

for the eccentricity of the

equilateral hyperbola.

105. To find the values of the focal radii, r, r


of a point
on the hyperbola in terms of the abscissa
the
of
point.
From equations (a) Art. 103, we have

PLANE ANALYTIC GEOMETRY.

144

From

the equation of the hyperbola, (4) Art. 103,

we have

Hence, substituting
r

x2

\/

V a2

+x

a2

=
hence

Similarly,

we

106.

Tio

and

= ex

2
,

Art. 104 (1),

a ...

= ex + a

(1)

(2)

construct the hyperbola having given the transverse

the foci of the curve.

FIG.

First Method.
foci.

+c

find

axis

2 ex

48.

Let A A be the transverse axis and F, F the


ruler whose length is L and attach

Take a straight-edge

THE HYPERBOLA.

145

its ends at F so that the ruler can freely revolve about


Cut a piece of cord so that its length shall be
that point.
L 2 a, and attach one end to the free end of the ruler,

one of

and the other end to the focus F.

Place the ruler in the

position indicated by the full lines, Fig. 48, and place the
Stretch
point of a pencil in the loop formed by the cord.
the cord, keeping the point of the pencil against the edge of

the ruler.

If

we now

revolve the ruler upward about

the

the pencil, kept firmly pressed against the ruler,


point
will describe the arc AP of the hyperbola.
By fixing the
end of the ruler at F, we may describe an arc of the other
of

It is evident in this process that the difference of


the distances of the point of the pencil from the foci F ,F,
is always equal to 2 a.

branch.

Take any point D on the transverse axis.


Second Method.
Measure the distances A D, AD. With F as a centre and A D
with F as a centre and
as a radius describe the arc of a circle
;

AD

as a radius describe another

arc.

The

intersection

of

these arcs will determine two points, P 1? P 2 of the curve. By


interchanging centres and radii we may locate the points E, 1?
,

In this manner we

n the other branch.


>

many

may determine as
may require.

points as the accuracy of the construction

107. To find the lotus-rectum or parameter of the hyperbola.


The LATUS-RECTUM, or PARAMETER of the hyperbola, is the
double ordinate passing through either focus.

Making x

we have

-j-

V&

.-.

in the

2y

Forming a proportion from

2y:2b
i.e,

equation of the hyperbola

a
this equation,

we have

::b:a-,

the latus-rectum of the hyperbola is a third proportional to

the axes.

PLANE ANALYTIC GEOMETRY.

146
108.

The equation of the

and axes

is

a 2y 2

The equation
and axes

-+-

x2

ellipse

=ab
2

of the hyperbola

in the sign of b 2
2

its

centre

its

centre

when

referred to

is

expressions

referred to

Comparing these equations, we


is

when

If,

see that the only difference

therefore, in the various analytical


for the ellipse, we substitute

we have deduced

+ V

for & 2 or,

what is the same thing,


I
1 for b, we
will obtain the corresponding
analytical expressions for the
b

hyperbola.

To deduce the equation of the conjugate hyperbola.


hyperbolas are CONJUGATE when the transverse and con
jugate axes of one are respectively the conjugate and trans
verse axes of the other.
109.

Two

FIG.

Thus

49.

AA

be the transverse axis of the hyper


in Fig. 49, if
for its conjugate axis, then the hyperbola

bola which has

which has

BB

BB

for its transverse axis

and

AA

for its conjugate

THE HYPERBOLA.
axis

its

is

147

conjugate; and, conversely, the hyperbola whose


is BB and conjugate axis is AA has for its

transverse axis

conjugate the hyperbola whose transverse axis

whose conjugate axis is BB


We have deduced, Art. 103,

is

AA

and

*a1

=
b

(5),

(1)

for the equation of the hyperbola whose transverse axis lies


wish to find the equation of its conju
along the X-axis.

We

has

from the figure that the hyperbola which


for its conjugate axis
for its transverse axis and

It is obvious

gate.

BB

AA

bears the same relation to the Y-axis as the hyperbola whose


and conjugate axis is BB bears to the
transverse axis is

AA

X-axis

hence, changing a to I and b to

a,

x to y and y to x

we have

in (1),

-...

for the equation of the conjugate hyperbola to the hyperbola

whose equation is (1).


Comparing (1) and (2) we see that the equation of any
hyperbola and that of its conjugate differ only in the sign of
the constant term.

COR.

Since

V^

+ u2 = V

any hyperbola and those of

The

its

+b

2
,

the

focal

distances

eccentricities of conjugate

hyperbolas, however, are

For the hyperbola whose semi-transverse axis


a and semi-conjugate axis is b, we have

not equal.

Art. 104, (1) e

For

its

Vj

of

conjugate are equal.

conjugate hyperbola, we have

is

PLANE ANALYTIC GEOMETRY.

148

EXAMPLES.
and

Find the semi-axes, the eccentricity


of each of the following hyperbolas

the- latus-rectum

1.

2.

^1

2,2

4 X2

-^!

4
2

3.

?/

36

5.

3 y2

6.

a?/

1.

cc

bx*

12.

=-

aft.

16 x 2

= - 16.

7.

4 z2

4.

=_

16 y z

= - 64.

8.

- if = m.
-

ma?

TO.

Write the equation of the hyperbola having given

foci

axis

= 12

axis

= 10

The transverse

9.

the distance between the

= 16.

10.

The transverse

parameter

8.

Ans.

25
11.

Semi-conjugate axis

=6

The equation

-L
36
64
X2

13. The conjugate axis


double the conjugate.

is

10,

x2

The transverse

axis

is

8,

.<

1.

3 if

3 yz

3?
-

Io

and the conjugate axis

distance between foci.

16

6.

+ 6 = 0.

and the transverse axis


A
Ans

14.

?/

2
of the conjugate hyperbola, x

Ans.

l.

= 10.

the focal distance

Ans.

12.

-=
20

-fID

is

THE HYPERBOLA.
15.

149

Given the hyperbola


2

.2

10

,,2

jr

_i

required the co-ordinates of the point whose abscissa


its

is

double

ordinate.

l
16.

Vf Vj

Write the equation of the conjugate hyperbola

to each

of the hyperbolas given in the first eight examples above.


2
4 x* =
Given the hyperbola 9 y
focal radii of the point whose ordinate

36

17.

is

=1

required the

and abscissa

positive.
18.

Determine the points of intersection of

49 =
^~

=
+ $16

and

1.

16

1.

110. To deduce the polar equation of the hyperbola, either


focus being taken as the pole.

F as the pole, Fig. 47.


PFD) = (r, 0) be the co-ordinates

Let us take
Let (FP,

on the curve.

From

From

r = ex - a
=
OD OF + FD
r cos
x = ae

Art. 105, (1),

FP =

i.e.,

Fig. 47,

of

any point

we have
.

(1)

0.

-f-

and reducing, we have

Substituting this value in (1)

(I

-^
e

...

cos

(2)

for the polar equation of the hyperbola, the right


being taken as the pole.

Similarly from Art. 105,

(2),

a-*
/ = *1
e cos

hand focus

we have

for the polar equation, the left

(3 )

hand focus being the

pole.

PLANE ANALYTIC GEOMETRY.

150

If

COR.

0,

/
90,

=
=

If

If

ae2

-f-

= a ae = FA
= a + ae = F A.

semi-latus rectum.

= semi-latus rectum.
=
=
a
a
r = - a + ae = FA,
T = a -ae= - F A
b
= semi-latus rectum.
a 4- ae
r =

ae 2

= ISO

If $

270

ae

= semi-latus rectum.

=
a

111. To deduce the equation of condition for the supple


mentary chords of the hyperbola.
By a method similar to that of Art. 81, or by placing b~
2
for b in (3) of that article, we have
=

aa
hence, the product of the slopes of any pair of supplementary
chords of an hyperbola is the same for every pair.
If

COR.

we have

b,

1, or, s

ss

1
,

S
.

tan

sum of

= cot

the two acute angles which

any pair of sup


make ivith the
plementary chords of an equilateral hyperbola
hence, the

X-axis

is

equal

to

90.

To deduce the equation of the tangent

112.

to

the hyperbola.

adopted in the
entirely analogous
or substituting
57
Arts.
41, 82,
circle, or ellipse, or parabola,
2
2
find
we
Art.
of
b for b in (5)
82,

By

to that

method

xx"

~a7"

yy"_
"

a)

to be the equation of the tangent to the hyperbola.

THE HYPERBOLA.

151

To deduce the value of the sub-tangent.


operating on (1) of the preceding article (see Art. 83),

113.

By
we

find

Sub-tangent

114.

The

hyperbola

(0,

x"

*-

"*

a*
.

slope of a line passing through the centre of an


is
0) and the point of tangency
(x",

y")

The

slope of the tangent


t

Art. 112, (1)

is,

^L

.*1

a2

y"

member by member, we have

Multiplying these equations,

=
Comparing

a)

$...

(1) of this article


ss

tt

with

(1) of Art. Ill,

we

find

(2)

Hence, the line from the centre of the hyperbola to the


point of tangency and the tangent enjoy the property of being
the supplemental chords of an
hyperbola whose semi-axes
bear to each other the ratio a

COR. If s = t, then s
t
if one
i.e.,
supplementary
chord of an hyperbola is parallel to a line drawn
through the
centre, then the other supplementary chord is
to the
;

parallel

tangent drawn to the curve at the point in which the line


through the centre cuts the curve.
115. The preceding principle affords us a
simple method
of drawing a tangent to the
hyperbola at any given point of
the curve.

PLANE ANALYTIC GEOMETRY.

152

FIG.

50.

Let P be any point at which we wish to draw a tangent.


7
to P O join C and A.
Join P and 0, and from A draw A C
will be the required
from
P
to
CA
The line P T, drawn
7

||

||

tangent.

To deduce the equation of the normal to the hyperbola.


can do this by operating on the equation of the tangent,
2
in the equa
as in previous cases, or by changing b into
tion of the normal to the ellipse, Art. 86, (3).
By either
116.

We

b"

method, we obtain

for the required equation.


117. To deduce the value of the sub-normal.
By a course of reasoning similar to that of Art. 87,

sub-normal

COR. If

we have

*/
%

$-

= a,
sub-normal

i.e.,

7/2

in the equilateral

x"

hyperbola the sub-normal

the abscissa of the point of tangency.

is

equal to

THE HYPERBOLA.

153

EXAMPLES.
Deduce the polar equation of the hyperbola, the pole

1.

being at the centre.


7.2

^!__
a 2 sin 2

-\-

cos

Write the equation of the tangents to each of the follow


ing hyperbolas, and give the value of the sub-tangent in
each case.

3.

9 y2

2.

"I

._

= - 36,
1,

at

at

4 ^2

= _ 36j
= abj

5.

6.

a?/

7.

^1-J^ = 1,

at

~=T7T

4 x2

to 2

(4,

(5,

ord.

ord.

4 Ord
>

+).

+).

+)

at

at

at

(aft, ord. +).

(Vm,

0).

7^

Write the equation of the normal to each of the above


hyperbolas, and give the value of the sub-normal in each
8.

case.
9.

x
6
The equation of a chord of an hyperbola is y
what is the equation of the supplemental chord, the

axes of the hyperbola being 12 and 8 ?


A
Ans.

10.

= 4- x

Given the equations


-

Y-

=~

-f-

1,

and y

-x=

required the equations of the tangents to the hyperbola at the


points in which the line pierces the curve.

PLANE ANALYTIC GEOMETRY.

154
11.

One

of
z

?/2

the supplementary chords of the hyperbola


144 i s parallel to the line y -=x what are

_ if} x = _

the equations of the chords ?

16

Ans.

aj

Given the

12.

2x

hyperbola

What

is

if

T
is

118.

3
the

required

at the positive

end of

the equation of a tangent to

x2

which

3 if

normal

equations of the tangent and


the right hand focal ordinate.
13.

_16.

_
:

_,

"T

parallel to the line 2?/

+ l = 0?

lines drawn to any point


by the tangent at that point.
Art.
equation of the tangent line,

The angle formed by the focal

of the hyperbola

is bisected

in the

o
Making y
we have

112, (1),

= OT.

Fig. 50.

From

OF = OF = ae
OF - OT = FT = ae - -^ =

Art. 104, (1)

hence

OF

\
x

(ex"

+ OT = F T =ae + -^ = ~-a
FT F T
+ a.
JC

.-.

::

+ a);

**.

ex"

ex"

(ex"

a).

But from Art. 105 we have


a
FF =
=
FT
+a
-a
FF FT
FT F T FF FT
ex"

ex"

.-.

Hence

PT

::

::

ex"

ex"

+ a.

FFF

divides the base of the triangle


i.e., the tangent
are proportional to the adjacent
which
into two segments,
the angle at the vertex.
bisect
therefore
must
it
sides
;

THE HYPERBOLA.

155

P N, Fig. 50, is perpendicular to


bisects the external angle formed by the focal

Since the normal

COR.

the tangent,

it

radii.

The

SCHOL.

method

of

Let

point.

FP FT
,

this

article

us

gives

Draw

be the point, Pig. 50.

The

P T drawn

line

between the focal


119.

of

principle

another

drawing a tangent to the hyperbola at a given


so as

the focal radii

to bisect

the angle

radii will be tangent to the curve at

To find the condition that the

fulfil in order that

it

line

= sx + c

must

touch the hyperbola

may

^ K=l
a2

By

method similar
s

a2

to that
b

=c

employed in Art.

89,

we

find

2
.

(1)

for the required condition.

COR. 1. Substituting the value of


equation of the line, we have
y

= sx

sV -

4-

drawn from

...

(1) in

the

(2)

for the equation of the tangent to the hyperbola in terms of its


slope.

120.

To find the
to the

hyperbola and a perpendicular

to it

tangent
as the point of tangency moves around the curve.

x 2 -f if
is

of a
a
from focus

locus generated by the intersection

a2

(1)

the equation of the required locus.

(See Art. 90.)

121. To find the locus generated by the intersection of tivo


tangents which are perpendicular to each other as the points of

tangency move around the curve.


x*
is

+if

=a

b*

the equation of the required locus.

(1)

(See Art. 91.)

PLANE ANALYTIC GEOMETRY.

156

Two

122.

without

tangents are draivn

tangency.

^_
a2

is

yy_
b

=i

(i)

To find the equation of the polar of

with regard

to the

the pole (x

),

hyperbola

_-!...
is

hyperbola from a point

(See Art. 92.)

the required equation.

123.

to the

required the equation of the line joining the points of

the required equation.

(1,

(See Arts. 49 and 93.)

To deduce the equation of the hyperbola when referred


a pair of conjugate diameters.

124.
to

A pair

of diameters are said

to be

conjugate

when they

are so

related that the equation of the hyperbola, when the curve is


referred to them as axes, contains only the second powers of the
variables.
~2

a
is

,,,2

%b

=1

(1)

the required equation, and

a 2 sin

sin

tan 6 tan

or

2
b cos 9 cos

<p

q>

=~

a2
-

is

= 0,

(2)

the condition for conjugate diameters.


(See Art. 94.)
From the form of (1) we see that all chords

COR.

parallel to

SCHOL.

drawn

one of two conjugate diameters are bisected by the other.

From

hence

Art. Ill, (1)

ss

= tan
= tan

we have

tan

<p.

we have

tan cp i.e., if one of


s
s
0,
a
to
is
diameters
tivo conjugate
chord, the other conju
parallel
to
the
is
diameter
supplement of that chord.
parallel
gate
If,

therefore,

THE HYPERBOLA.
From

157

we have

Art. 114

= il;
a2

hence
If,

tt

therefore,

= tan
= tan

we have

0,

= tan

<p

i.e.,

if one of

a tangent of the hyper


the other conjugate diameter coincides with the line joining

two conjugate diameters


bola,

tan

the point of tangency

125.

From

is

and

to

parallel

the centre.

the condition for conjugate diameters,


tan

tan

9?

2
.

a*

we

see that the products of the slopes of any pair of conju


is positive; hence, the slopes are both
positive

gate diameters

or both negative.

gate diameters must

126.

It appears, therefore, that


lie in the same quadrant.

any two conju

To find the equation of a conjugate diameter.

FIG.

51.

PLANE ANALYTIC GEOMETRY.

158
Let

P"R"

be any diameter

then

B/,

drawn through the


/A

parallel to the tangent at


Art. 124, Schol.
jugate diameter.
The equation of the tangent at

centre

(P

P"

N)

P"

a2

xx"

or

y")

is

hence, the equation of

(#",

will be its con

PB

is

yy"

"

7)2

^.^x
a

(2)

y"

^V

But

is

= cot

=1

P"OX

the equation of a diameter in terms of the slope of

its

conju

gate diameter.

127.

To find the

diameter, the

of either extremity of a

co-ordinates

of one

co-ordinates

extremity of

its

conjugate

diameter being given.

Let the co-ordinates of

By

P"

y"),

(x",

Fig. 51, be given.

a course of reasoning similar to that of Art. 96,

The upper

the point

signs correspond to

lower signs to the point B/

we

(x

y)

find

the

}.

128. To show that the difference of the squares of any pair


the
of semi-conjugate diameters is equal to the difference of
squares of the semi-axes.
By a course of reasoning similar to that of Art. 97, or, by
2
2
2
for b in (4) of that article, we
b for b\ - b
substituting

find

a2

2
.

(1)

THE HYPERBOLA.

b
COR. If a
b, then a
has equal conjugate diameters.

129.

i.e.,

the equilateral hyperbola

To show that the parallelogram constructed on any two

conjugate diameters
on the axes.

By

159

is

to the

equivalent

rectangle constructed

a method similar to that of Art. 98,

4 a

Area

i.e.,

b sin
(g>

0)

= 4 ab

we can show

that

MNM N = Area CDC D

Fig. 51.

EXAMPLES.

The

1.

line

= 2 x + c touches
_
_
""

-i

the hyperbola

what

,y

is

the value of

tangent to the hyperbola

c ?

Ans.
2.

10
its

Y-intercept

j
i
12
2 required
;

its

V32.

-L,

slope and equation.

VI76

Ans.

tangent to the hyperbola 4?/


angle of 45 with the X-axis required
3.

_*!

has

2x

=
=

VlTO x
6

+ 2.

makes an

its equation.
2
are
drawn
to
the
9 x2
tangents
hyperbola 4 y
- 36 from the point (1, 2) required the
of
the
chord
equation
of contact.
;

4.

Two

Ans. 9 x
5.

What

focus ?
6.

the equation of the polar of the


Of the left-hand focus ?

What

is

is

the polar of
4 if
x2

(1,

7.

8 y

36.

right-hand

with regard to the hyperbola

^5.

4?

Find the diameter conjugate

to

=x

cc

?/

= 4.

in the hyperbola

.2

J/_

16

Ans.

PLANE ANALYTIC GEOMETRY.

160
8.

Given the chord y

^ _ t.

= 2x
=1

6 of the hyperbola

-f-

required the equations of the supplementary chord.


Ans. y

9. In the last example find the equation of the pair" of


conjugate diameters which are parallel to the chords.
2 x.
Ans. y
2 x, 9 y

10.

144

The point

(5,

^)

lies

on the hyperbola 9

if

16 x 2

required the equation of the diameter passing through


also the co-ordinates of the extremities of its conjugate
it
diameter.
;

130.

To deduce the equations of the

rectilinear asymptotes

of the hyperbola.

An ASYMPTOTE

of a curve

is

line

passing within a finite

distance of the origin which the curve continually approaches,


and to which it becomes tangent at an infinite distance.

FIG.

52.

THE HYPERBOLA.

161

The equation of the hyperbola whose transverse axis


along the X-axis may be put under the form
y*

~ (^ -

...

lies

(i)

(ju

DD CO

The equations of the diagonals,


constructed on the axes
BB are
,

AA

y
or,

squaring,

of the rectangle

-x,
a

2y"

7,2

x2

(2)

a/
r

where y represents the ordinates of points on the diagonals.


Let P (x, y] be any point on the X-hyperbola and let
Sub
(x, y } be the corresponding point on the diagonal DD
tracting (1) from (2) and factoring, we have

D"

<y-z/)0/+2/)

y =

hence

D"P

2
;

(3)

y
y
As the points
P recede from the centre, 0, their ordi
nates D"]ST, P N increase and become infinite in value when
and P are at an infinite distance.
But as the ordinates
D",

D"

increase the value of the fraction

(3), which represents their


becomes zero when y and y are
and P are continually approach
ing each other as they recede from the centre until at infinity
they coincide. But the locus of
during this motion is the

difference, decreases and


infinite
hence, the points
;

D"

D"

DD

infinite

diagonal
hence, the diagonals of the rectangle
constructed on the axes of the hyperbola are the
asymptotes of
;

the curve.

Therefore

= + * x and y = - - x
a
a

are the required equations.


COB. 1. If a
b, then

=
y =

i.e.,

45

-4-

x and

?/

the asymptotes of the equilateral


hyperbola
with the X-axis.

make

angles of

PLANE ANALYTIC GEOMETRY.

162

COR. 2. The equation of the hyperbola COD jugate to


may be put under the form

Subtracting (1) from

hence, aw hyperbola and


totes of each other.

COR.

3.

ti"

if/ie

its

p/ == _

conjugate are curvilinear

Subtracting (2) from

y
hence,

we have

(4),

i,

-=

(1)

"P

asymp

we have

(4),

Y\"

"\v

+y

-jr~>

rectilinear asymptotes of

>

an hyperbola and of

its

con

jugate are the same.


131.

To deduce the equation of the hyperbola when referred

to its rectilinear

asymptotes as axes.

FIG.

The equation
is

53.

of the hyperbola

when

referred to

OY, OX,

THE HYPERBOLA.

163

We wish to ascertain what this equation becomes when


OY OX the rectilinear asymptotes are taken as axes.
Let P be any point of the curve let Y OX = XOX = 0.
Then (OC, CF) = (x, y) (OD, DP = (x y
From the figure, OC = OK + DR CP = RP - DK
,

}.

y =
But from the triangle OAB, we have
x

i.e.,

hence, x

+y

(x

cos

(ij

-x

sin

B.

(x

-+-

Va^+ft

Substituting these values in


f

(x
or,

(9

+ 7/) 2

(y

we have

(1),

reducing and dropping accents,


2
a2
b
Xy
...

= -+

J__

34

= a* -f ^

2
,

(2)

for the equation of the hyperbola referred to its asymptotes.


In a similar manner we may show that

is

the equation of the hyperbola conjugate to

ferred to

COR.

asymptotes as axes.
Multiplying (2) by sin 2 6

(1),

when

re

its

we may

place the result in

the form

yx sin 2
that

is

therefore

^1A

+*

2
"

DP P H = ON. AH
area ODP P = area OMAN

8jn

hence, the area of the parallelogram constructed upon the co


ordinates of any point of the hyperbola, the asymptotes being
axes, is constant and equal to the area of the
structed upon the co-ordinates of the vertex.

rhombus con

PLANE ANALYTIC GEOMETRY.

164
132.

when

To deduce

the equation of the tangent to the hyperbola

the curve is referred to its rectilinear asymptotes as axes.


Y,

FIG.

54.

a course of reasoning similar to that employed in Arts,


41, 57, 82, we find the required equation to be

By

y-Y = -^(*-*")
X
or,

...

(i)

symmetrically,

1^=2...
COR. If we make y

=2
OM =
x

But

x"

in (2),

x",

OT.

.-.

(2)

we have
Fig. 54.

OM = MT

.-.

TD

= TD

hence, the point of tangency in the hyperbola bisects that por


tion of the tangent included between the asymptotes.

THE HYPERBOLA.

Since

133.

+ b\
= a +b
OT OT = a + b

4
2

or

x"y"

x"

a2

2 if

2
;

i.e.,

we have

a point of the hyperbola,

is
?/")

(x",

(see Fig. 54)

165

(1)

hence, the rectangle of the intercepts of a tangent on the


totes is constant

and equal

to the

sum of

asymp

the squares on the

semi-axes.

From

134.

(1) of the last article

sm

ing through by

we

have, after multiply

OT-^LfYl v
sin 20 =

n- J_ A 2
"f-^-

sin 2

(a

+b

sin

cos

0.

But, Art. 131,

sin

-J
2

sin 2

arm OTT

i.e.,

(t

Va + b

OT or

hence

.-.

cos

Va +

= a&

is

equivalent

to

arm OAD B.

Ae triangle formed by a tangent

asymptotes

to

the hyperbola

Draw the chord RB/, Fig. 54, parallel


Draw also the diameter OL through D.
Since TD = T D, we have B/L = BL.

135.

and

its

the rectangle on the semi-axes.


to the tangent

T.

Since

i.e.,

OL

is

LK = LH
LK = BL LH

a diameter,

we have

BL
B K = BH

hence

hence, the intercepts of a chord between the hyperbola


asymptotes are equal.

and

its

PLANE ANALYTIC GEOMETRY.

166

EXAMPLES.

What

1.

are the equations of the asymptotes of the hyper~2

bola

,.2

f-gAns.

What

| x.

-J-

are the equations of the asymptotes of the follow

ing hyperbolas

_.
3.

3 ^2

2 ^2

Ans.

=_
y = -^ Vf x.

mx

Uy 2

6. What do the equations given in the four preceding ex


amples become when the hyperbolas which they represent are
referred to their asymptotes as axes ?

The semi-conjugate axis of the hyperbola xy


what is the value of the semi-transverse axis ?

7.

25

Ans.

What

is

8.

are the equations of the tangents to the following

hyperbolas

8.

To xy

9.

To xy

=+

10. at (1, 10).

Ans.

Ans.
10.

To xy

11.

To xy

m, at

= - p,

1,

at

+ 10 x = 20.
= 3 x 12.

12, at (2, 6).

-\-

m).
2,
/

Required the point of the hyperbola xy


4.
the sub-tangent
12.

12 for which

Ans.

(4, 3).

an hyperbola

13. The equations of the asymptotes of


whose transverse axis = 16 are 3 y = 2 x and 3 y

2x

required the equation of the hyperbola.


*

"

64

256

THE HYPERBOLA.

167

14.
Prove that the product of the perpendiculars
from any point of the hyperbola on the
asymptotes
stant and

let fall
is

con

GENERAL EXAMPLES.
1.

10

The point

(6, 4) is on the hyperbola whose transverse


required the equation of the hyperbola.

is

Am.
25

400

Assume

the equation of the


hyperbola, and show that
the difference of the distances of
any point on it from the
foci is constant and
2 a.
2.

Required the equation of the hyperbola, transverse


axis
2 x and 3 y
6, which has 5 y
13 x for the equa
tions of a pair of conjugate diameters.
3.

^_^/_ = 1

An*.

9
78
that the ratio of the sum of the focal radii of
any
point on the hyperbola to the abscissa of the point is con
stant and
2 e.
4.

Show

5.

fulfil

What

are the conditions that the line

= sx

-f c

must

= 0.

in order to touch

-^
a
2

-2
b

1 at infinity
J ?

Ans.

_L_

6. Show that the


conjugate diameters of an hyperbola are
also the conjugate diameters of the
conjugate hyperbola.
7.
Show that the portions of the chord of an
hyperbola

included between the hyperbola and its


conjugate are equal.
8.
What is the equation of the line which
passes through
the focus of an
hyperbola and the focus of its conjugate
hyperbola ?

Ans.

PLANE ANALYTIC GEOMETRY.

168

Show

9.

that

when

and

_
a

are the eccentricities of

two conjugate hyper

bolas.

Find the angle between any pair of conjugate diame

10.

ters of the hyperbola.

Show that in the hyperbola the curve can be cut by


of two conjugate diameters.
one
only
11.

Find whether the line y = f x intersects the hyperbola


9 x2 =
144, or its conjugate.

12.

16 y~

Show that the conjugate diameters of the


hyperbola make equal angles with the asymptotes.
13.

Show

14.

eral

drawn from any point

that lines

equilateral

of the equilat

hyperbola to the extremities of a diameter

make equal

angles with the asymptotes.

In the equilateral hyperbola focal chords drawn parallel


to conjugate diameters are equal.
15.

from the focus of an hyper


perpendicular is drawn
show
the
bola to
asymptote
that the foot of the perpendicular is at the distance a
16.

(a)

from the centre, and


(b)

that the foot of the perpendicular

from the
17.

is

at the distance b

focus.

For what point of an hyperbola

is

the sub-tangent

the sub-normal ?
that in the equilateral hyperbola the length of
the normal is equal to the distance of the point of contact
18.

Show

from the
19.

centre.

Show

chord of

that the tangents drawn at the extremities of any


diameter which
hyperbola intersect on the

the

bisects the chord.

THE HYPERBOLA.
20.

Find the equation of the chord of the hyperbola


J?L -

J^!

is

=1

12

which

169

bisected at the point

(4, 2).

Required the equations of the tangents to

21.

J*L.

I*-

16

10

which make angles of 60

with the X-axis.

that the rectangle of the distances intercepted on


drawn at the vertices of an hyperbola by a
drawn at any point is constant and equal to the

Show

22.

the tangents

tangent

square of the semi-conjugate axis.

Given the base of a triangle and the difference of the

23.

tangents of the base angles

Show

24.

required the locus of the vertex.

that the polars of

(m, n) with respect to

the

hyperbolas

^1
2

_ J/l =

If

25.

1,

b*

ll

- ^1 =

V2

a2

1 are parallel.

from the foot of the ordinate of a point

hyperbola a tangent be drawn to the

circle

(x,

y) of the

constructed on

the transverse axis, and from the point of tangency a line be


drawn to the centre, the angle which this line forms with the
Show
transverse axis is called the eccentric angle of (x, y).
values
deduce
from
these
and
b
tan
sec
that
go,
go),
y)
(a

(x,

the equation of the hyperbola.


26.

If

(x

),

(x",

y")

are the extremities of a pair of

conjugate diameters whose eccentric angles are


that

<p

<p

90.

go

and

cp,

show

PLANE ANALYTIC GEOMETRY.

170

CHAPTER

IX.

THE GENERAL EQUATION OF THE SECOND DEGREE.


136. The most general equation of the second degree be
tween two variables is
2
2
(1)
ay + bxij + ex + dy + ex

+f=

whatever.
d, e, /are any constant quantities
this
which
loci
the
of
the
To investigate
equation
properties
of the constants as to
represents under all possible values

in

which

a, b,

c,

sign and magnitude

is

the object of this chapter.

The equations of the lines in


have had to do in preceding chapters,
137.

By + C = 0.
2
=
(Ax + B# + C)
_
_
Two
o.
yS.
2/2

Ax

-f

_j_

x2

7/2 _|_

xt

7/2

2
7/

=a
_ o.

= 2px.

0.

a plane, with which


are

we

Straight line.
Two coincident straight lines.

straight lines.

Circle.

Two

imaginary straight

lines.

Parabola.

= a 2^ Ellipse.
_
=
crb
tfxay
Hyperbola.
= a 2j^ Hyperbola,
a 2^2 _

0,2^2 _|_

2^,2

#83.2

we
Comparing these equations with the general equation,
them may be deduced from it by making the
constants fulfil certain conditions as to sign and magnitude.
the lines which
We
therefore, prepared to expect that
see that all of

are,

these equations represent will appear among the loci repre


sented by the general equation of the second degree between

two

variables.

find that these

equation.

In the discussion which


lines

is

to ensue

are the only loci represented

we

by

shall

this

EQUATION OF THE SECOND DEGREE.

171

DISCUSSION.
138. To show that the

locus represented by

a complete equa

tion of the second degree between two variables is also represented


by an equation of the second degree between two variables, in
which the term containing xy is wanting.

Let us assume the equation


ex* +dy
ex
aif
bxy
(1)
and refer the locus it represents to rectangular axes, making
the angle
with the old axes, the origin remaining the same.
Prom Art. 33, Cor. 2, we have

+f=

=x

cos 6

sin 6

in

+bx

cos

Substituting these values

+ V + d y + e x +/=
2

y>

y sin

+y

for the equations of transformation.


in (1), we have,

a y*

(2)

which

= a cos
= 2 (a
c = a sin
d! = d cos
e = d sin
a!

+ c sin

6
sin B cos
2
-\- c cos
e sin
-J- e cos

c)

cos

b sin

+ b (cos 6
+ 6 sin cos
2

sin 2 0)
I

(3)

0, the angle through which the axes have been turned,


entirely arbitrary, we are at liberty to give it such a value
as will render the value of b equal to zero.
Supposing it to

Since

is

have that value, we have


or

2 (a

c)

(a

sin 2

c)

or

cos 6

sin

tan

20

(cos

^
.

Since any real

+b

-f b cos

number between

-f-

sin 2 0)

0,

(4)

(5)
GO

and

o>

is

the tan

gent of some angle, equation (5) will always give real value
for 20; hence the above transformation is
always possible.

Making

in (2),

a lf
for the

we

have, dropping accents,

+ eV + d y + e x +f =

equation of the locus represented by


equation, then, we shall confine our attention.

(1).

(6)

To

this

PLANE ANALYTIC GEOMETRY.

172

To find the value of a and c in terms of a, b,


Adding and then subtracting the first and third of

COR.

and

1.

c.

the equations in

we have

(3),

+a =c+a
(7)
= (c - a) cos 2 + b sin

Squaring

(4)

and

...

...

V(c

(7)

+ b*

a)

and

(8)

we have

adding to the square of (8),

Subtracting and then adding

(9)

we have

(9),

= i\ G + a- V(c - ay +Tj
= $ \c + a + V(c-a) + ^

(10)

COB.

and

To

2.

find the signs of a!

and

y=
...

Hence, the
2

i |( C

=-

of a

si#?is

(V

and

-(( -- 4 ac)

= 0,

.-.

)
.

(12)

depend upon the sign of the

ac.

<

+ * )h

2
<0

4
quantity b
The following cases present themselves
2
4 ac. The sign of the second
6
1.

2.

Multiplying (10)

we have

(11),

positive,

(11)

a and

= 4 ac. The
or c = 0.

member of (12)
are both positive, or both negative.
second

member

of (12)

becomes

zero,

is

.-.

cannot be equal to zero


would reduce (6) to
at the same time, for such a supposition
an equation of the first degree.]
2
4 ac. The sign of the second member of (12) is
3. b
and c negative, or a must be
a must be
be observed that
[It will

a and

>

negative,

positive

.-.

negative and

c positive.
7

To transform the equation a if + c x -f a ?/ + e a -f


the vari
f=Qinto an equation in which the first powers of
2

139.

ables are missing.

EQUATION OF THE SECOND DEGREE.

173

Let us refer the locus to a parallel system of rectangular


From Art. 32, we
axes, the origin being at the point (m, n).
have
a y = n
x = m
y
Substituting these values in the given equation, we have
2
*
=
c x
+
+f"
(2)

y +

in

e"x

d"ij

which
d"

c"

/"

Since

=2an +d
= 2 c w -f e
= a + cm +
?i

rf w-

??t

+/

make

as to

+d =

and 2

-f e

m=

and

=
.

2 a
see

3)

we may make

in general,

We

and w are entirely arbitrary, we may, in general,


2 a n

i.e.,

them such values

give

(4)

from these values that when a! and c are not zero,


is possible
and equation (2) becomes,

this transformation also

after dropping accents,

a if +cV+/"

Equation

...

(5)

we

observe, contains only the second power of


hence it is satisfied for the points (x, y) and

(5),

the variables

But only the equation of curves with centres


can satisfy this condition hence, equation (5) is the equa
tion of central loci.
When either a f or is zero, then n or in
(

y).

a?,

</

and the transformation becomes impossible.


two cases which require special consideration.

is infinite

arise

CASE 1. a! == o.
Under this supposition equation

Hence

140.

(6),

Art. 138,

cV-MV + ^+/=0

...

becomes

(1)

Referring the locus of this equation to parallel axes, the


origin being changed, we have for the equations of trans
formation
x
m x y n -f- y

PLANE ANALYTIC GEOMETRY.

174

Substituting in
C

V + dy + (2
2

Now,
make
i.e.,

we may

in general,

m+

we have

(1),

m + e ) x + c m* + d n + e m

= 0,

+f=Q

(2)

and w such values as to

m + d w + e m +/=
2

we may make

m=
m
If d
mation

is
is

-|- e^/^i, -f-

/_

4/c
4

rZ

not zero (since a!


0, c is not zero), this transfor
possible and (2) becomes, after dropping accents,

x2

Or
2

COR.

and

f ,

If

-\-

= 0,

dy

jr
a

-^^...

0, (1)

c x*
or,

and

give

/Q\
(3)

becomes

+ e x+f=0

solving with respect to

...

(4)

x,

CASE 2. c = o.
Under this supposition equation

141.

(6),

Art. 138, becomes

Transforming this equation so as to eliminate ?/ and the


constant term, by a method exactly similar to that of the
preceding

article,

we

find
r

n =

J*
d*

nyi

- 4 a!f
J_

4fcV
and,

if e

is

not zero,

we have

f=--,*

(of is

not zero since

(2)

= 0)

EQUATION OF THE SECOND DEGREE.


COB. If

142.

= 0,

equation

Summarizing the

175

becomes

(1)

results of the preceding articles,

we

find that the discussion of the general equation

aif

bxy

+ ex + dy + ex +/ =
2

has been reduced to the discussion of the three simple forms


1.

a! if

+ cV +

x2

/"

= -

--

x.

= 0.

Art. 139, (5)

Art. 140, (3)

Art. 141, (2)

^-.

Art. 140, (5)

3.

^-.

Art. 141, (3)

The discussion now involves merely a consideration


sign and magnitude of the constants

of the

which enter into these

equations.

143.

tf

<

ac.

Under this supposition, since of and d are both positive or


both negative, Art. 138, Cor. 2, neither a! nor c can be zero
hence, forms 2 and 3 of the preceding article are excluded
;

from consideration.
The first form becomes either

ay

or

which

c a

of and c may have


any real value and
and
any sign
any value. Hence arise four cases

in

f"

may have

PLANE ANALYTIC GEOMETRY.

176

CASE

1.

If

has a sign different from that of a and c


whose semi-axes are
,

f"

equations (1) are equations of ellipses

GASP:

2.

If

f"

has the same sign as that of a and

equa

curves.

tions (1) represent imaginary


has a different sign from that of
c and
CASE 3. If a

f"

has
If
circles.
equations (1) are equations of
the same sign as of and c then the equations represent imagi

a and

/"

nary

curves.

CASE
nary

4.

If/"

= 0, equations (1)

are equations of taw imagi

the origin.
straight lines passing through

Hence, when

z
<

ac, every

equation of the second degree

an imaginary curve,
between two variables represents an ellipse,
at the
a circle, or two imaginary straight lines intersecting
origin.
b

Under

= 4 ac.

either a
this supposition, Art. 138, Cor. 2,
form (1) of Art. 142 is excluded.

- 0,

or

hence,

Resuming the forms

(3)

we have

~~

(>

four cases depending upon the sign and magnitude

of the constants.
are not zero,
CASE 1. If d and c in the first form of (2)
not
are
of
zero, then
form
second
the
in
(2)
and if e and a
of parabolas.
equations (2) are equations

EQUATION OF THE SECOND DEGREE.

177

Since the first form of (3) is independent of y, it


lines parallel to each other and to the Y-axis.
two
represents
The second form of (3) represents, similarly, two lines which

CASE

2.

are parallel to the X-axis.

CASE

3.

If

2
e"
<

4/b the

form of

first

(3)

represents two

lines.

imaginary
two imagi
If d 2
4/o/, the second form of (3) represents
lines.
nary
CASE 4. If e 2 = 4/c the first form of (3) represents one
<

straight line parallel to the Y-axis.


4 fa the second form of (3) represents one straight
If d 2
line parallel to the X-axis.

2
4 ac, every equation of the second degree
Hence, when b
between two variables represents a parabola, two parallel straight
lines, two imaginary lines, or one straight line.

145.

l>

>

ac.

Under this supposition, Art. 138, Cor. 2, since a! and


f
must have opposite signs, neither a nor c can be zero
hence forms (2) and (3) of Art. 142 are excluded from con
The first form becomes either
sideration under this head.
c

ay

-<fc

-y +

or

We

+/"

cV+/"

1.

If

=OJ

f has a different sign from that of a

(1) are equations of hyperbolas

/r

0=v
If

have here three cases.

CASE

still

equations

whose semi-axes are

andJ=v/r,

f has a different sign from that of

equations (1) are

equations of hyperbolas.

CASE

2.

If a

eral hyperbolas.
CASE 3. If f"

= 0,

secting straight lines.

equations (1) are equations of equilat

equations (1) are equations of two inter

PLANE ANALYTIC GEOMETRY.

178

4 ac, every equation of the second degree


between two variables represents an hyperbola, an equilateral
hyperbola, or two intersecting straight lines.

Hence, ivhen

>

The preceding discussion has

SUMMARY.

146.

elicited the

following facts
1. That the general equation of the second degree between
two variables represents, under every conceivable value of the
:

constants which enter into


bola, or one
2.
3.

it,

of their limiting

When
When

When

4&c

<

it

4ac

an

ellipse,

an

represents
it

a parabola, an hyper

cases.
ellipse, or

a limiting

case.

represents a parabola, or a limiting

case.
4.

4&c

>

if.

represents

an hyperbola,

or

a limiting

case.

EXAMPLES.
1.

Given the equation 3y 2

-\-

2 xy

+3x

Sx

Sy

to classify the locus, transform and construct the equation.


Write the general equation and just below
(a) To classify.
it the given equation, thus
:

+ bxy + ex + dy -f ex +/ =
8x =
8y
3 if + 2 xy + 3 x
ay

Substituting
#>

_ 4 aCj

the

co-efficients

we have

4 ac

tf

the

in

=4-

2
ac.
b
hence
and the locus belongs to the ellipse

36

class

= -

(1)

characteristic

32

<4

(b)

To

class, Art. 146.

refer the locus to axes such that the term containing

xy shall disappear.

From

Art. 138, (5),

we have

tan 2

hence

tan

20

o
.-.

2(9

= 90

.-.

6>

=+
= 45

oc,

(2)

EQUATION^OF THE SECOND DEGREE.


new X-axis makes an angle of
X-axis.
Taking now (10), (11), (3), Art.
ing values, we have
i.e.,

the

of

179

45 with the old

-j-

138,

and substitut

= i \c + a - V(c - a)- + = 2.
= i \c + a + V(c- a) + J=4.
= dcosO e sin = V2 (d e) =
= d sin + e cos = V^ (^ + e =
l>*\

d
e

these

being zero),
2 y2

8 V2.

in

values

0.

J-

(9

Substituting

Art.

(6),

+ 4 x - 8 ViT x =0
2

138,

...

we have (/

(3)

To re/er ^^^ ^oci^s to its centre and axes.


(c)
Substituting the values found above in (4), Art. 139,

have

U.

2c

= an + cm
2

Hence/"

(3).

-f

cf

found above in
2 if

f"

8,

Art. 139,

+4x

for the reduced equation.

= V2
(d)

Draw
X-axis.

curve

and

To

together with the values of a

we have

(5), Art. 139,

+ %- = 1

or

+ em +/=

dn

Substituting this value of

and

we

...

0,

(4)

The semi-axes

of the ellipse are

2.

construct.

the axis

OX

See

Draw OY

when

(b).

making an angle of 45 with the old


The equation of the
-Lto OX

referred to these axes

is

given in

(3).

Constructing

PLANE ANALYTIC GEOMETRY.

180
the point
(c).

(V2,

Draw O

when

Y"

referred to

i-

See
0) we have the centre of the ellipse.
The equation of the curve
to OX" at
.

Y",

as axes is given in (4).

FIG.

55.

V2 and 2, we can construct the


by either of the methods given in Art. 78.

Having the semi-axes,


ellipse

DISCUSSION.
If

in (1),

x
If

If

to the
If

= 0,

for the X-intercepts 0,

=-

in (1),

we have

in (3),

we have y

OD,

for the Y-intercepts 0,

_|_0;

i.e.,

OC,

the ellipse

is

tangent

Y -axis.

in (3),

x
If

we have

x = o in

(4),

we have

0,

we have

y=

=1=2.

for the

X -intercepts

0, OB,

= 2 V2.
for the

Y"-intercepts

A,

OA

EQUATION OF THE SECOND DEGREE.


If

in (4),

we have

X -intercepts O B, O O,

for the

V2.

-j-

Given the equation ?/ 2


2 xy -f # 2
2y
1
and construct the equation.
To classify.

2.

181

0, class

ify the locus, transform


(a)

aif

y
hence

bxy

+ ex

ajy 4-

4 ac

a?

dy

-f

2y

=4

+/ =

-f ex
1

= 0,

(1)

hence the locus belongs to the parabola


class, Art. 146.
To refer the locus to axes such that the term
(b)
containing

xy shall disappear.

From

Art. 138, (5),

we have
b

20=

tan

hence, substituting

tan 2

-1

= -45

...

...

(2)

Substituting the values of the coefficients in (10)


of Art. 138,

(11)

(3)

we have

= \ c + a + V(e a)
= 2.
b*\
= d cos
= 2 ( V2) = V2.
e sin
e = d sin
+ e cos = - 2 (- 1 V2) = + V2.
Substituting these values in (1), Art. 140 (since a =
0),
we have
2 x - V2 y + V2^ - 1 =
(3)
2

-f-

d!

(c)

To

refer the

parabola

to

a tangent at the vertex and the

axis.

Substituting the values of the constants in (a), Art.


140,
r

we have

m=-

^-7

= - V2 = - -35

e
4fc =
~~T^~~
4 d c

T~7^
4 V2

=-

nearly.

90 nearly.
J

PLANE ANALYTIC GEOMETRY.

182

is

Substituting the values of d and


not zero), we have

= iV2.y

c>

in (3), Art. 140


(since

(4)

for the reduced equation.


(d) To construct.

Draw

OX

making an angle

OY 1 to OX

of -- 45

with the X-axis

See (). The equation of the parabola


when referred to these axes is given in (3).
vertex
.35,
Constructing the point (
.90), we have the

draw

and O
of the parabola
See (c). Draw O
parallel to
the axes
OY respectively. The equation of the parab
The curve can now
ola referred to these axes is given in (4).
be constructed by either of the methods given in Art. 54.
.

OX

X"

Y"

EQUATION OF THE SECOND DEGREE.

183

DISCUSSION.
If

If

If

we have

in (3),

we have

==

OD

Y -intercept OK,

.707.

we have

for the

- V2 + yio

Jj

X -intercepts OL, OL
- V2 - Vio
Z=

4
If

OD,

V2

in (3),

*/

for the

~=-

00

.4.

for the Y-intercept

-l.

=-

in (1),

00,

for the Y-intercepts

y = 2.4

y
If

we have

in (1),

in (4),

?/

if

in (4),

_j_

-\-

bxy

ex 2

-f-

dy

2y + 6x

ex

-\-

f=
.

0.

= 0, classify

Given the equation ?/ -^ 2 cc 2


2 ?/ + 6 x
3
the locus, transform and construct the
equation.
(a) To classify.
3.

ay

0.
.

(1)

hence, the locus belongs to the hyperbola class, Art. 146.


(b) To ascertain the direction of the rectangular axes

(xy

being wanting).
tan
6

.-.

i.e.,

the

new X-axis

26

= 0;
is

c-

= -5_ = Q
-3

parallel to the old X-axis.

To refer the hyperbola

(c)

to

its

centre

and

axes,

we

have,

Art. 139, (4),

hence

1,

m=

-.
t

Substituting in the value of

/ =
hence

a n2

+cm +
2

f"

dn

=1

+em

/",

Art. 139, (3),

+/ = 1

we have

+9

PLANE ANALYTIC GEOMETRY.

184

This value, together with the values of a? and


2 if
4 x* = - 1
139, gives
(3)
for the required equation.

To

(d)

in (5), Art.

construct.

FIG.

Construct the point

OX, and O

Y"
||

(%, 1),

is

and through

The equation

OY.

to

referred to these axes

57.

given in

We

(3).

tion that the semi-transverse axis

is

-.

it

draw O

of

the

X"
||

hyperbola

see from this equa

Laying

tance to the right and then to the left of


vertices of the curve A, A

off this dis-

we

locate the

DISCUSSION.
If

y
If

we have

in (1),

= o in

(1),

= 3, y =
we have

for the Y-intercepts

OC,

00

OD,

OD

1.

for the X-intercepts

+ V3

to

3- V3

EQUATION OF THE SECOND DEGREE.

If

in (3),

we have

= iVH^

2/

If

185

in (3),

we have

for the X-intercepts

A,

OA

.-4
From

this data

the student

readily determine the


focal distances of the

may

and the

eccentricity, the parameter,

hyperbola.

= 0,

Given the equation y 2 -\- x z


4 ?/
4#
1
the
ify
locus, transform and construct the equation.
4.

b2

(a)

<4

(b)

ac
.-.

the locus belongs to the ellipse class.

.-.

new X-axis

(m,w)=(-2,

(c)

x2

hence

class

-f-

is

to old X-axis.

||

2)and/"

= -9

=9
y-

the transformed equation of the locus, which from the form


of the equation is evidently a circle.
is

Locate the point

(d)

and with 3 as a

(2,

With

2).

this point as a centre,


it will be the re
;

radius, describe a circle

quired locus.
5.

2 xy

+x

b = 4 ac
= 45
2

(a)
(b)

.-.

new X-axis

We

to the old X-axis.

...

= 0.

parabola class.

.-.

a!

= 0,
2x

i.e.,

c = 2, d = 0,
- 2^0;
= 1 and x =

are the equations of the locus


(c)

inclined at an angle of

45

have also

when

The construction gives the

...

(1)

new

referred to the

lines

OX OY
,

axes.

as the

new

axes of reference.

Equations

drawn

||

(1) are

to the

the equations of the two lines


and at a unit s distance from

Y -axis

CM, C
it.

PLANE ANALYTIC GEOMETRY.

186

FIG.

58.

We may

construct the locus of the


given equation without
going through the various steps required by the general
method. Factoring the given equation, we have
(y

hence

+ V2)

=x

V2

(y

-x- V2) =

and y

=x

-f-

V2

the equations of the locus.


Constructing these lines
(OY,
being the axes of reference), we get the two

are

OX

CM, C

parallel lines

Classify, transform,

equations
6.

?/

and construct each of the


following

2 xy

+x +2y

2 x -f 1

= 0.
y

7.

8.

if

ajy

x2

- 1 = 0.

4. 5

a-s

12 x

- 12 y = 0.

=x

1.

EQUATION OF THE SECOND DEGREE.


9.

10.

2?/

4x

+ 2x -4?/2

+1

=0.
x2

7/2 _j_ jpa

_ 2 + 1 = 0.

x2

+ 2*+2 = 0.

187

2
7/

aj

(1, 0).

11.

2/

Imaginary
12.

ellipse.

+ x - 8 x + 16 = 0.
2

XT/

Parabola.
13.

2 xy -f x

if

+2x

= 0.
Parabola.

14.

4 ay

15.

?/

2x

0.

Hyperbola.

16.

2 or

+ 2 + 1 = 0.
?/

Two

intersecting lines.

- x + 2 y + 2 x - 4 = 0.
2

if

Equilateral hyperbola.
17.
18.
19.

20.
21.

+ x + 2 y + 1 = 0.
/ + 4 xy + 4 - 4 = 0.
2 XT/ + 2 x
2 y + 2 x = 0.
4
+ 4 x = 0.
if
x + 2 = 0.
2 jcy
2

?/

jcy

ic

2/

a;//

?/

PLANE ANALYTIC GEOMETRY.

188

CHAPTER

X.

HIGHER PLANE CURVES.


147. Loci lying in a single plain and represented by equa
tions other than those of the first and second degrees are

We

HIGHER PLANE CURVES.


shall confine our atten
tion in this chapter to the consideration of a few of those
curves which have become celebrated by reason of the labor
called

expended upon them by the ancient mathematicians, or which


have become important by reason of their practical value in
the arts and sciences.

EQUATIONS OF THE THIRD DEGREE.


148.

THE SEMI-CUBIC PARABOLA.

This curve
ordinate

OP

TT

let fall

is

the locus generated by the intersection of the


common parabola with the perpendicular

of the

from

its

vertex upon the tangent drawn at

as

the point of tangency moves around the curve.


1.

To deduce the rectangular equation.

T
be the point of tangency, and let P (x y )
be a point of the curve.
Let y 2 == px be the equation of the common parabola.
to the parabola
Since the equation of the tangent line T
Let

(x" ,

y")

is

Art. 57,

(6),

the equation of the perpendicular


vertex is

{r

(OM)

let

fall.

from the

HIGHER PLANE CURVES.

189

FIG

Since

TT

is

*=*",

Combining

OY, we have

parallel to

(1)

and
(2)>

for its equation

(2)

we have

But

_ V4j^_

hence

2p
Squaring and dropping accents, we have

for the equation


of the semi-cubic parabola.
This curve is remarkable as
being the first curve
rectified,

that

is,

the length of a portion of

it

which was
was shown to

PLANE ANALYTIC GEOMETRY.

190

be equal to a certain number of rectilinear units. It derives


its name from the fact that its equation (3) may be written

xl

= p% y

To deduce the polar equation.


r
Making x = r cos 6 and y

2.

reduction,
r

=p

tan

sec

sin 9 in (3),

we

have, after

(4)

for the polar equation of the curve.

SCHOL.

Solving

(3)

with respect to

y,

we have

An

inspection of this value shows


(a) That the curve is symmetrical with respect to the

X-axis;
(b)

That the curve extends

infinitely

from the Y-axis in

the direction of the positive abscissas.

To duplicate the cube by the aid of the parabola.


Let a be the edge of the given cube. We wish to con
struct the edge of a cube such that the cube constructed on it
shall be double the volume of the given cube
i.e., that the
3
8 =
satisfied.
shall
be
a
2
x
condition
149.

FIG.

60.

HIGHER PLANE CURVES.

191

Construct the parabola whose equation is


2
2 ax
i/
(1)
be the curve. Construct also the parabola whose
Let

MPO

equation

is

x*~y...
NPO be this
Then OA (= x)
Let

is

the abscissa of their point of intersection

the required edge.

we have
x8

(2)

curve.

For eliminating y between

(1)

and

(2),

= 2 a*.

This problem attained to great celebrity among the ancient


We shall point out as we proceed one of the
geometricians.

methods employed by them in solving


150.

The

THE

it.

CISSOID.

by the intersection (P) of


(OMM T) with the ordinate
DN

cissoid is the locus generated

the chord

(OM )

of the circle

s
FIG.

61.

PLANE ANALYTIC GEOMETRY.

192

MN (equal to the
M on the diameter

MN

ordinate

let fall

from the point

as the chord revolves about

through 0)

the origin 0.
It may also be defined as the locus generated by the inter
z
8 ax with the
section of a tangent to the parabola y
let fall on it from the origin as the point of
perpendicular
moves around the curve.

tangency

To deduce the rectangular equation.


Let OT = 2 a, and let P (x, y) be any point
From the method of generation in this case
of the curve.
1.

First Method.

ON = N T.
MN = M
ON M we have
lST

From

.-.

the similar triangles

ONP,

NP ON M N ON
NP = y, ON = M N = VON N T =
ON = 2 a - x
:

But

oj,

V (2 -),

Hence
is

.y x
:

y.

::

V(2 a

x)

...

x 2 a
:

x.

(1)

the required equation.

The equation

Second Method.
parabola if

8 ax

is

of the tangent line to the

Art. 65, (2)

+
.

SX

2a
s

of a line passing through the origin


line is
this
to
pendicular

The equation

Combining these equations

so as to eliminate

s.

and per

we have

for the equation of the locus.

This curve was invented by Diocles, a Greek mathematician


of the second century, B.C.,

and called by him the cissoid from

HIGHER PLANE CURVES.


word meaning

a Greek

It

"ivy."

193

was employed by him

in

solving the celebrated problem of inserting two mean propor


tionals between given extremes, of which the duplication of
the cube

is

PO^) =
0)
= OP = M K = OK - OM/.
OK = 2 a sec and OM = 2 a cos
r = 2 a (sec
cos 0),
r = 2 a tan 6 sin

From

the figure (OP,

we have
But

(r,

also r

hence

or
is

a particular case.

To deduce the polar equation.

2.

the polar equation of the curve.

SCHOL. Solving

(1)

with respect to

y,

we have

=
x

2 a

An

inspection of this value shows


(a) That the cissoid is symmetrical with respect to the
X-axis.

That x = and x = 2 a are the equations of its limits.


That x = 2 a is the equation of a rectilinear asymp

(b)
(c)

tote (SS

).

151. To duplicate the cube, by the aid of the cissoid.


Let OL, Fig. 61, be the edge of the cube which we wish to
Construct the arc BO of the cissoid, CO = a
duplicate.

CD = 2 CA =
B draw BO
LR intersecting BO in R.

being the radius of the base circle. Lay off


2 a and draw DT intersecting the cissoid in

and

at

Then

LR

erect the perpendicular


is the edge of the required cube

for the equation

of the cissoid gives

hence

HB =
2 a

The

HB = y, OH =

ig
PL _L

(since

- x).

CDT and HBT


CD CT HB HT.

similar triangles
:

::

give

x,

and

HT =

PLANE ANALYTIC GEOMETRY.

194
But

CD =

CT by
.-.

construction

hence

HT in the value of HB above gives


HB = 2QH3 hence HB = 2 OH
HB
triangles OHB and OLE are similar hence
HB OH LE OL
HB OH ::LK ::OL
HB = 2 OH hence LB, = 2 OL whence the
2

This value of

The

::

.-.

But

struction.

152.

HB = 2 HT

HT = HB

THE WITCH.

FIG.

62.

con

HIGHER PLANE CURVES.


The witch
nate

the locus of a point

is

011

DP

195

the produced ordi-

of a circle, so that the produced ordinate


the diameter of the circle
as the ordinate

DM

OA

DA

outer segment
It

may

to

is

to the

is

of the diameter.
of a point P
its foot

also be defined as the locus

linear sine

DP

DM of an angle

from

at a distance

011

the

equal

to twice the linear tangent of one-half the


angle.

To deduce the rectangular equation.

1.

From

First Method.

DP =

y,

we have

(2

- x),

y:2a:: V(2 a

x)

x 2a
:

x.

the required equation.

is

Second Method.

Let

But a
(1

(1

+ cos

hence

or,

MCO =

then by definition

= 2 a \ //a ^ ~ cos ^
V a (1 + cos 0)
- cos 0) = a - a cos = OC DC = OD == x, and
= a + a cos = OC + DC = OD = 2 a - x
0)
u = 2a
== 2

?/

of generation,

DA = 2 a
hence

mode

::

But

the

DP OA DM DA
OA = 2 a, DM = VOD DA = Vx

a tan 2

squaring

= ~2 a

This curve was invented by Donna Maria Agnesi, an Italian


mathematician of the eighteenth century.
SCHOL. Solving (1) with respect to y, we have

Hence

(a)

=t 2 a

the witch

is

x
J-~
V 2a

symmetrical with respect to the

X-axis.
(b)
(6-)

x
x

= and x = 2 a are the equations of its limits.


= 2 a is the equation of the rectilinear asymptote

SS

PLANE ANALYTIC GEOMETRY.

196

EQUATIONS OF THE FOURTH DEGREE.


THE CONCHOID.

153.

The conchoid is the locus generated by the intersection of


a circle with a secant line passing through its centre and a
fixed point A as the centre of the circle moves along a fixed

OX.
As the

line

intersection of the circle and secant will give two


points P, P, one above and the other below the fixed line, it
is evident that during the motion of the circle these points
will generate a curve with two branches.
is called the SUPERIOR BRANCH

MBM

FERIOR BRANCH.

(= OB)

is

called the

the DIRECTRIX

1.

The upper branch


the lower, the IN

The radius of the moving circle O P


MODULUS. The fixed line OX is called

the point A, the POLE.

To deduce the rectangular equation.

Let

(x, y),

the intersection of the circle

PP P

and the

HIGHER PLANE CURVES.


secant

and

let

AO P, be any
OA = a.

point of the curve.

The equation

of the circle

The equation

of the line

Making y

= sx

OP

Let

whose centre

AO P

197
b,

at

is

= OB =
(x

0) is

is

(1)

we have

in (1),

00

for the distance

But

00 =

hence
a

= b-

(2)

the equation of the circle.


If we now combine
(1) and (2)
the
s,
resulting equation will express the
relationship between the co-ordinates of the locus generated
by the intersection of the loci they represent. Substituting
the value of s drawn from
in
we have
is

so as to eliminate

(1)

(2),

*V-(P-tf)
is

We

might have deduced

triangles

Draw AT
ATP and O SP

||

PS:SO
e

2/)

(3)

the required equation.

simple way:

<>

this equation in the


following very
to OY.
OX, and
Since the

PT

to

are similar,

::PT:

TA

||

we have

Hence
_

This curve was invented


by Nicomedes, a Greek mathema
who flourished in the second
century of our era.

tician

was employed by him in


solving the problems of the
duplication of a cube and the trisection of an
angle.
It

PLANE ANALYTIC GEOMETRY.

198
2.

To deduce

From

the polar equation.

the figure

we have (AY being the

the pole)

(AP,

PAB)

But

AP =

hence

is

ACT

= a sec

and

0)

(>-,

OP

-t

initial line,

the polar equation of the curve.


SCHOL. Solving (3) with respect to

x,

we have

An

inspection of this value shows


is symmetrical with respect to the
(a) That the conchoid

Y-axis.
(b)
(c)

That y
That y

= b and y =
= gives x =

b are the
-J-

tote.

If

(d)

comes a
(e)

then

cc

-t

equations of its limits.


the X-axis is an asymp

V&

2/

i.e.,

the conchoid be

circle.

If b

(/)

0,

oo,

>

If b

a,

a,

the inferior branch has a loop as in the figure.


coincide and the loop
and
the points

disappears.
(g)

If b

<

a,

the inferior branch

is

similar in form to the

a) is isolated]
branch, and the point
(o,
its
co-ordinates
though entirely separated from the curve,
satisfy the equation.

superior

i.e.,

still

To trisect an angle by the aid of the conchoid.


Let PCX be the angle which we wish to trisect. From C
with any radius as CD describe the semi-circle DAH. From
the point A draw AB J_ to CX and make OB = CD. With
A as a pole and OB as a modulus construct a conchoid on
CX as a directrix. Join H, the intersection of the inferior
bnuich and the circle, with A and produce it to meet the
154.

directrix in

then

CKA =

PCX.

HIGHER PLANE CURVES.

199

FIG.

64.

H and C then from the nature


HK = HC = OB.
From the figure PCX == CAR + CKA
CAK = CH A = 2 CKA
but
PCX = 2 CKA + CKA.
hence
Therefore
CKA = 1 PCX.
For join

of the conchoid

We

might have used the superior branch for the same pur

pose.

155.

THE LIMACON.

FIG.

65.

PLANE ANALYTIC GEOMETRY.

200

The Iima9on is the locus generated by the intersection of


lines OP, CP which are so related that during their revo
lution about the points
and C the angle PCX is always
two

equal to | POX.
1. To deduce the polar equation.
Let
be the pole, and OX the initial
point of the curve, and let

(OP,

From

the triangle

POX)

::

Let

line.

be any

then

(r, 0).

OCP

sin

sin f

::

a sin

TT
Hence

POC, we have

OP OC
i.e.,

OC

<9

sin

sin 1

OPC

0.

From Trigonometry

= 3 sin \
r =

sin

hence

=
is

4 sin 3 \ = (3
a (3
4 sin 2 ^ 0),

+ 2 cos 0)

a (1

4 sin 2 \

0)

sin^ 0;

(1)

the polar equation of the limayon.


2.

To deduce

From

the rectangular equation.

we have

Art. 35,
r

cos

for the equations of transformation from polar to rectangular


co-ordinates.
Substituting these values in (1), we have

ax

Vx
x

or

- ^) =

4-

?/

(*

+y

for the required equation.

SCHOL.

1.

From

the triangle

ODA, we have

OD = OA cos =
From
hence

2 a cos

OP = a + 2 cos 6
OP - OD = DP = a
.

(1)

0.

...

(2)

HIGHER PLANE CURVES.

201

ODA

and the limayon


i.e., the intercept between the circle
of the secant through
is constant and equal to the radius of
the circle.
SCHOL.

If 6

2.

If B
If 6
If

156.

= 0, r = 3 a = OB.
= 90, r = a = OM.
= 180, r = - a = OC
= 270, r = a = OM

THE LEMXISCATA.

The lemniscata

is

the locus generated by the intersection of

a tangent line to the equilateral hyperbola with a perpen


dicular let fall on it from the origin as the point of tangency

moves around the curve.

FIG.
1.

66.

To deduce the rectangular equation.

Since

(a",

?/ ) is a

have, Art. 103, Cor

The equation

point of the equilateral hyperbola,

1,

==

a2

of the tangent line


xx"
yy"

(1)

TP

is,

a 2 ... (2)

Art. 112,

we

PLANE ANALYTIC GEOMETRY.

202

rr

Since the slope of this line

OP

pendicular

is

the equation of the per

is

,*.

0)

and solving for


Treating (2) and (3) as simultaneous
y",

we

x"

and

find
.r

Substituting these values in


-

-+ ff

we have

(1),

(x
2

or

(x

T/

=a

(x

2
.

7/ )

(4)

for the required equation.

This curve was invented by James Bernouilli. It is quadto the square constructed on the
rable, its area being equal
semi-transverse axis

OA.

To deduce the polar equation.


We have Art. 34, (3), for the equations of transformation
x = r cos 0, y = r sin 9.

2.

These values in
2

(4) give

+ sin
=a

6
\r (cos
r4
therefore
r2

or
is

=^

0)J
2
r cos 2

a 2 cos 2

(9

\r* (cos

sin

2
<9)};

0,

...

(5)

the required equation.


a.
1 .-. r
cos
SCHOL. If
0, cos 20
values with
.-. r has two equal
90
2
cos
B
cos
If
45,
<

<

opposite signs.
If
45, cos 2

If 6

45

and

If $

If

= cos 90 =

(9

<135

.-.

0.

.-.

.-.

r is imaginary.

= 135, cos 2 = cos 270 =


- 1
e = 180, cos 2 6 = cos 360
>

=i

0.

a.

An examination of these values of r shows that the curve


formed by the asymptotes of the
occupies the opposite angles
hyperbola.
The curve

is

to both axes.
symmetrical with respect

HIGHER PLANE CURVES.

203

TRANSCENDENTAL EQUATIONS.
157.

THE CURVE OF

This curve takes

= sin

and may be defined


of the

SINES.

name from

its

its

equation

x,

as a curve

whose ordinates are the sines

corresponding abscissas, the latter being considered as

rectified arcs of a circle.

FIG.

To construct the

6?.

Give values to x which differ from


each other by 30, and find from a
"TABLE OF NATURAL
SIXES the values of the
corresponding ordinates.
Tabulating the result, we have,
curve.

Value of x

30

=5=

Corresponding
.52

60

90

^=
=^=

1.04

1.56

120

= i5 = 2.08
6

Value of y

.50

.87

1.00

.87

PLANE ANALYTIC GEOMETRY.


Value of x
150
180

210

Corresponding

Value of y

=~- = 2.60
6
= = 3.14
= 3.66
=

.50

TT

.50

240

~=

4.18

270

= 4.70

1.00

300

= i^ =

-.87

5.22

330
360

= 5.75
=
= 2 = 6.28

.50

TT

a smooth curve
points and tracing
As x may have
locus.
the
have
required
through them, we
of the
the
and
GO
equation
to
yet satisfy
any value from

Constructing

these

the curve itself extends infinitely in


curve, it follows that
abscissas.
the
both
of
positive and negative
the direction

158.

THE CURVE OF TANGENTS.

FIG.

68.

HIGHER PLANE CURVES.


This curve also takes

= tan

its

name from

its

205

equation.

x.

To construct the curve. Give x values differing from each


other by 30 and find from a Table of Natural Tangents the
corresponding values of

Value of x

y.

Tabulating,

Corresponding

we

have,

Value of y

"

30

.57

"

.52

60

1.04

1.56

1.73

"

90

oo

120

= 4-^ = 2.08

"

= 5 - = 2.60

"

1-73

150

.57

180

210

= = 8.14
= 3.66
=

"

TT

.57

"

240

= 4.18

==

1.73

"

270

= 4.70

oo

"

300

330

360

= 5.22
^=
2

TT

"

5.75

6.28

1.73

.57

"

Constructing these points and tracing a smooth curve


through them, we have the locus of the equation.
This curve, together with that of the preceding article,
belong to the class of Repeating Curves, so called because

they repeat themselves infinitely along the X-axis.

PLANE ANALYTIC GEOMETRY.

206
159.

THE CYCLOID.

This curve is the locus generated by a point on the circum


ference of a circle as the circle rolls along a straight line.
The line
is called the BASE of the
cycloid the

OM

point P, the GENERATING POINT the circle BPL, the GEN


ERATING CIRCLE the line HB perpendicular to OM at its
middle point, the Axis. The points
and M are the VERTICES
;

of the cycloid.
1.
To deduce the rectangular equation, the origin
being
taken at the left-hand vertex of tJie curve.
Let P be any point on the curve, and the angle

which the

circle has rolled,

of the circle,

=2

ButOA = ayOB

CK =

a cos 6

AB

and

through
Let LB, the diameter

AP

= CB

CK.

=a AB = PK = a sin AP = y, CB = a,
0,

0,

hence, substituting,

=a9
y = a
x

0.

a.

OA = OB

Then

PCB

a sin

a cos

we have
...

(1)

HIGHER PLANE CURVES.


between these equations, we have

Eliminating

=a

207

V2 ay

cos"

- ^2 ay -if

y
.

vers"

(2)

for the required equation.


SCHOL. An inspection of (2) shows
values of y render x imaginary.
(a) that negative

When

(b)

or 4

TT

a, or 6

TT

a,

or etc.

When

= 0;

vers"

=2

but a vers" 1

TT

a,

hence there are an infinite number

and M.

of points such as
(c)

=a

0,

=2

a,

=a

vers"

TT

OB

but

3 TT a, or 5 ?r a, or 7 TT a, or etc. hence, there are


a vers" 2
an infinite number of points such as H.
2 a are equations of the limits.
and y
(d) y
v
and 2 a there
of y between the limits
(e) For e very value
are an infinite number of values for x.
2. To deduce the rectangular equation, the origin being at the
highest point H,
1

We

have for the equations of transformation


x
TT a
x
OA OB - PK
2
a
AP
B
/
y

=
=

These values in
x
y

But
H,

(1)

=
HK =

+
+

above give

= a (0
= a

a sin

TT)

,\

a cos

the angle through which the circle has rolled from


TT

Hence

=
= H

hence
x = a 6

-\-

a sin

= a (COS ff -

vers"

,.-.

1) }

-+V

2 ay

2
?/

(5)

The
usually attributed to Galileo.
With the exception of the conic sections no known curve
The fol
possesses so many useful and beautiful properties.
invention of this curve

is

lowing are some of the more important

PLANE ANALYTIC GEOMETRY.

208
1.

2.

3.

HDB = a
= 3 HDB = 3
OPHM = 4 HB = 8 a.

Area OPHDB O
area
Area of cycloid OHMO
Perimeter

TT

TT

a2

If two bodies start from


any two points of the curve
curve
being inverted and friction neglected), they will
(the
reach the lowest point
at the same time.
4.

5.

point

body rolling

down

this curve will reach the lowest

in a shorter time than if

it

were to pursue any other

path whatever.

SPIRALS.
160. The SPIRAL is a transcendental curve generated by a
point revolving about some fixed point, and receding from it
in obedience to some fixed law.

The portion of the locus generated during one revolution of


the point is called a SPIRE.
The circle whose radius is equal to the radius-vector of the
generating point at the end of the first revolution
the MEASURING CIRCLE of the spiral.
161.

THE SPIRAL OF ARCHIMEDES.

FIG.

70.

is

called

HIGHER PLANE CURVES.


This spiral

209

the locus generated by a point so moving


its radius-vector to its vectorial
angle is

is

that the ratio of

always constant.

From

the definition,

we have

=c
r = c

hence
is

(1)

the equation of the spiral.

To construct the

spiral.

Assuming values for and finding from


ing value for r, we have
Values of

45

(1)

the correspond

Values of r

Corresponding

=^

^c

90=^
4
135

= ^L

180

=<*

270

^
=^

360

=2

225

_^ c
4

3*

..

T"

Constructing these points and tracing a smooth curve


through them, we have a portion of the spiral.
Since
Since

number

=
=

gives r

0, the spiral passes through the pole.


gives r
makes an infinite
oo, the spiral
of revolutions about the
pole.
(9

oo

Since
2 TT gives r
the measuring circle.

=2

IT c,

OA (=

IT

c)

is

the radius of

PLANE ANALYTIC GEOMETRY.

210
162.

THE HYPERBOLIC

This curve

SPIRAL.

the locus generated by a point so moving that


the product of its radius-vector and vectorial angle is always
constant.

From

is

the definition
r

we have
c,

or
for the equation of the spiral.

FIG.

71.

To

construct the spiral.


Giving values to 0, finding the corresponding values of

we have
Values of

Corresponding

Values of r
oc

45

=Z

"

r,

HIGHER PLANE CURVES.


Values of

135=
180

211

Values of r

Corresponding

7T

7T

"

7T

7T

225

TT

4
270

OTT

=-TT

"

315

GTT

=-TT

"

360

TTT
"

TT

oo

Constructing the points we readily find the locus to be a


curve such as we have represented in the figure.
oo there is no point of the spiral
Since 6
gives r

corresponding to a zero-vectorial angle.


Since
oo gives r
0, the spiral makes

an

of revolutions about the pole before reaching


Since 6
2-n- gives

infinite

number

it.

c is

the circumference of the measuring circle.


Let P be any point on the spiral then

SCHOL.

(OP,

POA) =

(r, 0).

and OP as a radius describe the arc PA.


circular
r 6, and from (1) c
r
Arc PA
measure,
By
hence
Arc PA
c

With

as a centre

i.e.,

the arc of any circle between the initial line and the

spiral is equal to the circumference of the

measuring

circle.

PLANE ANALYTIC GEOMETRY.

212

THE PARABOLIC SPIRAL.


This spiral is the locus generated
by a point so moving
that the ratio of the square of its radius-vector to its
vectorial
angle is always constant.
From the definition we have
163.

r*

or,

(1)

for the equation of the


spiral.

FIG.

To construct the
Values of

45

72.

spiral.

Corresponding

Values of r

!T

4
90

2*
4

135

^
4

180

7T

((

V2 C7T
V3c

HIGHER PLANE CURVES.


Values of

Corresponding

225

270

= ^T

3i5

= IJL

360

=2

213

Values of r

-JV5C

4
7C7T

oo

00

Constructing these points and tracing a smooth curve


through them we have the required locus.
Since
gives r
0, the spiral passes through the pole.
Since
oo gives r
oo, the spiral has an infinite num
ber of spires.

=
=

=
=

THE LITUUS

or

This curve has for

its

164.

r2

TRUMPET.
equation

c,

FIG.

73.

PLANE ANALYTIC GEOMETRY.

214
If

0,

if

The Logarithmic

165.

This spiral
the ratio of
vector

oo

is

is

its

oo,

an asymptote to

initial line as

0.

its infinite

This curve has the


branch.

/SJjiraZ.

the locus generated by a point so moving that


vectorial angle to the logarithm of its radius

Hence

equal to unity.

or passing to equivalent
r

a*

numbers (a being the


.

(1)

for the equation of the spiral.

To construct

the spiral.
r

is

Let a

2,

then

= 29

the particular spiral

we wish

to construct.

base),

we have

HIGHER PLANE CURVES.


Values of 6

215
Values of r

Corresponding

= 57.3
2 = 114. 6
3 = 171.9
4 = 229. 2
1

2
4

"

16
"

CO

<

- 1

= - 57.3

.5

-2=-114.6
- 3 = - 171.9
_ 4 = - 229. 2

.25

.125
.062

oo

smooth curve traced through these points

will

be the

required locus.

Since
a, it

1 whatever be the assumed value of


gives r
follows that all logarithmic spirals must intersect the

initial line at a unit s

Since

number

distance from the pole.

oo
the spiral makes an infinite
gives r
of revolutions without the circle whose radius
1.

GO

OA =

Since 6

r =

GO gives
0, the spiral
of revolutions within the circle

number

makes an

OA

infinite

before reaching

its pole.

EXAMPLES.
1.

Discuss and construct the cubical parabola


B

X
= ~rp*

2.

What

the polar equation of the Iima9on, Fig, 65, the

is

pole being at

Ans.

= 2 a cos -

0.

3
3.

cata

Let
is

OF

= OF = a Vf

Fig. 66.

Show

that the lemnis-

the locus generated by a point so


moving that the

PLANE ANALYTIC GEOMETRY.

216

product of its distances from the two fixed points F,


constant and

Discuss and construct the loci of the following equations


4.
5.

6.

7.

= tan

= cos

2/

a;

8.

is

12.

y.

a3

=x

axy.

x.

13.

jcl -j-

= sec*.

14.

4 + |r =

y?

a2

= sin y.

15.

= cot x.

16.

1.

l.

&3

sin 2

0.

<?/

sin 2

9.

10.

= cosec

= 3x ~ l

20.

a?

=a

sin 3 -

Q
2

?/

-4- a-?/

18.

r 2 sin 2

20

1.

o
2

1 rv

-|

19.

1.

= -1 1

~T~ Sill

C
.

sin

Discuss and construct the locus of the equation

96 a 2 ;/2

+ 100 ciV

J- V(ic
21.

x3

no
.

17.

ic.

Show

asymptotes
Ex. 20.

that y
of the

6 a) (a

^ x are
locus

x^

+ 6 a)

or
(x

8 a) (x

+ 8 a).

the equations of the rectilinear


represented by the equation of

SOLID ANALYTIC GEOMETRY,

PART

II.

CHAPTER
CO-ORDINATES.

I.

THE TRI-PLANAR SYSTEM..

determined when
we know its distance and direction from three planes which
intersect each other, these distances being measured on
166. The position of a point in space

lines
it

is

is

Although
drawn from the point parallel to the planes.
immaterial in principle what angle these planes make

with each other, yet, in practice, considerations


and simplicity have made it usual to take
They are so taken in what follows.
angles.
Let XOZ, ZOY, YOX be the CO-ORDINATE
Let OX,
secting each other at right angles.

CO-ORDINATE AXES and 0, their

intersection,

of convenience

them

at right

PLANES inter
OY, OZ be the
the ORIGIN of

CO-ORDINATES.
- XYZ.
Let P be any point in the right triedral angle
the
we
know
lengths
Then P is completely determined when

and directions of the three lines PA, PB, PC let fall from
this point on the planes.
As the planes form with each other eight right triedral
which satisfy
angles, there are evidently seven other points
the condition of being at these distances from the co-ordi
nate planes.

The ambiguity

is

217

avoided here (as in the case

SOLID ANALYTIC GEOMETRY.

218

of the point in a plane) by considering the directions in which


these lines are measured.

Assuming distances

to the right of

YOZ as positive,

distances

to the left will be negative.

4-Z

A/

-Y

-X

4-X

Assuming distances

XOY

6o*;e

as positive, distances

will be negative.

in front

Assuming distances

o/XOZ

as positive, distances ^o

^Ae rear will be negative.


r
Calling x 7/ z (= BP,
,

AP, CP, respectively) the co-orc?ithe FIRST ANGLE, we have the follow

wa^es of the point P in


in the
ing for the co-ordinates of the corresponding points

other seven

SECOND ANGLE, above

XY

plane, to left

of XZ plane, (- x y z ) P 2
THIRD ANGLE, above XY plane, to
r
of XZ plane, (- x - y z ) P 8

YZ

plane, in front

left

YZ

plane, in rear

CO-ORDINATES.

219

XY plane, to right YZ
P
y
(x
below XY plane, to right YZ

plane, in

FOURTH ANGLE, above

XZ

rear of

plane,

front of

XZ

FIFTH ANGLE,

plane, (x

SIXTH ANGLE, below

XY

PS-

plane, to left

YZ

plane,

in

plane, in front

P6
)
y
plane, (
SEVENTH ANGLE, below
plane, to left YZ plane, in
f
z ) P7
x
rear of XZ plane, (
y
EIGHTH ANGLE, below
plane, to right YZ plane, in rear

of

XZ

XY

of

XZ

plane, (x

XY
- P

i/,

8.

EXAMPLES.
1.

In what angles are the following points

(1, 2,

3),

(-

1, 3,

2),

(-

1,

2,

4), (3,

2, 1).

2.
State the exact position with reference to the co-ordi
nate axes (or planes) of the following points
:

(0, 0, 2),

- 1,
- 1),

0), (0,
(5, 1,

3.

(0),

2, 1, 2), (3, 1, 0), (3,


(3,

(1, 1,

0,

1),

(1,

2,

3),

1, 2), (2, 0, 3),

(0,

0,

2),

((4,

1,

1,

2,

2),

1).

In which of the angles are the X-co-ordinates positive ?


In which of the angles are the Y-co?

In which negative

ordinates positive ?

In which are the Z-co-ordinates negative?

167. Projections. The projection of a point on a plane is


the foot of the perpendicular let fall from the point on the
Thus A, B, and C, Fig. 75, are the projections of the
plane.
point P on the planes XZ, YZ, XY, respectively.

The

projection of a line of definite length on a plane is the


on that plane.
OC, Fig. 75, is the projection of OP on the
plane.

line joining the projections of its extremities

Thus
The projection

XY

of a line of definite length on another line


is that portion of the second line included between the feet of
the perpendiculars drawn from the extremities of the line of
definite length to that line.

SOLID ANALYTIC GEOMETRY.

220

Thus OM, Fig. 75, is the projection of OP on the X-axis.


The projections of points and lines as above de
;NOTE.
Unless otherwise stated, all projections
fined are orthogonal.
will be so understood in what is to follow.
168.

To find the length of a

line joining two points in space.

P"

FIG.

76.

be the given points.


P (a? tf, ) and
and
the
be
L (=P
required length. Draw
N
Join
OX.
to
NB
to
OY
and
CD
NA
to OZ
to
NO.
P
M
draw
and
and C
We observe from the figure that L is the hypothenuse of a
are P M and
right angled triangle whose sides

Let
Let

V"

(z",

y" ,

*"}

P"C

P")

||

||

||

||

P"M.

Hence

= NO =

but

=
...

(P"C

CO- OR DINA TE S.

= 0,

COR. If x
0, if
with the origin and
...

= 0,

2
V*"

221

then the point

2
y"

2
s"

...

coincides

(3)

expresses the distance of a point from the origin.

Given the length and the directional angles


of a line
joining any point with the origin to find the co-ordinates of the
169.

point.

The Directional angles of a line are the angles which


makes with the co-ordinate axes.
Let
its

(x, y, z),

Fig. 75, be

distance from the origin.

the line

OP = L will be
POX, POY, POZ = a, fa y,

point, then

any
Let

respectively.

OM, ON, OR

Since

on X, Y,

(== x, y, z) are the projections of

Z, respectively,

y
z

OP

we have

= L cos u
= L cos
= L cos y

ft

...

(1)

for the required co-ordinates.

COR.

Squaring and adding equations

+y + =L
x* + y + * = L

x*
but

hence

That

cos u -f cos
is,

the

/? -f-

(1),

we have

2
-f- cos
(cos
Art. 168 (3)

cos 2 /

sum of the squares of

ft

-f cos

y)

(2)

the directional cosines


of a

space line

is equal to
unity.
SCHOL. The directional angles of
any line, as P
Fig. 76,
are the same as those which the line makes
with three lines
drawn through 1J/ to X, Y, Z. The
on
projections of P
- x f if tf, z
three such lines are
Art. 168; hence
P",

||

x"

x"

y"
"

P"

x = L COS a
- y = L cos
z = L cos

ft

"

_^

"1

...

(3)

SOLID ANALYTIC GEOMETRY.

222

EXAMPLES.
Required the length of the
points
1.

(-

(1, 2, 3),

(3,

- 2,

following

(0, 3, 0), (3,

(0, 0, 0), (2, 0, 1).

Ans.
5.

0), (2, 3, 1).

(0, 4, 1),

V27.

(-

2,

1,

^ws.
6.

1, 0).

Ans.
7.

joining the

V14.

^tws.
3.

4.

2, 1, 1),

Ans.
2.

lines

(1,

-/5.

-2).
V38.

2, 3), (3, 4, 6).

Ans.

5.

Find the distance of the point

(2, 4,

3)

7.

from the origin

also the directional cosines of the line.


8.

line makes equal angles with the co-ordinate axes.


are its directional cosines ?

What
9.

Two

What

is

of the directional cosines of a line are

If (x

10.

Vf

and

the value of the other ?


f
,

and
show that
(a;",

y",

z")

are the co-ordinates of the

extremities of a line

x"

+
2

y"

z
"z"\
z"

2-J

are the co-ordinates of its middle point.

THE POLAR SYSTEM.


170. The position of a space point is completely determined
when we know its distance and direction from some fixed point.

For a complete expression of the direction of the point it is


The angles
necessary that two angles should be given.
usually taken are

The angle which the line joining the point and the
makes with a plane passing through the fixed
the line join
and
2d, The angle which the projection of
point
line in the
fixed
a
with
makes
that
on
the
plane
points
ing
1st,

fixed point
;

plane.

CO-ORDINA TES.

FIG.

223

77.

Let
be the fixed point and P the point whose
position we
wish to determine. Join O and P, and let XOY be
any plane
passing through 0. Let OX be a given line of the plane
XOY. Draw PB 1 to XOY and pass the plane PBO through
PB and OP. The intersection OB of this plane with XOY
will be the projection of

BOX

and the distance

(go)

OP on XOY. The angles POB (0),


OP (r), when given completely de

termine the position of P. For the angle cp determines the


determines the line OP in that plane,
plane POB, the angle
and the distance r determines the point P on that line.
This method of locating a point is called the POLAR SYS
TEM. The angles
and qo are called VECTORIAL
and

ANGLES,

the distance r

The point
terms of

called the

RADIUS VECTOR

when written (r, 9,


POLAR CO-ORDINATES.

P,

its

is

<p),

is

of the point.

said to be expressed in

It is evident
<P,

and

may

all

to 360 to
and
by giving all values from
values from
to oo to r that every point in
space

be located.

171. Given the polar co-ordinates of a point to


find its rec
tangular co-ordinates.
Draw OY 1 to OX and in the plane BOX draw OZ 1 to
;

SOLID ANALYTIC GEOMETRY.

224

OY

and OX, and

Draw

BA

||

to

(OA, AB, BP)

From

From

OB =

From

(x,

BOP, we have
sin

6.

ABO, we have

= OB

r cos

cos

.:

cp.

= r cos

the same triangle

y
y
x

cos

cp.

we have

= OB sin
= r cos 6 sin
qo,

= r cos 6 cos
=
r cos 9 sin
y
z = r sin

Henee

co-ordinate axes.

then, Fig. 77,


y, z) are the rectangular co-ordinates of P.

=r

the triangle

But

the triangle
z

OX, OY, OZ be the

let

OY

(p.

cp

cp

...

(1)

express the required relationship.


Con. If P (x, y, ) be the co-ordinates of any point on a
locus whose rectangular equation is given then equations (1)
are evidently the equations of transformation from a rectangu
lar system to a polar system, the pole being coincident with the
origin.

y,

Finding the values of


we have

r,

and

cp

from

(1) in

terms of x and

r= Vx + if +
= tan+
= tan2

(9

for the equations of transformation from a polar system


rectangular system, the origin and 2)ole being coincident.

to

225

CO-ORDINA TES.
EXAMPLES.
Find the polar co-ordinates of the following points
1.

2.

3.

(2,1,1).

(V3,

1,

(10,2,8).

4.

2 V3).

(3,

1, 4).

Find the rectangular co-ordinates of the following


5.

(5,

7.

30, 60).

(6,=,

Find the polar equations of the following


and origin being coincident

surfaces, the pole

9.

10.

+ ^2 + = a
z + sx + ty - c = 0.

X2

Ans. r

,2

sin

+ 5 cos

Find the directional cosines of the


ing pairs of points
11.

(1, 2,

12.

(4,

(-

1, 3, 2).

and
15. If (V, /,
)
the point
space points, show that
(z",

mx"

g>

cos

-f

sin

a.

qo

lines joining the follow

1), (3, 2, 1).

1, 2),

cos

+ nx

divides the line joining


n.
each other the ratio
:

14.

(0, 2, 0), (3, 0, 1).

1,

5), (4, 5, 6).

be the co-ordinates of two

")

+ ny
m+w

them

(2,

my"

7wT+ %

y",

13.

into

+ nz
m+n

mz"

two parts which bear to

SOLID ANALYTIC GEOMETRY.

226

CHAPTER

II.

THE PLANE.
To deduce the equation of the plane.
Let us assume as the basis of the operation the following
172.

property
If on a perpendicular to a plane two points equidistant from
the plane be taken, then every point in the plane is equidistant
:

these two points,


equally distant.

from

and any point not

in the plane

is

un

FIG.

Let
it

ABC

in E.

Draw

be any plane.

Produce

OR

until

the plane is equally distant


be any point of the plane let
;

OR 1

to

ABC, and meeting

RR = OR = p.
and R
from

Every point
Let

ON, MN,

MR

in

(x, y,*,)

the co-ordinates

THE PLANE.
of

R =

d,

e,

From

Then from

/, respectively.

PR

==

- xY + (e-

(d

the same article, equation

OF =
2

xs

227

(3),

+f+*

we have

Art. 168, (2),

yY + (/- zY
we have

2
5

hence, by the assumed property,


(d

- xY + (e-

yY

+ (/- *) = x + y + *
2

2
-

Simplifying this expression, we have


6 + f
+ ey +fz = d*+
*

dx

/1X
(1)

...

for the required equation.

To find the equation of a plane in terms of

173.

pendicular

to it

from

the origin

and

the

per

the directional cosines of

the perpendicular.

be the directional angles of the perpendicu


Since ON, MN,
lar OR (=2p), Fig. 78.
(= d, e, f)
the projections of OR on the co-ordinate axes, we have (Art.

Let

/8,

and

169, (1)

MR

= 2p cos
e = 2^ cos

(3

Substituting these values in


cos2 ft H~ cos2 Y
that cos 2

cc

cos

?/

cos

for the required equation.


EQUATION of the plane.

Since

(1),
1?

(1)

Art. 172,

+ z cos 7 = p

^8

and remembering

we h ave

Equation

OR = 2p = V^ +

cos

...

cos 7

2p

+/

(2) is

called the

(2)

NORMAL

2
,

equations (1) give


e

ANALYTIC GEOMETRY.

.SOLID

228

Substituting these values in

we have

(2),

e
~

for the equation of the plane expressed in terms of the co-or


dinates of a point on the perpendicular to it from the origin

and the perpendicular.


COR.

1.

If

we have

in (2),

+ y cos + z cos y =

x cos

ft

(4)

for the equation of a plane through the origin.

COR.

2.

= 90,

If

y cos
is

= 0,

cos

(3

+ z cos 7

hence

=p

(5)

the equation of a plane 1 to the YZ-plane.


90, we obtain similarly
ft

If

x cos

+ z cos 7 = p

for the equation of a plane


If Y
90, then

(6)

to the XZ-plane.

J_

x cos
is

the equation of a plane

COR.

3.

= 90

If

cos

?/

and

1L
.--cos

=p

/?

(7)

to the XY-plane.

(3

= 90,

...

then

(8)

is

the equation of a plane

to

YZ

and XZ, and hence

XY.
Similarly,

we

find

= --*
cos

(10)

COS
for the equations of planes

to
||

XZ

and

YZ

respectively.

||

to

THE PLANE.
COR.

If

4.

and

in (8), (9),

...

229

(10),

then

(11)

are the equations of

XY, XZ, and YZ,

respectively.

To find the equation of a plane in terms of

174.

in

its

tercepts.

OB =

OA

Let, Fig. 78,


=a,
perpendicular to the plane

OC

b,

OE (=p)

Since

c.

ABC, we have from

the right

is

tri

OEA, OEB, and OEC

angles

COS

...

0080=|
COS 7

(a)

=
J

Substituting these
reducing,

we have
*
a

values

+ -|b + *c = 1

normal

the

in

and

(1)

for the required equation.


Equation (1)
METRICAL EQUATION of the plane.

175.

equation

is

called the

Every equation of the first degree between

SYM

three vari

ables represents a plane.

The most general equation of


three variables is of the form

the

-f

By

-f

C*

=D

(1)

Dividing both members of this equation by


we have

VA +
2

degree between

ABC
Ax

first

B2

+C

-2
2

A2

VA ~+ B + ni
C
^/A22

~"~

+B +C
2

T^22

*
y*

VA

-|-

B2

+C

VA + B + C
2

2
,

SOLID ANALYTIC GEOMETRY.

230

of Art. 173, we see that the co


the directional cosines of some
are
efficients of the variables
co-ordinates of one of its
the
of
terms
in
line

Comparing

(2)

with

(3)

expressed

and that the second member measures the distance of


the
a plane from the origin hence (2) and therefore (1) is

points,

equation of a plane.

To find the equations of the traces and the values of


the intercepts of a plane given by its equation.
176.

FIG.

ABC

Let

be the plane and

Ax

79.

let its

+ By + C2 = D.

To find the equations of the

1.

equation be

traces

AB, BC, AC.

the given plane with


traces are the intersections of
their equations, we
the co-ordinate planes hence, combining

The

have

Ax

+ By + Cz = D |
z

Ax

Ax

/<

Ax

+ B^ =

D<

Trace

011

XY (AB)

.(1)

+ By + C = D |
Cz

=D

")

Ax + Cs = D
I

.
j>

r
,

Q,,

__

j)^

Trace on

X Z (AC)

Trace on YZ (BC)

(2)

... (3)

THE PLANE.

231

intercepts OA, OB, OC.


The points A, B, C are the intersections of the given plane
with the co-ordinate planes taken in pairs ; hence, combining

To find the

2.

we have

their equations,

A*

+ By + C* = D
(4)
-nL

y-Q

-D
.

._ 00
COR.

If

the

=
cept = OB

oo

(5)

(6)

plane is perpendicular to XZ its Y-inter0.


Making B
hence, equation (5), B

Ace

in the general equation,

we have

+O=D

(7)

a perpendic
(7) and (2) are the same equations; hence,
ular plane and its trace on the plane to ivhich it is perpendic
ular have the same equation.

But

cos 7
177. If x cos a. -j- y cos ft -|p be the normal equa
a plane, then x cos u -\- y cos (3 -j- z cos 7
p -^ d is the

tion of

equation of a parallel plane at the distance d from it.


For the directional cosines of the perpendiculars are the

same hence, the perpendiculars are coincident hence, the


The distance of the planes apart is equal
planes are parallel.
to the difference of the perpendiculars drawn to them from
the origin but this difference is p -j- d
p i.e., J- d. Hence,
;

the proposition.
COR. If (V, ?/,

from the origin is p


X COS
_J_ d

be a point in the plane whose distance


d then

-]-

-|-

if COS

(3

COS 7

(1)

SOLID ANALYTIC GEOMETRY.

232
is its

distance from the parallel plane whose distance from


From equations (a), Art. 174, we have
is p.

the origin

=-

cos

a
hence

+ cos

cos 2

These values in

(1)

cos

+ cos

j8

cos 7

P;

^& + %
v

-~ -

1-

give

for the expression of the distance of a point


which is given in its symmetrical form.

from a plane

Let the student show that the expression for d becomes

_A^Vy

+ Cz:_-^
+B +C

(3)

of the plane is given in its general form.


the significance of the double sign in (1), (2), and

when the equation

What

is

(3)?

178.

the equation of a plane which passes through

To find

three given points.

Let (V, y

),

(x",

y" ,

*"),

we seek

Since the equation

(*"

is

y",

which A, B,

C,

")

^ the given points.

that of a plane,

Ax + By + Cz = D

in

it

must be

(1)

are to be determined by the conditions

imposed.
the co
Since the plane is to contain the three given points,
its
equation hence,
ordinates of each of these must satisfy
the following equations of condition
;.

Ax
Ax"

Ax

+ B// + C.~ = D
=D
+ B/ +
= D.
C
+ By +
C,~"

"

"

"

These three equations contain tlie/ow unknown quantities


from the equations the values of A,
A, B, C, D. If we find

THE PLANE.
B,

in terms of

233

and the known quantities, and substitute

term of the resulting equation will


(1), each
Let
as a factor.

these values in

D
A=

contain

AT>,

Substituting in

= B D,

(1),

= CD be

the values found.

we have

A Daj + B D?/ + C Vz = D.
A a + B y + C s = 1 ...

...

is

(2)

the required equation.

179.

The preceding discussion has

every equation of

the

first

elicited the

fact that

degree between three variables


It remains to be shown that every

represents a plane surface.


equation between three variables represents a surface of some
kind.

Let

=/(,

y)

1)

Since
be any equation between the three variables (x, y, z).
x and y are independent, we may give them an infinite number
For every pair of values thus assumed there is a
of values.
These values in (1) give the corre
point on the XY plane.
sponding value or values of z, which, laid off on the perpen
dicular erected at the point in the XY plane, will locate one

But the number


or more points on the locus of the equation.
of values of z for any assumed pair of values of x and y are
necessarily finite, Avhile the number of pairs of values which
be given x and y are infinite
a surface of some kind.

may

hence

(1)

must represent

If

-/(*,

q>

y)

(x, y)

(2)

be the equations of two surfaces, then they will represent their


For these equa
line of intersection if taken simultaneously.
tions can only be satisfied at the

same time by the co-ordinates

of points common to both.


Hence, in general, two equations
between three variables determine the position of a line in space.

SOLID ANALYTIC GEOMETRY.

234

If

=f(x,y)

=
=

<P

(^ y)
(x, y}

be the equations of three surfaces, then they will represent


simul
their point or points of intersection when considered as
Hence, in general, three equations
variables determine the positions of space points.

taneous.

letiveen

three

EXAMPLES.
Find

4.

11.

the traces

and intercepts

of the following planes:

The

from
directional cosines of a perpendicular let fall

of the
required the equation
the origin on a plane are |
o
o o |;
== 4.
plane, the length of the perpendicular
,

An,

5+^ + f-l.

the plane whose intercepts are


Required the equations of
as follows

12.

1,2,3.

13.

2,

14-

1,

15.

3.

i,,
-

1,

-2.
"

4.

of
the equation of the plane, the equations
?
4
z
x
4
and
3 y
-jwhose traces are x
4.
z
3 ?/
a
16.

What

is

+ =

THE PLANE.
The

17.

co-ordinates of the projection of a point in the


2ov. the
plane are (2, 1) required
3y-}-2z

plane x

235

XY
XY plane.

the distance of the point from the

Ans.

f.

Write the equations of the planes which contain the follow


ing points

(-

18.

(1, 2, 3),

19.

(4, 1, 0), (2, 0, 0), (0, 1, 2).

20.

(0, 2, 0), (3, 2, 1),

21.

(2, 2, 2), (3, 3, 3),

1, 2,

1), (3, 2, 0).

((-

1, 0, 2).

!,-!,-

1).

Find the point of intersection of the planes


23. 2 x - y + z = 10.
22. a- + ^ _ z = 4.
x + ?/-2 = 3.
2^-33 + ?/ = 10.
2 x - 4 y + 5 = 6.
x -f T/ - 2 = 2.
24.

Find the distance of the point

(2, 1, 3),

from each of the

planes
25. .x cos

26.

27.

+ y cos

60

+
x

+ ^2 + 3 z = 4.

+z

60

3y

= 9.

cos 45

8.

--- + - =

325

28.

1.

Find the equation of the plane which contains the


and is parallel to the plane x
2 ij -\- z
6.
Reduce the following equations to their normal and sym
metrical forms
29.

point

(3, 2, 2)

30.

2x

3y

+z=
2

qo

o<6.

31.

1
,

~\-

cc

+2y

= ~.

o.

33. If s, s
represent the sides of the triangle formed by
the traces of a plane, and a, b. c represent the intercepts,
2
2
2
s
2 (a 2 -f b 2
s
c ).
show that s 2
s"

"

236

SOLID ANALYTIC GEOMETRY.

CHAPTEE

III.

THE STRAIGHT
180.

To deduce

LINE.

the equations of the straight line.

straight line in space is determined when two planes


which intersect in that line are given. (See Art. 179.) The
equations of any two planes, therefore, may be considered as

The

when taken simultaneously. Of the


of pairs of planes which intersect in and de
termine a space line, two of its projecting planes
that is,
two planes which pass through the line and are perpendicular
representing a space line
infinite

number

two of the co-ordinate planes


give the simplest equations.
For this reason two of these planes are usually selected.

to

THE STRAIGHT LINE.


Let
then

237

PBM

its

be the plane which projects a space line on XZ,


equation will be of the form

x
in which s = tan

= sz

a ( see Art. 176, Cor.)


and a
OA.

-j-

ZBP

Let P B
be the plane which projects the line on YZ,
then its equation will be

in

y=t*+b,

which t
tan ZB P and b
OA.
But the two planes determine the line
x

=
y

hence

= sz + a

are the required equations.


COR. 1. If a
and b

= sz
= tz

0,

then

which pass through the


0, we have

are the equations of a line


and t
COR. 2. If s

for the equation of a line

origin.

to the Z-axis.
Since equations (1) express the relation
existing
between the co-ordinates of every point on the space line, if
we eliminate Z from these equations we obtain the immediate

COR.

||

3.

relation existing between x and


y for points of the line. But
this relation is evidently the same for all
points in the pro
and therefore for
jecting plane of the line which is 1 to

XY

trace on

its

on

XY

XY.

But the trace

hence, eliminating,
tx

is

the projection of the line

we have

= bs

at
(4)
for the equation of the
projection of the line

sy

on XY.
have found, Art. 169, Schol., for the length of a
joining two points the expression

181.
line

We

T.-^-y _
COS a

y"

-j/

COS

(3

*"-:.
COS 7

SOLID ANALYTIC GEOMETRY.

238

L and

Eliminating
ordinates of

letting

cos

cos
for the

182.

z"

x",

y",

(=

x, y, z)

any point on the line, we have


x _ y
x
y _ z

To find where a

x-jx

cos 7

(3

SYMMETRICAL EQUATION

be the co

of a straight line.

line given by the equations

of

its

projections pierces the co-ordinate planes.

Let
1.

==
__

? I be the equations of the


fz-\-0 )
y
To find where the line pierces the XY-plane.

The equation

of the

line.

"f

XY-plane

is

Since the point of intersection is common to both the line


its co-ordinates must satisfy their equations.

and the plane,

Hence

= sz
y =

t-Z

-j-

-\-

are simultaneous equations.

So treating them we find

(a, b, 0)

to be the required point.


To find where the line pierces the XZ-plane.
2.

The equation

of the

XZ-plane

is

y-o.
Combining

this with the equations of the line,

for the required point.


To find where the line pierces the XZ-plane.
3.

x
y

= sz
=
tz

-{-

-f-

hence
<),

s
is

the required point.

b y are

simultaneous

we have

THE STRAIGHT LINE.


To find the equations of a

183.

pont.
Let

(x

239
a given

line passing through

be the given point.

Since the line

is

straight its equations are

a)

= te + b

y
in

ft

>

which the constants are unknown.


Since

to pass through the point (#


satisfied for the co-ordinates of

it is

must be

the equations of condition

y ,z

) its

equations

this point

hence

= sz + a
= tz + b \
f

,n\

As the

three conditions imposed by these four equations


in
cannot,
general, be fulfilled by a straight line, we must
eliminate one of them.
Subtracting the first equation in

group (2) from the first in group (1) and the second in group
(2) from the second in group (1), we have
x = s (z
z
- y = (Z - Z

,,

|
}

for the general equations of a straight line


passing through

point.

To find the equations of a

184.

line

passing through two

given points.

Let

(x

As the

fl

),

(x

y",

z")

be the given points.

line is straight its equations are

...
in

which the constants are

As

it is

to be determined.

to pass through (x

= sz
= tz

(i)

),

we must have

SOLID ANALYTIC GEOMETRY.

240

As

it is

to pass
x"

y"

through

=
=

sz"

tz"

(#",

+
+b

y",

z"),

we must have

also

(3)

As these six equations impose four conditions on the line,


we must eliminate two of them. The conditions of the
the line to pass through the
hence we must eliminate the other two.
Elimiting a and b from groups (1) and (2), by subtraction,
we hav6
z )
x
x = s (z
proposition, however, require

two points

-y =

Now, eliminating a and


X -X =

(z

-*

Eliminating

-y"

and

s (Z
t

(z

and

(2)

_
-

4)

(3),

we have

z")

z")

between

from

"

,j

) j

t
j

and

(4)

(5),

we have

for the required equations.

EXAMPLES.
1.

Given the

line

the projection on

x
^

= z 1
=4^ _ 3
*?

-4-

)
r

required the equation of

XY.
Ans.

2.

How

2x

5.

are the following lines situated with reference to

the axes ?

x=2\ y=0\

?/

= 0)

a;

Find the co-ordinates of the points in which the following


lines pierce the co-ordinate planes

x=-z-l)

2x

THE STRAIGHT LINE.


6.

Given

(2, 1,

2), (3, 0, 2)

241

required

The length of the line joining the points.


The equation of the line.
The points in which the line pierces the

(a)
(7>)

(c)

co-ordinate

planes.

Find the equations of the


points

7. (2, 1, 3), (3,


8. (

1,

- 1).

9.

- 1, 2, 3), ( - 1,0, 2).

11.

of 45

which pass through the

lines

The

(2,

10. (1,

1,

1, 0), (3, 0, 0).

(-

2),

projections of a line on XZ and YZ make angles


respectively with the Z-axis, and the line in
(1, 2, 3)

required the equations of


~ ^^ w.
-/

*v

t*/
/y>

y
12.

The

2, 4, 3)
13.

(-

15.

= -4- - V 3 +
V3

vertices of a triangle are (2, 1, 3),


required the equations of its sides.

Is the point (2,

1,

- 2) ?

3)

Assume two

(3,

0,

1),

on the line which passes through

Write the equations of a

NOTE.

them

is

1, 3, 2), (3, 2,

14.

- 3).

and 30

space contains the point


the line.

-2,

1,

line

which

points in the plane

the plane

lies in

the line joining

will be a line of the plane.

Find the equation of a line through


y + z = 4.

(1,

2,

2)

which

parallel to the plane x


16.

Find the point

pierces the plane 3 x


17.

-\-

in

2 y

which the
z

= 4.

line

^
y

+2^ = 3
z + 2 = 0)

Required the equation of the plane which contains the

SOLID ANALYTIC GEOMETRY.

242

Find the point of intersection of the planes


z = 4, x
x + 3y
y + z = 2, 2 x

18.

+ y = 3.

Find the equations of the projecting planes of the


=4
a._2v

19.

line

20.

planes cross ?
z
2y
2, x

Which angles do the following


x -y. + z = 4:,2x +y -3z =

185.

To find the

equations.

Let

- =

1.

intersection of two lines given by their

= sz + a )

y-. + ft|

Mld

s z

y-^ +

a \
f

ft
;
intersection is com
of
the
Since
be the given equations.
point
mon to both lines, its co-ordinates must satisfy their equations.
Hence these equations are simultaneous. But we observe that
there are four equations and only three unknown quantities
the lines
hence, in order that these equations may consist (and
a certain relationship must exist between the con
;

intersect),

which enter into them. To find this relationship, we


eliminate x between the first and third, y between the second
and fourth, and z between the two equations which result.
stants

We

thus obtain
(s

s ) (b

&

(t

(a

-a =
)

for the required equation of condition that the two lines shall
If this condition is satisfied for any pair of as
intersect.
lines the lines will intersect, and we obtain the
co-ordinates of this point by treating any three of the four
which represent them as simultaneous. So treating

sumed

equations
the first, second, and third

we

obtain
a

^>

a
s

for the co-ordinates of the required point.


were prepared to expect
NOTE.

We

a
s

that our analysis

some conditional equation, for in assuming the


it would be an exceptional case
equations of two space lines

would lead

to

THE STRAIGHT LINE.


if

we

243

assumed them that the

so

lines which they represent


Lines may cross each other under
any angle in
space without intersecting. In a plane, however, all lines
except parallel lines must intersect. Hence, no conditional

intersected.

equation arose in their discussion.


186.

To find the angle between

tiuo

lines,

equations, in terms of functions of the angles


make with the axes.

Let

= sz

-f-

given by their

which the

lines

be the equations of the two lines. The


angle under which
two space lines cross each other is measured
by the angle
formed by two lines drawn through some
point parallel to
their directions.

FIG.
.

Let

OB

and

OC

will be their equations.

angle sought.

be two lines drawn through the origin

parallel to the given lines.

Let

81.

cp

Then

The angle between these

(= BOG)

be this angle.

lines

is

the

SOLID ANALYTIC GEOMETRY.

244

which the line BO makes


Let a
y represent the angles
the angle which
and
with X, Y, Z, respectively
P (x y z }
Take
axes.
same
the
with
makes
point
CO
any
on CO and join them by
on OB and any point
,

0",

",

y"

P"

Let

OF - I/,

From

OP"==

the triangle

But Art.

+ TL

IfO
j

P
P

OP".

ff>

168, equation (3)

and

(x"

//2

x"*+

cos

=L

L"

Substituting in
cos

<f

(1),

(*"

- *r

we have
~

<p

Ov

T~^T"

169, (1)

x
x"

y"*

Substituting these values in

But Art.

/-i\

(2)

=x +y +*
=
+^
- xj + (if - yj +
L"

-Lr

L.

2L07-

L2

P"

we have

OP",

COS

and

L",

z")

y",

(x",

a right line forming the triangle

= cos

cos
cos

cos

/?

=L
=
"

L"

(2),

cos

y"

+ cos

/3

cos

/3"

+ cos y

for the required relation.


90
COR. If v

=
+ cos

"

cos

cos

187.

To ymd ^e

^3

cos y
cos

L"

y"

)3",

we have

"

cos

=L
=

?/

",

cos

)8"

+ cos

awr/Ze ^//-ic/i

^<?o

cos

=
y"

cos

y"

(3)

(4)

each
space Zmes maA;e with

the projections
other in terms of functions of the angles which
axes.
the lines make with the co-ordinate

of

be, as

of the lines
in the preceding article, the equations

THE STRAIGHT LINE.


drawn through the origin

(x, y

),

= sz
=

I/ 2

and, Art. 168,

Eliminating,

we

parallel to the given lines.

a point on the

is

Fig. 81,

first line,

2
x"

+y +z\
2

find

and since
have

vi

r/
P"

(x

x"

y"

is

y",

")

=s
= M,
=x +

L"

+s +
2

_~

vi

+ *+T

a point on the second line,

we

z"

and, Art. 168,

Since

we have

"

vi + s +

245

"

y"

-f-

^ //2

Hence,

Vl +

s*

But, Art. 169,

L/

Substituting these values


reducing,

we have
_

Vl +

2
.9

for the required


expression.

in

equation

Vl + s
(3),

/2

Art. 186,

12

and

SOLID ANALYTIC GEOMETRY.

246
COB.

1.

If

<f>

= 0,

are parallel

the lines

and equation

(1)

becomes
i

T= =4J^===.

Clearing of fractions and squaring,

we have

and
Performing the operations indicated, transposing
have
we
lecting,

of these quantities cannot be


hence
is equal to zero

But the sum of the squares

each separately
equal to zero unless

<

s?

t,

col

st

=s

(2)

The first
lines.
are the conditions for parallelism of space
in
lines
two
space are
two of these conditions show that if
are
their projections on the co-ordinate planes
parallel, then
conse
mere
a
is
s
third condition
t)
parallel

also.

The

(sf

and
quence of the other two,

may

be omitted in stating the

conditions for parallelism.


to each
90, the lines are perpendicular
COB. 2. If 9

other,

and equation

(1)

becomes
1

hence
is

+ ss +
=

+ ** +

it

3
>

the condition for perpendicularity in space.


with any one ot
Since the angle which a line makes

188.
of the angle which the
the co-ordinate axes is the complement
to which that axis is
line makes with the co-ordinate plane
the complements of
p, y
if we let
ft y be
perpendicular
,

respectively,

we have

for the sines of the angles

co-ordinate planes.

which a space

line

makes with the

THE STRAIGHT LINE.

247

TRANSFORMATION OF CO-ORDINATES.
189. To find the equations of transformation from one
system of co-ordinates to a parallel system, the origin being
changed.

FIG.

82.

Let X, Y, Z be the old axes and X Y Z the new.


Let P be any point on the locus CM. Draw
PB,
,

OL

||

to

OZ and

meeting

XOY

in B,

A K,
B and L. Draw BE
Y draw LX to BE

and produce it to A BE will be to


and LE to OX. Then (OA, AB,
BP)
;

||

||

||

y,

z)

are the

old co-ordinates of the point P.

(O

A A B B P) =
,

(a;

are the

new

co-ordinates of

the point P.

(OX, XL, LO )
new origin
From the figure
.

(a, b, c)

are the old co-ordinates of the

SOLID ANALYTIC GEOMETRY.

248

OA = ON + O A AB
,

hence

-f-

==

NL + A B BP = LO + B P
,

=b

-\- ?/,

=c+z

are the required equations.

To find the equations of transformation from a rec


tangular system in space to an oblique system, the origin being
190.

the same.

FIG.

Let OX, OY,

OZ

83.

be the old axes, and

OX OY OZ
,

the

new.

Let u

OZ

Let

OZ

be the angles which

OX

makes with OX, OY,

u",

fi",

f be

the angles which

OY

makes with OX, OY,

respectively.

Let

OZ

ft,

respectively.

"

",

f"

be the angles which

OZ makes

with OX, OY,

respectively.

Let

be any point on the locus

CM.

Draw PB and PB

THE STRAIGHT LINE.


||

249

OZ and OZ respectively, and let B and B be the points


which these lines pierce the planes XOY and X OY

to

in

Draw B

OY

to

||

(OA, AB, BP)


point P.

(OA

then

A B B P) =

(x

are the old co-ordinates of the

y, z)

(x,

new

are the

co-ordinates of the

point P.

From

P,

on the X-axis

and

the perpendiculars PA,

let fall

then from the

figure,

BT>,

AL

we have

OA = OL + LD + DA
and DA are the projections of OA A B and
PB respectively on the X-axis, and each, therefore, is equal
to the line whose projection it is into the cosine of the
angle
which that line makes with the X-axis. (See Art. 169 (1) .)

LD

But OL,

OL = OA
=x
i.e., OL

cos

.-.

cos

LD = A B cos
DA = B P cos
LD = y cos
DA = z cos
,

"

",

hence, substituting, we have


r
X
x COS
-f- y COS

Similarly

=
=
x
y
z = x

cos

"

</

p+y

n"

-f z COS a

p +
-f if cos f + z

cos y

cos

cos
f

cos

"

p"

...

(1)

Y"

Of the nine angles involved in these equations, six only are


independent, for since the old axes are rectangular, we must
have (See Art. 169, equation
cos 2

cos 2

"

Con.

1.

we must have
cos

cos

cos

cos

"

cos

"

"

2)

the

new axes

to be rectangular also

in addition to equation
(2) the following condi
See Art. 186, Cor.
:

-f cos /? cos
cos
cos

"

y"

"

we suppose

tional equations

cos

"

If

).

2
-f cos /
cos 2

$"

cos 2

(2)

=1
+
p
cos
=1
+
+
cos
+ cos
+
f=1
cos 2

+
+

p
cos p

/3"

+ cos f cos =
+ cos f cos =
+ cos f cos y =
7"

p"

cos

/"

"

/?

(3)

"

Hence, in this case, only three of the nine angles involved


in equation
(1) are independent.

SOLID ANALYTIC GEOMETRY.

250

THE CONIC SECTIONS.


191..

The CONIC SECTIONS, or, more simply, THE CONICS,


from the surface of a right circular cone by

are the curves cut

a plane.
We wish to show that every such section is an ellipse, a
Art.
or one of their limiting cases.
parabola, an hyperbola,
146.

To deduce the equation of the

192.

FIG.

Let

any element

Draw DP

PK

to
||

84.

CAEA C be the conic surface,


CA about OZ as an axis.

the element

OZ,

as
||

KB

meet the base

generated by revolving
Let P be any point on

= c and OEC = 6.
and
intersecting OZ in D; draw
XY-plane
and K producing it to
to OY, and join

CE

to

conic surface.

let

OC

||

circle in E.

Then (OB, BK, KP) = (a?, y, z) are the co-ordinates


From the similar triangles COE, CDP, we have

50.-00_tan..
DP

OE

.(1)

of P.

THE STRAIGHT LINE.


DC = 00 - PK = c -

But

z,

and

251

DP = OK = Vz +
2

hence,
z

i.e.,

is

(c

z)

= tan
=

(x

+ if)

tan

2
.

(2)

the required equation.

193.

To find the equation of the


and a plane.

intersection of

a right

cir

cular cone

GALA

be the cone and


OY the cutting plane. Let
angle which the cutting plane makes with the
plane of the cone s base,
Let P (x, y, z) be any point on the curve of intersection

Let

X OX,
BPB

the

We

<p.

wish to find the equation of this curve when

OX as axes.
Draw PD to OY PL and DK

referred to

OY,
||

(OK, KL, LP)

(x, y, z)

||

to

OZ

then

are the space co-ordinates of P,

SOLID ANALYTIC GEOMETRY.

252

= (x

and (OD, DP)


to

OX

y) are the co-ordinates of

P when referred

OY.

From

OD cos

KL =

the figure

PL = KD = OD sin

DP,

qp,

OK =

go.

i.e.,

= x s m Vi

z
V>

=x

cos V-

But these values of x, y, & must subsist together with the


curve of
equation of the conic surface for every point on the
and
intersection; hence substituting in (2), Art. 192, reducing
2
2
2
we
tan.
cos
sin
that
have,
cp
?,
dropping
y
remembering

accents,
2
?/

tan

l9

+x

cos 2

+2 casing

tan

<p(tan

(1)

for the equation of the intersection.


By giving every value to y from

value from

to

equation

oo,

(1)

to 90 and to c every
can be made to represent every

section cut from a cone by a plane except sections


planes that are parallel to the co-ordinate planes.

COR.

1.

Comparing

(1)

= tan.

= cos

with

(1),

Art. 138.

we

made by

find

2
(JP

(tan

tan

2
g>)

Hence, equation (1) represents an ellipse, a parabola, an


Art.
hyperbola or one of their limiting cases according as,
146.
b

4 ac

<

IP =.

Case

and

1.

>

>

If 6

hence,

and

>

We

<p

this
(0, 0)

>

<p.

ellipse.

4 ac

hence

it is

find this supposition in (2) gives a


4 ac, i.e., the intersection is

>

an

<

the equation resulting from introducing


the point
(1) can only be satisfied by

0,

supposition in
;

ac.

the equation of two imaginary lines inter

secting at the origin.

THE STRAIGHT LINE.


]f

= 0,

cp

equation

that

CASE
c

2.

.-.

qp

and

2
?/

which

is

tan

6>

=c

2
,

a circle.

is

Hence the

This supposition in

g>.

= 4c.

Z>

(9

If

+x

the intersection

is,

becomes

(1)

if tan

From

0.

tan

2
<9

253

is

a parabola,

i.e.,

the equation of the X-axis


a straight line.
90and<?= oo,then the cone becomes a
cp
cylinder,

= =

If

and the cutting plane is perpendicular to its base.


section is therefore two parallel lines.
CASE 3. 9
This supposition makes a
cp.

The

<

>

If 6

Hence the

ac.

<

q>

and

c
if-

intersection

then

tan

(1)

=x

is

0,

>

.-.

and

>

we have

(1),

gives a

(2)

intersection

inter

<

an hyperbola.

becomes

cos 2

(tan

9)

tan

<p

0)

which is the equation of two intersecting lines.


CASE 4. Planes to the co-ordinate planes.
to XY-plane.
Let z = m be the
(a) Plane
\\

\\

such a plane. Combining


surface, we have

it

equation of
with the equation of the conic

tan

which
(b)

is

the equation of a circle for

Plane

||

such a plane.

or

7/

tan

2
which, since b
values of n.

(c)

Plane

a plane.
tion
a;

\\

all values of m.
Let x = n be the equation of
Combining with (2), Art. 192, we have
to

YZ-plane.

>

to

z?

+ 2 cz + n

ac, is

tan

XZ-plane. Let y

z2

tan

(4)

the equation of an hyperbola for

Combining with
2

...

= p be

(2), Art. 192,

+ 2 c +^

tan

all

the equation of such


after reduc

we have
c

(5)

SOLID ANALYTIC GEOMETRY.

254
which, since
values of p 2

2
>

ac, is

the equation of an hyperbola for

all

Hence, in

of the cutting plane, the


possible positions
an ellipse, a parabola, an hyperbola, or one of

all

intersection is

their limiting cases.

NOTE.

Equations

(3),

(4),

(5)

of

case

4 are the equa

on the
tions of the projections of the curves of intersection
the
But
are
projection of any
parallel.
planes to which they
to the given
curve
a
is
a
on
equal
parallel plane
plane curve
the conclusions of case 4 are true for the
curve

hence

curves themselves.

We have defined the conies, Art. 191, as the curves


the surface of a right circular cone by a plane, and
from
cut
found and discussed their
assuming this definition we have
194.

general equation, Art. 193.


A conic, however, may be otherwise defined as the locus
in a plane that the ratio of its
generated by a point so moving
is always constant.
distance from a fixed point and a fixed line

195.

To deduce the general equation of a

conic.

FIG.

the basis of the


Let us assume the definition of Art. 194 as
the fixed line.
Let F be the fixed point and
operation.
of its path.
Let P be the generating point in any position

OY

THE STRAIGHT
Draw FO
Draw PL

OY, and take OY and OX as co-ordinate axes.


OY, PD J_ to OY, and join P and F. Let OF

to

||

FP =

definition

From
but

FL

DP =
2

e*x

2
.

These values

in (1) give
e

or, after

a constant.

FPL, FP == FL + PL 2 ;...(!)
2
(OL - OF) = (x - p)\ LP 2 = f- and FP 2

triangle
2

255

J_ to

= p.
By

LINE.

x2

reduction,

y-

-p)

(x

-e

(1

x2

+ if

- 2px + p* =

(2)

for the required equation.

COR.

Comparing
a

hence

CASE

1.

(2)

4 ac

with

1, b

(1),

= 0,
4

c
2

e )

(1

we

Art. 138,

and

The fixed point not on

= (1
= 4 (e~

find

e ),

1)

the fixed line

(3)

i.e.,

not

zero.

If e

an

<

1, b

2
<

ellipse.

= 1,

If e

ac.;

= 4 ac

hence equation

(2) is the

hence equation

equation of

the equation of a

(2) is

parabola.
If e

>

1, 6

2
>

4 ac

hence equation

(2) is the

equation of

an hyperbola.

CASE

The fixed point


(2) becomes

2.

is

on the fixed

^2

line, i.e.,

= 0.

In this case

f+
If e

<

1,

(1

e )

(4)

equation (4) represents two imaginary lines inter

secting at origin.
If

= l,

equation

(4)

represents

one

straight line (the

X-axis).
If e

>

1,

equation

(4) represents

two straight

lines inter

secting at the origin.

Hence, equation
iting cases.

(2) represents the conies or

one of their lim

SOLID ANALYTIC GEOMETRY.

256

GENERAL EXAMPLES.
Find the point of intersection of the
2
x =
1
x = 2

1.

lines

and the cosine of the angle between them.


^%s.

Required the equation of the


to
2, 3) and is parallel

2.

(1

_92

(3, 5, 1)

line

y~l-V}What

3.

g>

^|

=2z

?/=

the angle between the lines

is

What

-.+

Ans.
4.

which passes through

_]_ 1 )

cos

is

the distance of the point

3, 2,

90.

from

1)

the line

y
5.

= 4z

-\-

makes equal angles with the co-ordinate axes


the angles which it makes with the co-ordinate

line

required

planes.

4y
2x
The equation of a surface is x -f y* +
_ 2 what does the equation become when the surface
2

6.

Qz

is

at

referred to a parallel system of axes, the origin being


2
16.
z
Ans. x 2
if
?

(1, 2, 3)

7.

Given the

the line on

XY

line

x
?

+
^

2
,

8.

Ans.

Required the distance cut


I

the projections of the line

j~_

of
required the projection

and the point on which the

co-ordinate planes.

off

__

line pierces the

in part, 2 y

on the Z and

+ = 4.
a-

axes by

^ x == 9

on Y^s

Ans.
?/

"

= -

_3o

6
.

THE STRAIGHT LINE.


How

9.

257

are the following pair of lines related ?

= 2z

= 2z-

10. What are the equations of the line which passes through
the origin and the point of intersection of the lines

What is
origin ? What
11.

on

the distance of the point (3. 2,


angle does this line make with

= 3z\

from the

4)
its

projection

XY ?
12. A

straight line makes an angle of 60 with the X-axis


and an angle of 45 with the Y-axis what angle does it make
Ans. 60.
with the Z-axis ?
;

What

13.

"1

are the cosines of the angles

make with
14.

which the

lines

through the point

line passes

angles with X, Y,

the co-ordinate axes ?

Z whose cosines

V2

are

and makes

(1, 2, 3)

1
?

res P ect

"

ively; required
(a) the

equation of the line,


the equation of the plane J_ to the line at the point,
to the
(V) to show that the projections of the line are J_
traces of the plane.
(b)

15.

The

directional cosines of

two

lines are

212 and

Xfrj
L

1,1. What

is

the cosine of the angle which they

2i

make with each other

Ans.

Cos

y>

3
-

+2 V2
b

16.

The projecting planes

=2y

-\-

2.

jects the line

What
on

YZ ?

is

of a line are x

=3z

and

the equation of the plane which pro


3.
2y
Ans. 3 z

SOLID ANALYTIC GEOMETRY.

258

and YZ each form with


The projections of a line on
the
45
of
an
equation of the line
Z-axis
the
required
angle
which passes through (2, 1, 4) parallel to the line.

XZ

17.

Find the equation of the

18.

(3,.

2, 1)

and meets the

Given the

19.

line

lines

quired
of
(a) the value

line

which contains the point

* Z

^ _~3

at right angles.
[

and

**+_l\ }

x
y

\ J

re-

order that the lines may be parallel


order that the lines may be perpen
in
of s
(b) the value
dicular
the value of s in order that the lines may intersect.
s in

(c)

212
- -

directional cosines of a line are

required
o o
the co-ordi
the sines of the angles which the line makes with
nate planes.
20.

The

21.

Find the equations of the

the origin and


-

and
y
22.

2,

Find

is

1 )

line

which passes through

two lines
perpendicular to the
^
Ans. x

11
&

the

angle

+ By + C* = D

and

included

between

Ax + By +

If

_^_

3z

j-

2 z\

the two planes

=D

AA +

+B +
a

23.

j-

two planes are parallel show that the

C"

coefficients of

the variables in their equations are proportional.


24.

Find the condition for perpendicularity of the two

planes given in

Example

22.

Ans.

AA + BB + CO - 0.

SURFACES OF THE SECOND ORDER.

CHAPTER

259

IV.

A DISCUSSION OF THE SURFACES OF THE


SECOND ORDER.
By

A. L. Nelson, M.A., Professor of Mathematics in


ton and Lee University, Va.

EVERY equation

Washing

involving three variables represents a sur

If the equation be of the first


degree the surface will
be a plane. If the equation be of a higher
degree the surface
will be curved.
It is proposed in this
to determine
face.

chapter
the nature of the surfaces represented
by equations of the
second degree involving three variables. The most
general
2
2
form of the equation of the second degree is Ax 2
By/ -f- Cz

+ Day + Vxz + -Fyz +

Gx

+ Hy +

Iz

+K=

(1)

where

the coefficients A, B, C, etc.,


may be of either sign and of any
magnitude. Let us suppose the co-ordinate axes to be rectan

The form of equation (1) may be simplified


gular.
by a
transformation of axes. Let us turn the axes without chan
ging the origin.
The formulae

of transformation are
(Art. 190)

=X
y = x
z =x
X

z COS a
+ y COS
cos p + / cos $ + z cos
cos f + if cos / +
cos f

COS

"

a"

-f-

"

ft

Substituting these values, equation (1) becomes

AV* + By + C V + DVv/ + E
+ TV + K =
(2).
2

W+

F yV

+ GV + Hy

SOLID ANALYTIC GEOMETRY.

260

Since the original axes were supposed rectangular the nine


are connected by the three relations
angles
ft f etc.,

+ cos

cos 2
cos 2
COS 2 a
If

ft

2
4- cos
2
4- COS

"

"

ft"

2
4- cos

/J"

we take the new axes

able, the

2
4- cos

y"

2
4- COS

I.

= 1.
=

1.

/"

also rectangular,

which

desir

is

nine angles will be connected by the three additional

relations

cos a cos

"

-j-

cos a

cos
cos

"

cos

u"

cos

cos

ft

+ cos

"

/?

4- cos

cos

ft"

4- cos

ft"

"

ft

cos

f cos

f=

4- cos y cos

"

/?

-4-

f"

f cos

cos

y"

0.

= 0.
= 0.

This will leave three of the nine angles to be assumed arbi


Let us give to them such values as to render the co
trarily.
efficients D E and F each equal to zero in equation
(2).
The general equation will thus be reduced to the form
,

AV

or,

4-

By

4-

CV

4-

GV + Hy 4- IV 4- K = 0,

omitting accents,

Ax 2
In order

By 4 Cs
to make a
2

4-

+ GJB + Hy + I

Equation

a 4- #

(3) will

(b

+ //)

(3)

>

=b +y

r
,

f-

4- %

become

Y +B

(a 4- x

move the origin without changing


The formulae of transformation

will be (Art. 189)

K=

further reduction in the form of the

equation let us endeavor to


the direction of the axes.

4-

(b

+ y Y + C (c + * )* + G
+K=

4- I (c 4- *

(a

+x 4
)

0.

2
2
2
4- B&
Developing, omitting accents, and placing A
HZ
Ic
4- Ga
L, the equation takes the form
Ax 2 4- B// 2 C,? 2 (2 Att 4- G) x 4- (2 B^ +H) y 4- (2 Cc +1 )z

+
+ L = 0.

In order

and

c,

now

+K=
+

to give definite values to the quantities a,

which were entirely

2A

arbitrary, let us

= --1L_
2B

assume

= _ _JL or

b,

SURFACES OF THE SECOND ORDER.

=
If these values of
a,
reduces to the form

b,

and

Cc

+I=

(4)

be Unite, the general


equation

+ By + C* + L =
2

261

[A],
a form which will be set aside for further
examination.
It may be remarked that
equations (4) are of the first
degree, and will give only one value to each of the
quantities
a, b, and c, and there is therefore
only one position for the

new

origin.

If,

however, either A, B, or

become
distance.

be zero, then

a, b,

and the origin will be removed


This, must be avoided.

infinite,

Let us suppose
then assume 2 B6
assume 2 Aa
G

A=

0,

B and C are finite. We may


and 2 Cc + I = 0, but we cannot

while

+ H = 0,

= 0.

or c will

to an infinite

Having assumed the values of


assume the entire constant term

and

c as

indicated, let us
This will give

equal to zero.

+ Cc + Ga + Hb + I C + K = 0,
a = - g + Co + Kb + Ic + K

B6 2

or

G
and the general equation will be reduced
to the form
2
By + C* a + G* =
.

(B),

a second form set aside for


examination.
must observe that this last
proposed transformation
will also fail when G
0, that is, when the first
power of x
as well as the second
power of x is wanting in the general
equation.

We

And

without making the second


transformation we have a
examination, viz.

third form for

IV*

+ C* + Hy + I* + K =
a

(C)
Lastly, two of the terms
involving the second powers of
the variables
may be wanting, and the equation
then
.

(1)

becomes

Cs 2

Gx

+ Hy 4. Iz + K =

(D)

262

SOLID ANALYTIC GEOMETRY.

It is apparent, therefore, that

every equation of the second


degree involving three variables can be reduced to one or
another of the four forms

A* 2 + By
2
2
By + C*
B?/

We

will

with the

+ C.?
+ Gx

+ Cz* + L =
+ Gx =
(B)
+ H// + I* + K + Hy + Iz + K =

examine each of these forms


form

first

(A)

in order,

(C)

(D)

beginning

Ax 2

+ B?/ +
2

C* 2

(A)

This equation admits of several varieties of form


according
to the signs of the coefficients.

1.

A, B, and

2.

A, B, C, and L positive.
Two of the coefficients as

3.

positive,

and

negative in the

and

first

member.

positive,

C and L

negative,

negative.

Two

4.

and L

Ko

of the coefficients as

and

positive,

positive.

other cases will occur.

CASE 1.
Ax 2 + B?/ 2 + Cs 2 = L,
in which form all of the coefficients are

positive.

In order to determine the nature of the surface


represented
by this equation, let it be intersected by systems of planes

The equations
parallel respectively to the co-ordinate planes.
of these intersecting planes will be x
c.
Com
a, y
b, z
bining the equations of these planes with that of the surface,

we

find the equations of the


projections on the co-ordinate
planes of the curves of intersection.

When
"

"

=
y =
z =
x

Thus we

a,
b,
c,

By
Ax
Ax

2
2

+ C.? = L - Aa
+ Cz = L - IW
+ By = L - Cc
2

an
an
an

ellipse.

ellipse.

ellipse.

see that the sections parallel to each of the co

ordinate planes are ellipses.

SURFACES OF THE SECOND ORDER.


The

made

section

Aa

or a

>

<

by the

-j-

plane

=a

-~ and
imaginary
,

is

real

263

when

in the
contrary

case.

The
ft
<

=i=

<

Thus we

>

and imaginary when

see that the surface

lar parallelepiped

When
sections

When

ft

>

is

-J-

=b

is

y
=c

-[-

>

real

when

real

when

is

i/

enclosed within a rectangu

whose dimensions are

become
a

plane

made by the plane

section

i y ~7T

by the

an d imaginary when

V/~fT~>

The
c

made

section

or 4
points.

= 0, = 0,

and
by the co-ordinate planes
ft

By +
2

Ax
Ax

2
2

or

= 0,
C,-

we

L.

+ C,v = L.
By = L.
2

-f~

Vc

find the sections

the

made

SOLID ANALYTIC GEOMETRY.

264

These are called the principal sections of the surface. The


principal sections are larger ellipses than the sections parallel
to them, as is indicated by the magnitude of the absolute
term.

The surface

called the Ellipsoid.

is

may be generated by the motion of an ellipse of variable


dimensions whose centre remains on a fixed line, and whose
It

plane remains always perpendicular to that line, and whose


semi-axes are the ordinates of two ellipses which have the

same transverse

but unequal conjugate axes placed at

axis,

The axes of the principal sections


right angles to each other.
are called the axes of the ellipsoid.
c,

If

we

we

shall

represent the semi-axes of the ellipsoid by

a, b,

and

have

and the equation of the surface


Ax 2 + B?/ 2 -j- Cz 2
a2

b c

-f-

2 2

=L

becomes

2
-f-

a2b 2z 2

a 2b 2 c 2

These are the forms in which the equation of the ellipsoid


is

usually given.
If

we suppose B

= A, then b = a, and the


z
x + y
_
2

equation becomes

2~

and the surface

is

the Ellipsoid of Revolution about the axis

of Z.
If

A=B
2

-j_ ?/

If

-(-

= 0,

then a

== C,

=a

2
,

=b=

c,

and the surface

the axes 2

and the equation becomes


is

a sphere.

2
i/-^-,

reduce to zero, and the ellipsoid becomes a point.

SURFACES OF THE SECOND ORDER.

265

be negative in the second member, the equa


2
L will represent an imaginary
B?/ -j- Cz
will
be
no
there
and
geometrical locus.
surface,
Hence the varieties of the ellipsoid are

CASE

tion

If

2.

Ax 2

The ellipsoid proper with three unequal axes.


of revolution with two equal axes.
(2) The ellipsoid
The
sphere.
(3)
(4) The point.
(5) The imaginary surface.
CASE 3. In this case the equation takes the form
(1)

Ax 2 + By 2 - Cz 2 = L,
C, and L are essentially

which A, B,
positive.
Cutting the surface by planes as before, the sections will be,
2
L
Aa 2 a hyperbola, having its
when x
Cz 2
a, ~By
in

when a

transverse axis parallel to the Y-axis


parallel to the Z-axis

v/

when a

-j-

t/
V

but

A.

And when

>

<

-J-

the intersection becomes two straight lines whose pro-

.A.

jections on the plane of

When

b,

Ax 2

YZ pass through

Cz 2

=L

the origin.

B& 2 a hyperbola, with simi


,

lar conditions as above.

When
values of

c,

Ax 2

+ B?/ =
2

+ Cc

2
,

an ellipse real for

all

c.

Since the elliptical sections are all real, the surface


it consists of a
single sheet.

is

con

tinuous, or

The

principal sections are found by

= 0,
b = 0,
c = 0,
a

which gives By 2

Ax 2
Ax 2

"

Cz 2
Cz 2

+%

making successively

= L, a hyperbola.
= L,
= L, an ellipse.

The surface is called the elliptical hyperboloid of one


The equation may be reduced to the form
v?_
2

.y*_
2

^L
*

sheet.

SOLID ANALYTIC GEOMETRY.

266

may be generated by an ellipse


centre remains constantly on
whose
dimensions
is
whose
and
perpendicular to that axis,
plane
semi-axes are the ordinates of two hyperbolas
same conjugate axis coinciding with the Z-axis,
This surface

of variable

the Z-axis,

and whose

having the
but having
different transverse axes placed at right angles to each other.

FIG. B.

If

we suppose

A = B,

then will a

b,

and the equation of

the surface becomes

the hyperboloid of revolution of one sheet.


If

A=B=

C,

we have x 2

+ if - ^ = a

z
,

the equilateral

one sheet.
hyperboloid of revolution of
a right cone having an
If L
0, the equation represents
B this base becomes a circle.
base and if

elliptical

A=

SURFACES OF THE SECOND ORDER.


Hence we have the following

267

varieties of the hyperboloid

of one sheet.

The liyperboloid proper, with three unequal axes.


The hyperboloid of revolution.
3. The equilateral hyperboloid of revolution.
4. The cone.
2
= - L, where A, B, C, and
CASE 4. Ax 2 + B?/ 2 1.

2.

are essentially positive.

a,
Intersecting the surface as before we have, when x
Aa 2 a hyperbola having its transverse
L
QgZ
axis parallel to the axis of Z.

__

j>^2

When
its

ing

When
c

-j-

>

b,

Ax 2

Cz"

I/V C

c,

Ax 2

+ B// =

B6 2
-f

and imaginary when

sections between the limits


real

hyperbola hav

transverse axis parallel to the axis of Z.

beyond those

-j-

Cc 2 an ellipse real when


,

i/
V

-J- t

<

Since the

V C

are imaginary, but

two

limits, it follows that there are

distinct

sheets entirely separated from each other.


The surface is called the hyperboloid of two sheets.
The principal sections are found by making successively
2
2
a
0, which gives By
L, a hyperbola with its
transverse axis parallel to the Z-axis.

O
O

= 0,

which gives Az 2

L, a hyperbola with its

transverse axis parallel to the Z-axis.


c

= 0,

which gives Ax 2

The semi-axes

+ By =
2

of the

L, an imaginary ellipse.

section

first

Those of the second section are I/

V A

of

the imaginary section

The distances 2

i/-^_X
I

are

i/
V

and

Jj

V/ B
i

and

And
!/_.
V

1)

and

i /

those

2 4/1*1. and 2 \/
T

are

1).

are called the axes

SOLID ANALYTIC GEOMETRY.

268

FIG. C.

and
Eepreseiiting the semi-axes by a, b,
form
the
to
reduced
the equation of the surface may be
of the surface.

5 + -?
If

we suppose

A=B

then a

c,

&,

and the equation

duces to

the hyperboloid of revolution of two sheets.

re

SURFACES OF THE SECOND ORDER.


If

A=B=

C, the equation

becomes

+f_^= _

x*

269

a*

which represents the surface generated


by the revolution of
an equilateral hyperbola about

its transverse axis.


the
surface
becomes a cone having an
Finally,
elliptical base, and the base becomes a circle when
B.
We have, therefore, the following varieties of the
if

= 0,

A=

hyper

boloid of two sheets:


1.

2.

3.
4.

The hyperboloid proper having three


unequal
The hyperboloid of revolution.
The equilateral hyperboloid of revolution.
The cone.

We

will

now examine the second


2
2
By + C* + G x =

axes.

form,
.

(B)

Three cases apparently different present themselves for


examination.

B and C

positive

(2).

B, C, and

(3).

(1).

and

negative in the

first

mem

ber.

CASE

positive.

C and G negative.
The equation may be written

positive and

1.

B?/

in

which B,

+ O. = Gx
2

and

are essentially positive.


C,
Let the surface be intersected as usual
by planes parallel
respectively to the co-ordinate planes.

When

By

+O =

Ga, an ellipse real when a

and imaginary when a


0.
2
When y = b,
= Gx

>

0,

<

B6 2

parabola with

its

axis

its

axis

parallel to the axis of X.

When

c,

By

= Gx -

a parabola

with

parallel to the axis of X.

The
and

principal sections are found by

When
When
origin.

making a

= 0, b = 0,

0.

a
b

= 0, By + C,? =
= 0. C.s = Ga;, a
2

0,

a point, the origin.


its vertex at the

parabola with

SOLID ANALYTIC GEOMETRY.

270

When c, =

2
G#, a parabola with vertex at the origin.
0, B?/
Since every positive value of x gives a real section, and
every negative value of x an imaginary section, the surface
consists of a single sheet extending indefinitely and contin

uously in the direction of positive abscissas, but having no


from the origin.
points in the opposite direction
The surface is called the elliptical paraboloid. It may be

dimensions
generated by the motion of an ellipse of variable
the
same
on
remains
whose centre
straight line,
constantly

and whose plane continues perpendicular to that line, and


whose semi-axes are the ordinates of two parabolas having a
common transverse axis and the same vertex, but different
to each
parameters placed with their planes perpendicular
other.

FIG. D.

in the
2. If we suppose G to be positive
form
the
take
will
the
so that
equation

CASE

By

+C = 2

Go;,

first

member

SURFACES OF THE SECOND ORDER.

271

the sections perpendicular to the X-axis will become imaginary

when x

>

0,

and

real

when x

<

0.

In other respects the results are similar to those deduced


case

in

1.

will represent a surface of the same form


but turned in the opposite direction from the

Thus the equation


as in case

1,

co-ordinate plane of
If B = C, the

CASE

surface becomes the paraboloid of revolution.

By

3.

YZ.

-C =
a

Gx.

Intersect the surface

by planes as before.
Gz 2 = Ga, a hyperbola with transverse
and in the
axis in the direction of the Y-axis when a
direction of the Z-axis when a
0.

When

= a,

By

>

<

When

b.

in the direction of

Gx

B6 2 a parabola having its


the X-axis and extending to the left.

Cz 2

-\-

axis

Gx -j- Cc a parabola having its axis in


When z c B?/
the direction of the X-axis and extending to the right.
2

Since every value of x, either positive or negative, gives a


real section, the surface consists of a single sheet extending
This
indefinitely to the right and left of the plane of YZ.

surface

is

called the Hyperbolic Paraboloid. To find its princi


make x, y, and z alternately equal to zero.

pal sections

When x = 0, B^/ = Cz two straight lines.


When y = 0, Cz =
Gx, a parabola with axis to the left.
When = 0, B?/ = Gx, a parabola with axis to the right.
2

The hyperbolic paraboloid admits of no variety.


Now taking up form (C), By 2 + Cs a + Hy + Iz + K = 0,
we see that it is the equation of a cylinder whose elements
are perpendicular to the plane of YZ, and whose base in the
plane of YZ will be an ellipse or hyperbola according to
the signs of B and C.

+K=

The fourth form (D), Cz


Ga;
Hy -f Is
sents a cylinder having its bases in the planes
2

XZ

repre

and

YZ

SOLID ANALYTIC GEOMETRY.

272

VA

\ /

FIG. E.

parabolas, and having its right-lined elements parallel to the


and to each other, but oblique to the axes of and Y.

plane

XY

The preceding discussion shows that every equation

of the

second degree between three variables represents one or an


other of the following surfaces
the ellipsoid
1. The
ellipsoid with its varieties, viz.
:

the

of

proper,
ellipsoid
and the imaginary surface.

revolution, the

sphere, the

point,

SURFACES OF THE SECOND ORDER.


The hyperboloid

2.

of

one

or

two

sheets,

273

with

their

the hyperboloid proper of one or two sheets,


the hyperboloid of revolution of one or two sheets, the
varieties, viz.

equilateral hyperboloid of revolution of one or two sheets,


the cone with an elliptical or circular base.

The

3.

paraboloid, either elliptical or hyperbolic, with the

variety, the paraboloid of revolution.

The

4.

cylinder, having its base either an ellipse,

hyper

bola, or parabola.

The general equation of surfaces


be deduced by a direct method, as follows

Surfaces of Revolution.
of revolution

may

Ri

FIG. F.

Let the Z-axis be the axis of revolution, and


tion

x2

of

= fz.
Let

AB,

the

let

the equa

generating curve in the plane of XZ, be

be the point in this curve which generates the circle

SOLID ANALYTIC GEOMETRY.

274

PQR, and

be the radius of the

let r

The value

circle.

We

will

have

of r2

may also be expressed in terms of z from


the equation of the generatrix in the
plane of XZas follows
:

= CP =
2

r2

(JD =fz.

Equating these two values of r

we have

as the general equation of surfaces of revolution.


It will be observed that the second value of r 2

is the value
of x 2 in the equation of the
generatrix.
Hence, to find the
equation of the surface of revolution we have only to substi
2
tute x 2
if of the surface for x in the generatrix.

2
Surface of a Sphere.
Equation of generatrix x
Hence the equation of the surface of the sphere is

z2

K,.

Ellipsoid of Revolution.

Generatrix

Surface

JL_

x*

-f-

~^~

J!L

= 1,
1

-4-

a2

Similarly, the equation of the hyperboloid of revolution

***-*-*
Paraboloid of Revolution.
x2
4:pz, the generatrix.

=
+ IT = 4=pz, the
Cone of revolution,
z = mx
x2

surface of revolution.

-+- j3

~T
Hence
or

the generatrix,

~~i^~

2 -

(x

if)

(z

ft)

is

SURFACES OF THE SECOND ORDER.

275

EXAMPLES.
1.

9z
7/2

(^
3.

Of ^ 2

+ if = r- ?

Qf

if

Determine the nature of the surfaces x 2

+^

= 36 ?
+ =r ?

the locus in space of 4 x 2 -f 9 y-

is

- 16 y = 144 ?
+ 8x = o?

2.

What

.2

-\-

,-

y"-

-(-

4 z2

Of
Of

= 25,

79>

Find the equation of the surface of revolution about the


Z whose generatrix is z = 3 x -\- 5.

axis of
4.

Find the equation of the cone of revolution whose inter


2
= 9, and whose vertex
-f- y

section with the plane of XYisx 2


is

(0, 0, 5.)

5.

Determine the surfaces represented by

+ 4 y + 9 s = 36.
+ 4 y _ 9 # = 36
- 36.
x + 4y = 9
4 y + 9 2 = 36 x.
9z* = 36x.
2

x*

a;

14

DAY USE

RETURN TO DESK FROM WHICH


4S
This book

.OWED

^Sic

dbl
below.or
date stamped
is cfil on tfie last
on the date to which renewed. recall

Renewed books

are subject to immediate

LD

21-40m-5, 65

(F4308slO)476

General Library
University of California
Berkeley

^
vo

\*
t

Das könnte Ihnen auch gefallen